Unz评论•另类媒体选择$
美国主流媒体大都排除了有趣,重要和有争议的观点
 严申档案馆
智商还是数学/言语分裂?

书签 全部切换总目录添加到图书馆从图书馆中删除 • B
显示评论下一个新评论下一个新回复了解更多
回复同意/不同意/等等 更多... This Commenter This Thread Hide Thread Display All Comments
同意不同意谢谢LOL轮唱
这些按钮可将您的公开协议,异议,感谢,LOL或巨魔与所选注释一起注册。 仅对最近使用“记住我的信息”复选框保存姓名和电子邮件的频繁评论者可用,并且在任何八个小时的时间内也只能使用三次。
忽略评论者 关注评论者
搜寻文字 区分大小写  确切的词  包括评论

房源搜索

作为过去10多年左右一直关注HBD的人,这个话题经常引起的热情使我同时感到高兴和启发。 HBD人群的普遍想法是,他们对科学真相拥有深刻的见识,从而开辟了新的理解途径,而这些新的理解途径与那些被掩盖在令人迷惑的PC神话中的人完全相去甚远。 在大多数情况下,我都对这种观点表示同情。 本文并不是在挑战HBD的既定原则,而是要澄清围绕我多年来参与的各种HBD相关讨论的一些概念混乱,无论是直接讨论还是间接讨论。 我希望我的外行人的直觉可以激励其他人对这个话题有所不同。 令我惊讶的是,我发现通常情况下,比我聪明得多的人仍然倾向于以僵化和受限的方式来思考主题。

我经常听到人们谈论智商,好像这是一件大事。 毫无疑问,这在很大程度上归因于一般智力或g现象,据推测这可以解释为什么人们在各种心理子任务上的表现似乎是相互关联的。 如果您在一个认知领域中高于平均水平,那么您可能在其他认知领域中高于平均水平。[14]https://www1.udel.edu/educ/gottfredson/reprints/1998...or.pdf 我想提出一种关于智力和HBD的不同思维方式,它并不否认一般智力的重要性,而是认为对于精英表演来说,数学/言语分裂可能更重要。 数学/语言分裂只是一种现象,即某些人在认知上倾向于数学或言语推理,因此他们在现实生活中的追求反映了他们的认知特征。 特别是,了解数学/言语分裂的重要性可以阐明东亚人与欧洲人之间的潜在差异,或者更普遍地讲,东亚人与非东亚人之间的潜在差异,即使是假定的人类奉献者也常常无法很好地注意到我感到惊讶的差异。生物多样性。

聪明的东亚人

美国,中国,韩国或日本血统的东亚人通常被定型为聪明人。 与美国其他种族的成员相比,他们在学术上表现出色,并且无论是在硅谷的精英公司还是在美国各地的顶级科学实验室中,它们在现实生活中的STEM都占主导地位。 1987年,《泰晤士报》的一篇文章讨论了东亚移民取得的不成比例的成就,尤其是在数学和科学领域,并指出这是因为“在国外接受教育的亚裔学生以扎实的数学基础来到美国,但很少或几乎没有。英语知识。 他们还受到大学毕业后的一份好工作的承诺的影响。 “亚洲人认为数学和科学等领域的歧视将会减少,因为他们将得到更加客观的判断,”亨特学院的教育学教授雪莉•洪恩(Shirley Hune)说。 而且,她指出,教育投资的回报“在工程方面比在文科学位上更直接。”[1]http://content.time.com/time/subscriber/article/0,3...0.html HBD的支持者肯定会指出,智商是最终的根本解释,而不是文化或其他此类因素。 他们基本上是正确的,但我想强调指出,东亚人的基本特征是我所说的数学/语言分裂。 人们常常不经意间注意到东亚人对数学的亲和力,而不必将这种理解推导出逻辑上的结论。

长期以来,心理计量学文献记载了东亚人偏向数学而不是口头表达的倾向。 查尔斯·默里(Charles Murray)和理查德·赫恩斯坦(Richard Hernstein)在臭名昭著的《钟形曲线》一书中指出,东亚人倾向于在非语言方面胜于语言推理。 Vernon引用的一项研究表明,华裔美国人的平均表现智商为110,平均语言智商为97。[2]默里,C 和赫恩斯坦,RJ(1996 年)。 钟形曲线:美国生活中的智力和阶级结构。 300-301。 这是基于 1975 年使用 Lorge-Thorndike 智力测验对旧金山唐人街的中国儿童进行的测试。[3]弗林(J.Flynn)(2007)。 什么是智力? 超越弗林效应。 116。 理查德·林恩 (Richard Lynn) 等其他学者也一直指出,东亚人在数学/语言方面表现出明显的偏斜。 Lynn提出,蒙古语的智力从根本上具有“高通用智力(Spearman's g),高视觉空间能力和低语言能力”的特征。[4]https://www.sciencedirect.com/science/article/pii/01...901358 因此,相对于欧洲人,东亚人的语言智力平均较低,但在数量和空间推理方面却表现出色。

默里(Murray)提出,这种数学/语言上的分裂解释了为什么东亚人在社会科学,人文和法律领域的代表性不足,而偏向于科学和工程领域。[2]默里,C 和赫恩斯坦,RJ(1996 年)。 钟形曲线:美国生活中的智力和阶级结构。 300-301。 正如我稍后将要论证的那样,东亚人歪曲的认知特征不仅解释了为什么他们回避非科学,而且还解释了为什么即使在自然科学中,东亚人也对口头负荷的生活表现出明显的偏爱定量物理科学的原因。科学。 数学/语言上的分裂也阐明了东亚在美国精英学术比赛中的表现。 例如,在数学竞赛(如MATHCOUNTS或AIME / USAMO / IMO)中,东亚人的人数过多,但在拼写蜜蜂中的出席率却相对较低,最近,后者在口头流利的南亚人中占主导地位。

基于姓氏的信封估算值的背面表明,64年USAMO资格预选赛中约有65%至2016%属于东亚。[5]https://www.maa.org/sites/default/files/USAMO%20Qual..._0.pdf 相比之下,自1980年以来,只有一个具有东亚名字的人赢得了“拼字比赛”,而最后10位获胜者或共同获胜者都是南亚。[6]https://en.wikipedia.org/wiki/List_of_Scripps_Nation...mpions 同样,对291年进入华盛顿特区的2017名拼字比赛入围者的姓名和照片进行的分析表明,保守估计,大约有25名左右的东亚人血统,比率仅为8.6%。[7]http://spellingbee.com/meet-the-spellers/2017 (一些东亚姓氏实际上是越南人,我目前已将其排除在我对东亚人的定义之外。)

支持亚裔美国人认知上偏向数学推理的观点的进一步证据来自于亚裔美国人在标准化考试中的得分。 尽管数据已有十年之久,但博客作者Steve Sailer还是着重强调了在主要的标准化测试(例如GMAT,GRE,MCAT和LSAT)上的比赛表现。[8]https://www.unz.com/isteve/scores-by-race-on-lsat-gr...d-dat/

史蒂夫·塞勒(Steve Sailer)的比赛标准化考试成绩

Steve Sailer 按种族划分的标准化考试成绩?

一个显而易见的事实立即显而易见。 亚裔美国人在数学而非语言推理测试中的表现要好得多。 虽然他们在 MCAT Verbal 等高语言负荷测试中落后于白人,但在 GRE Math 等定量测试中,他们相对于白人表现出色。 事实上,尽管美国白人一再抱怨东亚人在玩弄系统并人为地夸大他们的考试成绩,但东亚人在标准化考试中的表现反映了与心理测量文献所表明的相同的数学/语言分裂这一事实表明,实际上这些测试产生的结果正是人们所期望的。

科学部分地根据一致性原则运作。 当各种独立的证据来源都集中在同一个基本事实上时,这表明所考虑的数据在更广泛的连贯理论的背景下是有意义的。 亚裔美国人标准化考试成绩是紧张准备或作弊的结果,因此需要一些反驳解释的异常现象,这种想法变得不那么可信。 与异常的亚裔美国人考试成绩相反,它们实际上揭示了智商测试多年来一直告诉我们的事情。 在口头测试中,东亚人的平均得分低于白人。 主要区别在于,他们在定量能力测试中往往要好得多。 (更不用说大量的实证文献表明,诸如 SAT 之类的相对 g-loaded 测试无论如何都不太适合进行广泛的准备,尽管有些人似乎确信 Tiger Mothers 以某种方式找到了破解所有那个或其他东西,可能是通过魔法泥土。正如史蒂夫·许指出的那样,“即使对这个话题进行随意调查也表明, 至少平均而言,即使通过大量的努力也很难轻易提高SAT分数。”[39]http://infoproc.blogspot.com/2012/02/test-preparati...s.html)

天才游and和斯皮尔曼的收益递减定律

当我们读到历史上不平衡的天才时,数学/语言拆分的重要性就变得很明显。 很多这样的例子比比皆是。 我想特别关注理查德·费曼 (Richard Feynman) 和特伦斯·陶 (Terence Tao) 两个,以说明杰出的人在生活的各个方面不一定都同样出色。 当然,一般智力或 g 表明他们可能在大多数(如果不是所有)认知能力领域都高于平均水平,但我相信这没有抓住要点。

已故的天才理查德·费曼(Richard Feynman)
已故的天才理查德·费曼(Richard Feynman)

理查德·费曼 (Richard Feynman) 是传奇,不仅因为他作为理论物理学家的贡献,而且还因为他 125 的 IQ 平平。[9]https://en.wikipedia.org/wiki/Richard_Feynman#Education (我也想,因为他总体上对生活充满幽默感和狂热的热爱,并且如果有人相信伪造的故事,那是因为我们的好主人罗恩·恩茨(Ron Unz)遭受了一次残酷的物理打击。)[44]http://web.cos.gmu.edu/~vkunkel/feynman1.html 这应该是一个关键数据点,驳斥了智商测试作为现实生活成就的有用预测指标的普遍效用。 如果即使是像费曼这样的知识巨头,其智商测试也只有 125 左右,那么智商测试实际上有多大用处,争论就这样结束了?

实际上,Feynman表现出清晰的数学/语言分裂,经常报告的适度IQ得分125可能仅仅是口头测试的结果。 例如,维基百科指出:“ 1939年,费曼(Feynman)获得了学士学位,并被任命为普特南研究员(Putnam Fellow)。 他在普林斯顿大学的物理学研究生入学考试中取得了完美的成绩,取得了空前的成就,并且在数学上也取得了优异的成绩,但在历史和英语方面却表现不佳。[9]https://en.wikipedia.org/wiki/Richard_Feynman#Education 同样,物理学家史蒂夫·许(Steve Hsu)怀疑“费曼在许多口头测试中会得分接近上限。 他经常犯语法错误,拼写错误(甚至是物理学中常用的单词)等。有时他不知道 含义 他周围其他人使用的术语(甚至是物理学中常用的单词)。”[10]http://infoproc.blogspot.com/2017/09/feynman-schwin...s.html

另一位神童陶天Tao(Terence Tao)
另一位神童陶天Tao(Terence Tao)

另一位神童陶哲轩,当今最杰出的数学家之一,也反映了费曼的明显认知偏差。 正如那些在他年轻时研究他的人所指出的那样:“毫无疑问,Terry Tao在数学上几乎令人难以置信地推理,并且快速地学习数学和相关学科。 他在8岁时参加的澳大利亚数学竞赛和大学理事会学术能力测验(SAT-M)的数学部分中的表现令人赞叹。 他是第一次参加60项60分钟的选择题SAT-M。 在美国,只有1%的大学入学男性12年级学生在此上的得分为750分或更高(College Board,1985)。 他得到了760分。据我所知,只有另外一个8岁的孩子也做到了。 那个住在芝加哥郊区的男孩第五次参加考试! 他在800岁之前就获得了10分的成绩。 特里没有在9岁时接受SAT-M的重新测试,因为那似乎没有必要。然而,在8岁10个月时,当他同时参加SAT-M和SAT-口头考试时,特里在后者上仅获得290分。 只有9%的大学入学男性12年级学生在SAT-V上得分不超过290分; 机会得分大约为230。在M的最低第10个百分位点与V的第99个百分位点之间高出9个点的差异表示大约3.7个标准差的差距。 显然,特里在数学推理项目上的表现要好得多(请参见附录),比阅读段落并回答有关它们的理解性问题或弄清反义词,口头类比或缺少单词的句子要好得多。”[11]http://www.davidsongifted.org/Search-Database/entry...A10116

《纽约时报》在2007年的一篇文章中也提到了陶:“他5岁那年就读于一所公立学校,他的父母,管理人员和老师为他建立了个性化的课程。 他按照自己的步调学习每个科目,迅速提高了数学和自然科学的几个年级,同时保持与其他学科的年龄段接近。 例如,在英语课上,当他不得不写论文时,他变得很慌张。 他说:“我从来没有真正掌握过这一点。” 这些非常模糊,不确定的问题。 我总是喜欢有明确的操作规则的情况。” 特里被派去写一个关于家里发生的事情的故事,特里在一个房间里走来走去,并详细列出了里面的物品。”[12]http://www.nytimes.com/2007/03/13/science/13prof.html

如上所述,最有可能的道和费曼都明显偏向于言语,而倾向于空间/量化能力。 天才的认知能力肯定会有所偏差,不一定在所有事情上都具有天赋。 确实,未能理解这一事实很可能导致了心理计量学最大的假否定之一。 1920年左右,心理学家刘易斯·特曼(Lewis Terman)尝试通过对智商进行测试来寻找加利福尼亚州的聪明青年。 跟踪那些得分最高的1%的人,以进行进一步的纵向研究。 尽管成为未来的诺贝尔物理学奖得主,威廉·肖克利和路易斯·阿尔瓦雷斯都未能入选。[13]https://www.scientificamerican.com/article/recognizi...intel/ 就像费曼臭名昭著的 IQ 分数只有 125 一样,这再次被认为是智力测试局限性的证据。 特曼怎么可能没能认出这两个崭露头角的神童呢?

威廉·肖克利和路易斯·阿尔瓦雷斯。 众所周知,两人都未能在特曼的智商测试中取得初步成绩。
威廉·肖克利和路易斯·阿尔瓦雷斯。 众所周知,两人都未能在特曼的智商测试中取得初步成绩。

尽管这种怀疑从表面上看是合理的,但心理学家大卫·卢宾斯基和卡米拉·本博提供了一个更有说服力的解释,他们认为“特曼的斯坦福-比奈智商测试中的许多项目,与许多现代评估一样,未能利用被称为空间能力。 关于认知能力的最新研究正在加强几十年前一些心理学家的建议:空间能力,也称为空间可视化,在工程学和科学学科中起着至关重要的作用。 然而,口头上更多的智商测试以及当今使用的许多流行的标准化测试并不能充分衡量这一特征,特别是在那些最有才华的人中。[13]https://www.scientificamerican.com/article/recognizi...intel/ 如果对阿尔瓦雷斯(Alvarez)或肖克利(Shockley)进行空间定向或定量定向的能力测验,很难想象他们中的任何一个都不会做出Terman的榜样。 这种异常现象仅仅是情报界未能充分理解数学/言语分裂的结果。

当然,所有这些都没有暗示总体智商或一般智力的概念不是有意义的概念。 相反,我怀疑总体智商对于分析更大范围的人口更有用,而数学/语言拆分对理解认知分布右尾的个人表现更有用。 正如琳达·戈特弗雷德森(Linda Gottfredson)这样的心理学家所争论的那样,通用情报的用处在于这样一个事实,即“半个世纪的军事和民用研究已经融合在一起,画出了沿着IQ连续体的职业机会的肖像。 成人智商分布前5%(智商125以上)中的个人基本上可以训练自己,而且很少有工作在心理上无法实现。 平均智商(介于90到110之间)的人在大多数专业和行政级别的工作上没有竞争力,但在美国经济中的大多数工作岗位上容易接受培训。 相比之下,智商排名靠后的百分之五(低于5岁)的成年人很难训练,而且在能力方面,任何职业都没有竞争力。”[14]https://www1.udel.edu/educ/gottfredson/reprints/1998...or.pdf

从,说明生活中的结果如何与智商相关。
[14]https://www1.udel.edu/educ/gottfredson/reprints/1998...or.pdf,说明生活中的结果如何与智商相关。

不过,在分析尾端人才时,我认为最可能的特定认知亚因素起着更重要的作用。 实际上,斯皮尔曼的收益递减定律表明,“ g 在一个人口群体的所有子群体中可能并不统一。 Spearman的收益递减定律(SLODR),也称为认知能力分化假说,预示着,在较聪明的亚组中,不同认知能力之间的正相关性较弱。 进一步来说, 斯洛德 预测g因子占g因子较高分数的认知测试得分中个体差异的比例较小。”[15]https://en.wikipedia.org/wiki/G_factor_(psychometric...eturns

评论家Gwen在Infoproc博客上暗示了这一现象的可能的神经学基础,并指出“我有一点推测:神经影像学研究似乎始终指向全球连通性的效率,而不是单个地区的效率或其他特征。 您可以将其解释为各种各样任务的总因,因为在执行任务时,仅因协调困难而使它们都或多或少受到了阻碍; 因此,可能导致Spearman的原因是全局连接变得尽可能高效,不再是大多数任务的瓶颈,而是各个大脑区域开始主导其他性能改进。 因此,在一定程度的全球通信效率上,存在一个通用的智力因素,但是诸如空间与语言之类的特定能力却分崩离析,不再具有常见的瓶颈,而且大脑倾斜也更加清晰地显现出来。”[10]http://infoproc.blogspot.com/2017/09/feynman-schwin...s.html 这看起来似乎很合理。 我们希望那些比我们聪明得多的人将在未来几十年中慢慢探究这些问题的根源。

总而言之,我想在这里指出的观点应该是明确的。 许多人的认知特征相当不平衡。 历史为我们提供了具有启发性例子的知识巨人的例子,这些巨人在认知上存在偏差。 在某一领域成为天才并不一定意味着您在所有其他领域都具有同等的才能。 SLODR建议,在考虑的亚组越聪明的情况下,一般智力在解释总认知变异方面的相关性就越小。 尽管g对于琳达·戈特弗雷德森(Linda Gottfredson)所引用的研究而言可能对较大人群的总体广泛卒中分析有用,但精英表现可能更依赖于特定的认知亚因素。 的确,当刘易斯·特曼(Lewis Terman)未能将阿尔瓦雷斯(Alvarez)或肖克利(Shockley)标记为认知精英青年时,忽略数学/语言拆分可能导致了心理学领域最臭名昭著的错误否定之一。

讨论了数学/语言分裂在理解历史神童轨迹方面的重要性之后,现在让我们转向这一现象如何从总体上阐明现代东亚崛起轨迹的话题。

中国的科学快速崛起和东亚科学

21世纪最有趣的现象之一st 世纪以来,中国科技迅速崛起。 文革动乱之后,中国在过去几十年里一直在快速投资科技。 据估计,在 2017 年期间,中国在研发上的支出约为 279 亿美元,比上年增长 14%,是几十年来从极低的起点开始快速增长的研发成果。[16]https://www.cnbc.com/2018/02/26/china-spent-an-estim...r.html

但是,正如随着中国科学技术兴起之后最谨慎的人所知道的那样,中国显然偏爱定量领域,特别是物理,化学,工程,数学和计算机科学。 正如澳大利亚学者西蒙·马金森(Simon Marginson)所指出的那样,“在2000年,中国仅撰写了0.6%的化学论文,就Web of Science而言,其被引率为全球最高的百分之一。 仅在12年后的2012年,中国发表论文的比例就占领先论文的16.3%的2014%,是美国的一半,这是惊人的提高速度。 在工程,物理和计算机领域也有类似的模式,其中中国发表的论文比美国和数学高出百分之一的论文(NSF,XNUMX年)。中国,台湾,韩国,日本以及新加坡在某种程度上都将研究重点集中在物理科学和相关的应用领域,例如工程,计算机和材料。 在韩国和日本,这支持先进的制造。 中国还强调支持加速现代化的研究:能源,城市化,建筑,交通和通讯。 在这个阶段,医学和生命科学还很薄弱。”[17]http://www.researchcghe.org/perch/resources/publica...p9.pdf

同样,《自然》杂志在2014年指出,中国的WFC高达90%来自物理科学和化学领域,而不是生命科学领域。[18]https://www.nature.com/articles/515S73a 中国2017年在《自然》杂志上的最新产出基本上呈现出相同的偏差。[46]https://www.natureindex.com/country-outputs/china 这与美国或英国等倾向于生命科学和医学的国家的科学产出形成鲜明对比。 例如,联合国教科文组织在 2010 年报告说,虽然日本在物理、化学、工程和技术方面具有优势,但美国和英国往往专注于生物医学研究和临床医学。[19]教科文组织(2010)。 联合国教科文组织《 2010年科学报告:世界科学的现状》。 12

从中可以看出,中国在自然界的产出显然偏向于物理科学和化学,而与生命科学背道而驰。
[46]https://www.natureindex.com/country-outputs/china, 中国在 Nature 上的产出明显偏向于物理科学和化学,远离生命科学。

进一步的证据支持莱东排名的说法,即东亚国家的科学输出明显偏向量化领域。莱顿排名利用Clarivate Analytics编制的Web of Science的书目数据来确定哪些机构发表了各种论文中影响最大的论文。领域。[20]http://www.leidenranking.com/ranking/2017/list 莱顿将论文分为五个主要领域,分别是生物医学和健康科学,生命与地球科学,数学和计算机科学,物理科学和工程学以及社会科学和人文科学。

根据引用率最高的10%中的论文数量,东亚大学相对于其他三个类别显然在数学,计算机科学,物理科学和工程学方面表现出色。 在2012-2015年期间,按引用率排名前10%的论文总数进行排名,东亚在物理科学和工程学排名前5的大学中有10所,在数学和计算机科学领域排名前8的大学中有10所。

莱顿市10年至2012年数学和计算机科学领域前2015%的论文数量排名中的顶尖学校。
莱顿市10年至2012年数学和计算机科学领域前2015%的论文数量排名中的顶尖学校。
根据莱顿10-2012年物理科学与工程领域前2015%的论文数量排名的顶尖学校。
根据莱顿10-2012年物理科学与工程领域前2015%的论文数量排名的顶尖学校。

相比之下,在查看生物医学和健康科学领域前 10% 的论文总数时,排名最高的东亚大学是 48岁的上海交通th。 就生命科学和地球科学而言,排名最高的东亚大学是 浙江20th. 而在社会科学和人文科学方面,东亚大学排名最高的是 新加坡国立大学80岁以下th 的地方。[20]http://www.leidenranking.com/ranking/2017/list 东亚大学在定量领域和口头领域之间产生的高影响力工作之间的差异很难说清楚。 相反,西方国家往往在生命科学,医学,社会科学和人文科学领域表现卓越。 英国在最新的QS世界大学学科排名中的表现进一步增强了这一点,正如所指出的那样,“该学科高度集中于艺术与人文学科,生命科学以及社会科学与管理”。[43]http://www.universityworldnews.com/article.php?stor...510534

需要注意几个问题。 首先,与其他东亚地区相比,日本在生命科学领域的影响更大。 从免疫学到细胞生物学再到再生医学,它在生物科学的各个领域都有实力,任何熟悉诸如Yoshinori Ohsumi,Mazaki Kazutoshi Mori,Shimon Sakaguchi,Tasuku Honjo或Tadamitsu Kishimoto等名字的人都可以证明。 最著名的是,山中伸弥(Shinya Yamanaka)早在2006年就发明了iPS细胞,催生了一个崭新的再生医学领域,日本已在该领域确立了其世界领导者的地位。[21]https://www.ft.com/content/254853b2-8f23-11e7-9084-d...42ba93

Shinya Yamanaka,2006 年首次创造诱导多能干细胞的科学家。
Shinya Yamanaka,2006 年首次创造诱导多能干细胞的科学家。

另一方面,与日本或韩国相比,中国在数学和计算机科学方面发表的影响更大的著作。 例如,《美国新闻全球》最近将清华大学列为全球排名第一的计算机科学计划。[22]https://www.usnews.com/education/best-global-univers...cience 除此之外,总的来说,所有东亚国家都倾向于使用物理,化学,材料科学和工程学等领域。

那么我在这里的主要预测是,基于HBD,我不希望中国或东亚在生命科学和医学或其他口头上的科学领域能与盎格鲁圈媲美。 也许中国可以在发展生命科学各个领域的实力方面反映日本。 鉴于其人口众多,这的确可以转化为生物学和生物医学领域非平凡的高端产品。 但是,随着该地区科学的成熟,东亚国家的核心优势将主要集中在物理或化学等定量领域,而我预测,这是该地区在未来几年中将大放异彩的地方。 中国最近在量子密码学领域的尝试就是一个例子。[40]https://www.technologyreview.com/s/610106/chinese-sa...nents/

因此,尽管有些人指出了物理科学,生命科学和社会科学各个领域的整体科学产出,证明了东亚人没有得到应有的代表,但更为细微的理解表明,实际上东亚人只是对他们天生擅长的事物着迷。 像中国和日本这样的国家在心理学或临床医学等领域相对于物理学或化学领域都表现出色,并且在前者而不是后者方面不成比例地发表,这几乎不是一个谜。 这只是底层HBD的反映。

超越科学:技术与工程

东亚在技术和工程方面也很出色。 东亚国家是国际专利大国,如果您没有注意到,实际上,先进的现代消费类电子硬件几乎每一个都是在东亚制造的。 这是 HBD 相关讨论中经常被忽略的一点。

《金融时报》指出,“日本仍然是一个创新强国,根据专利地域分析显示,东京-横滨是世界上最大的此类集群。 这项研究来自总部位于日内瓦的世界知识产权组织(Wipo),该组织分析了根据《专利合作条约》在950,000年至2011年间发布的所有2015件国际专利申请中指定的发明人的住址。 另外两个日本集群,大阪-神户-京都和名古屋,在全球前十名中。 结果还显示,东亚其他地区的发明活动十分活跃,中国的深圳-香港在Wipo的排名中位居第二,领先于加利福尼亚州的硅谷和韩国的首尔。 欧洲集群的排名似乎较低,巴黎排名第 10,法兰克福-曼海姆排名第 12。英国表现不佳,伦敦排名第 21,剑桥排名第 55,牛津排名第 88。”[23]https://www.ft.com/content/dbb3bc26-413b-11e7-9d56-2...e998b2

实际上,正如任何一直关注的人所注意到的那样,现代技术本质上是加利福尼亚和东亚的事务,前者专注于软件,后者则专注于硬件。 美国公司在互联网基础设施和平台领域占主导地位,而东亚在消费电子硬件领域占主导地位,尽管如前所述,中国确实在百度、阿里巴巴和腾讯等公司拥有自己版本的通用技术巨头。 相比之下,除了一些成功的应用程序(例如Spotify或Skype)以及实体(例如诺基亚或爱立信)之外,今天的欧洲拥有相对较少的知名科技公司。[24]https://www.unz.com/akarlin/europe-cant-into-big-tech/ 过去,它曾经拥有更多成熟的技术公司,但是来自美国和东亚的竞争猛烈冲击了欧洲的技术行业。

1991 年《华盛顿邮报》在西方抨击日本的高峰期发表的一篇旧文章指出,““日本恐惧症”现象之所以流行,主要是因为似乎没有简单的答案可以防止欧洲失业率已经远高于美国,即将增加,或者如果采取保护主义措施来挽救就业,价格将不得不飙升,从而伤害欧洲消费者。 欧洲的计算机行业正处于崩溃的边缘,因为它们无法与能够更快地适应快速变化的技术的日本和美国公司竞争。 今年,荷兰的电子巨头飞利浦,意大利的奥利维蒂(Olivetti)和法国的布尔(Bull)被迫裁员数千人。 即使有更多政府提供数十亿美元的救助,他们的生存前景也很黯淡。”[25]https://www.washingtonpost.com/archive/politics/1991...japan- bashing/0b9b1f7b-e578-4aae-926b-e2bdd6809f68/

的确,当人们将东亚与全球每个地区的最知名品牌一起考虑时,东亚与高科技的联系就显而易见。 在东亚最著名的品牌中,很大一部分是科技公司。 相比之下,欧洲知名品牌通常倾向于时尚奢侈品或汽车公司。[26]http://www.independent.co.uk/news/business/news/app...1.html 快速,说出你想到的最著名的东亚公司。 我的猜测是你可能会抛出三星、LG、东芝、松下、索尼、联想、比亚迪、大疆或华为等名字。 现在,列出您能想到的最著名的欧洲品牌。 在这里,我猜你可能首先想到的是 Gucci、Burberry、Versace、Louis Vuitton、Hermes、Armani、Chanel 或 Prada 等品牌。

BrandZ的数据显示,中国顶级品牌通常与技术相关
[47]http://www.wpp.com/wpp/marketing/brandz/china-50-2018/ BrandZ数据显示,中国顶级品牌通常倾向于与技术相关
来源:BrandZ数据显示,法国不辜负其作为世界奢华时尚之都的声誉
[48]http://www.millwardbrown.com/brandz/top-french-bran...s/2018,BrandZ数据显示,法国不辜负其作为世界奢华时尚之都的声誉

尽管许多人会指出中国或美国等享有庞大、完整市场的制度因素来解释欧洲科技的衰落,但我实际上想提供一个更面向 HBD 的解释,不仅是为了解释为什么欧洲在技术和工程方面似乎相对滞后到美国和东亚,还有为什么美国的技术偏向软件,而东亚的技术偏向硬件。 我相信上述各种现象都可以用一种共同的潜在机制来解释,即数学/语言分裂。 简而言之,如果你真的很擅长数学,你就会被硬件所吸引。 如果您的技能更倾向于口头表达,那么您会倾向于软件。 一般来说,你在工程和技术领域工作的机会因空间和数量上的熟练而大大增加。

因此,HBD 最终解释了东亚认知资本的重要百分比被分配到何处。 除了偏向数学和物理科学,许多东亚人最终从事实用技术和工程工作。 这意味着仅考虑科学,尤其是所有科学领域,就会忽略许多东亚人倾向于 STEM 首字母缩略词中间两个字母的事实。 在指出现代消费电子产品本质上是一个东亚产业之后,我还想强调一个明显的事实,即东亚裔美国人在硅谷的科技行业中所占比例过高,并且在那里也做出了许多重要贡献。 一个很好的例子是现代计算机图形,它基本上由 Nvidia 和 AMD 主导,它们在 2006 年收购了 ATI Technologies,并将其并入自己的图形部门。 Jen Hsun-Huang 是 Nvidia 的联合创始人之一,今天仍然是其首席执行官和主要发言人。 ATI Technologies 后来成为 AMD 的 Radeon 图形部门,由四名加拿大华人 Lee Ka Lau、Francis Lau、Benny Lau 和 Kwok Yuen Ho 于 1985 年在加拿大安大略省创立。[28]https://en.wikipedia.org/wiki/ATI_Technologies#History 在诸如Nvidia之类的知名科技公司中,东亚裔美国人无疑在技术人员中所占的比例也过高。[31]http://www.nvidia.com/object/fy15-workforce-perform...e.html

事实上,尽管进步人士一再声称硅谷是如此、如此白人,但在湾区的许多精英科技公司中,白人与黑人和西班牙裔人一样,实际上代表人数不足。 谷歌、Facebook 或优步等许多顶级公司发布的多元化数字的总体情况表明,白人在总体上的代表性往往略有不足,在技术职位上的代表性不足,而在高管层中,相对于白人的总百分比而言,其代表性过高。总人口。[27]https://www.washingtonpost.com/news/volokh-conspirac...white/[45]https://www.usatoday.com/story/tech/news/2017/03/28/...26318/ 过去几年在技术行业工作,这似乎也激发了我个人的印象。 通常,对技术的要求在技术上和数量上越严格,亚裔美国人代表人数过多的程度就越大。 大多数技术公司的最白人部分往往在市场营销,产品,销售,设计或行政级别等领域都不足为奇。

来自今日美国。 优步技术职位中按种族划分的员工
来自今日美国。 优步技术职位中按种族划分的员工

接下来,让我们花点时间欣赏一下东亚人和东亚美国人在现代技术和工程中所扮演的角色。 只需考虑一下我们从三星,索尼或LG等东亚公司购买的所有现代装备,例如智能手机,平板电视,平板电脑和SSD,它们已成为我们21世纪的重要组成部分st 世纪的生活方式。 这是好东西,所以请保持这种状态,让我们带上那些可弯曲的OLED屏幕![29]https://www.youtube.com/watch?v=Cxnv_QKxXf0 (在有人提出通常的论点之前,让我指出,OLED的父亲通常被认为是华裔美国人清唐,他在1980年代在伊士曼柯达工作时曾做过这件事。鉴于他的种种,这不足为奇。诺贝尔化学奖也即将获得表彰。[30]https://en.wikipedia.org/wiki/Ching_W._Tang#Biography)

被认为是OLED之父的Ching Tang
被认为是OLED之父的Ching Tang

结论

我希望我已经说服了您,关于HBD的正确思考方式基本上是按照数学/言语分裂的思路,而不是按照整体智商或g的思路,而不是这些概念没有与之相关的领域。用。 一旦考虑到数学/语言拆分,我相信某些事情可以说变得不那么神秘了。 当然,这并不是说HBD的支持者从未讨论过数学/语言拆分。 例如,博客作者史蒂夫·赛勒(Steve Sailer)曾经指出,主要种族差异倾向于反映性别差异,尤其是日本人具有认知上的男性技能,例如在数学上表现出色以及物体的3D旋转,这有助于他们在技术和制造能力上的优势。世界舞台。[32]https://isteve.blogspot.com/2008/04/rev-wright-on-bl...e.html。 尽管我很少看到这种理解和所有伴随的经验预测都能得出其逻辑结论。 我希望本文是朝这个方向迈出的一步。

如果我的主张是正确的,我预计在未来几十年中,我们将越来越多地看到不同的人群专门研究他们最擅长的领域。 这意味着东亚人和东亚社会的特点是偏向于定量的STEM领域,例如物理,化学和工程学,偏向于硬件和高科技制造,而西方社会的特点是偏向于生物科学和科学。医学,社会科学,人文科学以及软件和服务。[41]https://www.theguardian.com/business/economics-blog/...d-land 同样,印度似乎也是一个优势在于软件和服务而不是硬件和制造业的国家。 我的基本论点是,所有这些最终都是潜在 HBD 的反映,尤其是数学/语言分裂。 我相信这是其他人提供的分析中缺乏的关键洞察力。

因此,与其他种族相比,东亚人可能显然聪明得多,而他们在数量上的倾向明显更大。 可以这么说,擅长数学只是一种智力。 像丹尼尔·丹内特(Daniel Dennett)这样的哲学家可能不是陶伦斯·陶(Terence Tao),但根据有关陶林相对于其杰出数学才能的言语能力较弱的传闻证据,可以肯定地说,陶·丹内特都不是。 相反,他们是两个男人,他们的认知特征各不相同,优缺点也各不相同。 最终,每个人​​都追求最适合其先天才能的职业。 一个成为哲学家,另一个成为数学家。 就像他们说的那样,对每个人都有自己的看法。

考虑到所有这些,为什么美国社会始终将东亚人描​​述为非常聪明,而不是采用更细微的看法? 这可能与数学才能所具有的巨大声望有关。 即使一个人在其他方面并不擅长,只要你擅长数学,人们通常仍然将你与才华联系起来。 与社会科学或人文学科相比,我们经常听到关于数学和科学是真实学科的笑话。 甚至在自然科学中,人们也常常认为,相对于生物学而言,物理学是一门更为卓越和高级的科学。 在杰罗姆·卡根(Jerome Kagan)的《三种文化》一书中更明确地阐明了这种隐性的知识等级,他将物理学描述为太阳,而数学则将其描述为核心,随着行星越来越远离和绕太阳公转,其他各种次要学科也有所描述。[33]卡根,J.(2009 年)。 三种文化:21世纪的自然科学、社会科学和人文科学st 世纪。 事实上,除了生命科学的明显效用之外,许多其他主要依赖语言能力的领域似乎往往提供有问题的价值,而数学似乎几乎普遍有用。 特别是,现代心理学、社会科学和人文学科似乎经常受到意识形态偏见的困扰,并且明显缺乏可复制性。[34]https://www.theatlantic.com/science/archive/2016/03/...72272/ 如今,社会科学可能被更好地描述为社会正义,因为所谓的科学家通常只是加强其意识形态先驱,并宣扬政治上正确的,胡言乱语的胡说八道。 意识形态伪装成科学,唉。

相比之下,数学是美丽的、优雅的,似乎是宇宙的语言。 一个聪明的外星物种几乎肯定不会读写英语或地球上任何现存的语言。 但它几乎肯定会拥有我们智人拥有的许多相同的基本数学概念。 很难看出它会如何。 正如物理学家 Steve Hsu 认为的那样,“高语言能力对于显得聪明、或赢得争论并给他人留下深刻印象很有用,但真正高的数学能力对于发现世界上的事物——即发现真理或推理——有用。严格。”[35]http://infoproc.blogspot.com/2011/06/high-v-low-m.html 的确,除了仅通过口头概念就能表达的知识以外,对数学的深入理解就释放了一个知识领域。 关于物理学家尤金·维格纳(Eugene Wigner)关于数学的不合理有效性的评论,物理学家史蒂芬·温伯格(Steven Weinberg)在他的《最终理论的梦想》(Dreams of Final Theory)一书中谈到了同样同样不合理的哲学无效性,表明他认识的任何物理学家都没有在二战后时代中从中受益他们从哲学上以任何方式开展工作。[36]http://emilkirkegaard.dk/en/wp-content/uploads/Stev...9D.pdf

那时真是太了不起了,数学如何不仅可以帮助我们解锁对自然的深刻理解,还可以使我们成为自然的大师。 Blogosphere的Blogger Lion,又名The Artist,以前称为Half Sigma,因此这样说。 数学能力高度有助于价值创造,而语言能力高度有利于价值转移。[37]http://halfsigma.typepad.com/half_sigma/2011/07/ste...s.html 精通数学的书呆子是真正的价值创造者,而他们更外向、在社会上占主导地位、口齿伶俐的同行则将潜在价值转移给了他们作为企业高管的自己。 工程师擅长为他人创造价值以供消费。 另一方面,律师和商人似乎大多精通提取他人为自己创造的财富。

也许就不足为奇了,尽管当今日本是一个高科技工程大国,出口着世界其他国家想要购买的有形物品,但法律领域似乎在日本相当荒唐,律师的字面意思是没事做。[38]https://www.unz.com/isteve/wsj-japans-lawyers-need-m...tcies/ 也许同样不出所料的是,十年前美国经济几乎被银行家和蛇油推销员摧毁,总的来说,这个国家似乎由一群律师和电视明星管理,而相比之下,中国领导人似乎不成比例地拥有工程学位反而。[42]https://www.quora.com/Why-do-Chinese-political-leade...egrees

在美国,尽管数学具有很高的声望,但似乎同时存在潜在的数学恐惧症。 美国人与数学之间这种奇怪的爱恨交加关系,意味着通常那些最缺乏数学敏锐度的人以某种方式使自己相信,聪明只是你对别人大声喊叫或表达自己的观点时所表达的态度的函数。 (如果您不相信我说的是真的,请看今天舆论电视台上所有讲话的话题,从他们的欺负讲坛上无休止地赞美。)我们经常听到这是陈词滥调,表达了美国人被教导如何接受批评的陈词滥调。思维。 相比之下,受HBD的激励,我长期以来一直拥护一种被我称为“认知精英主义”的哲学。 也许最好将这种意识形态称为定量至上主义。 (一方面与认知精英主义形成鲜明对比,另一方面与著名的HBD评论员威士忌威士忌的哲学世界观形成鲜明对比,后者热衷于指出白人女性讨厌,讨厌,讨厌beta男性,而更偏爱高个,深色和英俊的男性。颜色可能恰好被称为公鸡自然精英主义)。

智力对于现代 STEM 社会的运作很重要,而且相当重要。 平等主义的悖论意味着,随着环境在不同人口中越来越趋于平等,按照定义,生活结果中剩余差异的更大百分比必须归因于先天智力的差异。 正如数学/语言分裂或数学和言语能力在实用性方面的不对称性这样的现象所暗示的那样,也许我们应该欣赏并崇敬那些最擅长定量推理的个人和群体,而不是仅仅从事廉价的谈话和虚张声势的吹牛,悄悄地在幕后发动,不知疲倦地为现代科技引擎提供动力。 所以,和我的兄弟姐妹们一起大声喊出来。 从每个街角和山顶大声喊出它,就像我们的好朋友约翰·德比郡 (John Derbyshire) 在不知疲倦地警告我们某些太阳能族群的危险时所表现出的一样无情的活力和坚韧。 数学很好。 数学很有用。 数学是崇高的。 阿门。 现在我们都是定量至上主义者!

这就是故事。 人类生物多样性只是意味着在不同条件下进化的不同人群可能具有不同的身体和认知属性分布。 了解这些细微差别可能是了解 21 世纪未来的最佳方式。st 世纪。 令我惊讶的是,比我聪明得多的人们仍然仅凭整体智商来讨论HBD。 理查德·林恩(Richard Lynn)或查尔斯·默里(Charles Murray)等学者已经很好地证明了东亚人之间数学/语言分裂的存在,但在有关智力的公开讨论中,人们总是倾向于回避通常的谈话要点。 您卑微的通讯员希望,通过更强有力地阐明已建立的心理学计量科学,可以消除概念上的混乱,使男人和女人都从教条的沉睡中醒来,从而使人们可能谦卑地称之为哥白尼式革命,从而在我们的公众理解中蓬勃发展情报和HBD。

参考资料

[1] http://content.time.com/time/subscriber/article/0,33009,965326-2,00.html

[2] 默里,C 和赫恩斯坦,RJ(1996 年)。 钟形曲线:美国生活中的智力和阶级结构。 300-301。

[3] 弗林(J.Flynn)(2007)。 什么是智力? 超越弗林效应。 116。

[4] https://www.sciencedirect.com/science/article/pii/0191886987901358

[5] https://www.maa.org/sites/default/files/USAMO%20Qualifiers%202016_0.pdf

[6] https://en.wikipedia.org/wiki/List_of_Scripps_National_Spelling_Bee_champions

[7] http://spellingbee.com/meet-the-spellers/2017

[8] https://www.unz.com/isteve/scores-by-race-on-lsat-gre-mcat-gmat-and-dat/

[9] https://en.wikipedia.org/wiki/Richard_Feynman#Education

[10] http://infoproc.blogspot.com/2017/09/feynman-schwinger-and-psychometrics.html

[11] http://www.davidsongifted.org/Search-Database/entry/A10116

[12] http://www.nytimes.com/2007/03/13/science/13prof.html

[13] https://www.scientificamerican.com/article/recognizing-spatial-intel/

[14] https://www1.udel.edu/educ/gottfredson/reprints/1998generalintelligencefactor.pdf

[15] https://en.wikipedia.org/wiki/G_factor_(psychometrics)#Spearman’s_law_of_diminishing_returns

[16] https://www.cnbc.com/2018/02/26/china-spent-an-estimated-279-billion-on-rd-last-year.html

[17] http://www.researchcghe.org/perch/resources/publications/wp9.pdf

[18] https://www.nature.com/articles/515S73a

[19] 教科文组织(2010)。 联合国教科文组织《 2010年科学报告:世界科学的现状》。 12

[20] http://www.leidenranking.com/ranking/2017/list

[21] https://www.ft.com/content/254853b2-8f23-11e7-9084-d0c17942ba93

[22] https://www.usnews.com/education/best-global-universities/computer-science

[23] https://www.ft.com/content/dbb3bc26-413b-11e7-9d56-25f963e998b2

[24] https://www.unz.com/akarlin/europe-cant-into-big-tech/

[25] https://www.washingtonpost.com/archive/politics/1991/06/16/europes-new-rage-japan- bashing/0b9b1f7b-e578-4aae-926b-e2bdd6809f68/

[26] http://www.independent.co.uk/news/business/news/apple-most-valuable-brand-iphone-7-google-coca-cola-a7345501.html

[27] https://www.washingtonpost.com/news/volokh-conspiracy/wp/2014/05/29/how-the-asians-became-white/

[28] https://en.wikipedia.org/wiki/ATI_Technologies#History

[29] https://www.youtube.com/watch?v=Cxnv_QKxXf0

[30] https://en.wikipedia.org/wiki/Ching_W._Tang#Biography

[31] http://www.nvidia.com/object/fy15-workforce-performance.html

[32] https://isteve.blogspot.com/2008/04/rev-wright-on-black-white-cognitive.html

[33] 卡根,J.(2009 年)。 三种文化:21世纪的自然科学、社会科学和人文科学st 世纪。

[34] https://www.theatlantic.com/science/archive/2016/03/psychologys-replication-crisis-cant-be-wished-away/472272/

[35] http://infoproc.blogspot.com/2011/06/high-v-low-m.html

[36] http://emilkirkegaard.dk/en/wp-content/uploads/Steven-Weinberg-%E2%80%9CAgainst-Philosophy%E2%80%9D.pdf

[37] http://halfsigma.typepad.com/half_sigma/2011/07/stem-majors.html

[38] https://www.unz.com/isteve/wsj-japans-lawyers-need-more-crime-and-bankruptcies/

[39] http://infoproc.blogspot.com/2012/02/test-preparation-and-sat-scores.html

[40] https://www.technologyreview.com/s/610106/chinese-satellite-uses-quantum-cryptography-for-secure-video-conference-between-continents/

[41] https://www.theguardian.com/business/economics-blog/2015/may/18/as-the-uk-has-discovered-there-is-no-postindustrial-promised-land

[42] https://www.quora.com/Why-do-Chinese-political-leaders-have-engineering-degrees-whereas-their-American-counterparts-have-law-degrees

[43] http://www.universityworldnews.com/article.php?story=20180228175510534

[44] http://web.cos.gmu.edu/~vkunkel/feynman1.html

[45] https://www.usatoday.com/story/tech/news/2017/03/28/uber-diversity-reports-shows-familiar-tech-deficiencies/99726318/

[46] https://www.natureindex.com/country-outputs/china

[47] http://www.wpp.com/wpp/marketing/brandz/china-50-2018/

[48] http://www.millwardbrown.com/brandz/top-french-brands/2018

 
• 类别: 科学 •标签: 东亚人, IQ, 种族和智商 
隐藏423条评论发表评论
忽略评论者...跟随Endorsed Only
修剪评论?
    []
  1. 正好7500字? 你可以编辑它。

    有呀。

    • 同意: Realist
  2. Yan Shen 说:

    感谢罗恩出版了我的作品。 经过多年与这些圈子里的不同人的接触,很高兴终于成为一名发表的贡献者!

    一些评论。 这篇文章是我在工作中花了一天半的时间拼凑出来的,算是第一个版本。 我从这次迭代中删除了一些内容,并添加了一些我正在处理的附加材料,我估计这些材料将为这篇已发表的文章增加 50% 的额外内容。

    我第一次没有谈到的一些话题是:1)东亚人相对于欧洲人的认知特征以及男性相对于女性的认知特征之间的相似性,2)自闭症患者的认知特征似乎与自闭症患者相似。东亚人在数学圈中的比例也相应较高,3)一般来说,如 SMPY/SVPY 所示,数学/语言倾向与教育程度和职业地位都有明显的相关性,4)充实了我文章的各个部分,包括为我所说的“数量至上主义”做出更清晰的论证,不仅解释为什么数学比语言更有用,而且解释为什么硬件胜过软件以及为什么制造业胜过服务。 所以也许稍后无论是在这里还是其他地方,本文的 2.0 版本也会发布。

    不过现在,尽情享受吧!

  3. JJ 说:

    数学和语言的分离似乎确实能够解释很多事情。 但我不明白为什么软件更受语言技能而不是数学技能的影响? 此外,庞大的人口数量使中国医生和研究人员在获取临床数据方面具有巨大优势,以及政府的大力支持、较少的法律限制和更宽松的宗教态度,我认为中国在生物医学和临床研究方面具有更大的潜力与美国或欧盟相比。 例如,中国目前在基因编辑领域据称处于世界领先地位,尽管中国进入该领域的时间相对较晚。

    • 回复: @Zumbuddi
  4. 非常有趣且内容丰富的文章,我希望将来能在 UR 上看到更多严申的著作发表。

    • 同意: Talha, mark green
    • 回复: @Realist
  5. ……几乎所有先进的现代消费电子硬件都是在东亚制造的。

    ……因为注重细节的制造业劳动力在那里很便宜。

    也许这就是“美国科技偏向软件,而东亚科技偏向硬件”的原因之一。

    经济和商业帮助决定了东亚培养了哪些类型的人才。 美国资本涌入的地方,智慧之花就会生长。

    也许有人可以,呃,写一篇长论文,讨论贸易如何使发展向特定方向倾斜。

    • 回复: @PandaAtWar
    , @ThreeCranes
  6. Anonymous • 免责声明 说:

    我不知道 Unz Review 有目录功能。 我猜罗恩是为了他自己的一篇巨篇文章而开发它的。 他可能很高兴能在这里使用它。

  7. Zumbuddi 说:

    下层阶级/智力较低的亚洲人无法进入美国。

    高智商的第五代(欧洲又名白人,非犹太人)美国千禧一代受到了社会工程的影响,旨在不利于他们,而偏爱白人犹太人和移民。 主导教育系统的犹太人设计了天才计划,这些学生在其中度过了整个 K-5 公立学校教育。
    除非他们的父母足够精明,让他们离开公立学校进入蒙台梭利学校或家庭学校,否则这些孩子就毁了。 他们是 SJW。 21 年 2017 月 XNUMX 日,你在华盛顿看到他们戴着 Pussy 帽子。
    这就是社会工程精英对美国最优秀、最聪明的白人年轻人所做的事情。

  8. Anonymous • 免责声明 说:

    只有一个问题。 会有测验吗?

    因为在前几千字之后我就有点走神了……

    然而:如果这样将严先生隔离在这个网站上他自己的区域将有助于让他远离评论线程,我完全赞成。

    • 哈哈: bomag
  9. Zumbuddi 说:
    @JJ

    东亚人在华盛顿地区基因组学实验室中的比例过高。

  10. niteranger 说:

    一篇优秀的文章,经过深入研究。 我同意很多结论,但是,我的第一个问题是你关于 SAT 考试不被泄露的说法。 高中时,我的朋友想去西点军校。 他的 SAT 成绩为 815 分。 这是很多年前的事了,不是今天的 SAT。 他在一所军事预科学校学习了一年。 他将 SAT 成绩提高到了 1250。他考入了西点军校,现在是一名上校。

    同班第一名的女生,SAT考试考不到800分。 她只是无法参加考试,而且会变得非常紧张,以至于会流泪。 除了 SAT 考试恐惧症之外,她所有科目都取得了 A 的成绩。 她成为了一名医生。

    我的一位钓鱼伙伴是一位退休的指导顾问。 他招收顶尖学生,教他们如何参加 SAT 考试并提高分数。 这对他有用。 一名比我早两年的高中男学生在 SAT 考试中只漏掉了一个问题。 他现在在精神病院里。

    我面临的下一个问题是以发表的论文数量作为优秀标准。 只要有钱,什么都可以发表。 计算机软件正在发布假文章:https://www.theguardian.com/technology/shortcuts/2014/feb/26/how-computer-generated-fake-papers-flooding-academia。 今天发表的大多数论文都是笑话,大多数撰写这些论文的人并不理解他们使用的统计数据。 塔勒布多次指出这一点。

    您还应该检查撤回的论文数量。 我相信亚洲国家在科学方面的数量是最高的。

    在我看来,你的想法是,因为你在一个领域很出色,并不意味着你在另一学科也会很出色,甚至有能力。 艾萨克·牛顿爵士是一位才华横溢的物理学家,但对生物学的理解却像孩子一样,而且还是一位秘密炼金术士。 有一位著名的化学家,几年前获得了诺贝尔奖,他不懂基础生态学,发表了一些愚蠢的言论,让人好奇他是如何获得诺贝尔奖的(不记得他的名字了)。 我认识一位女士,她以编辑和修改工程书籍为生。 她认为工程师是世界上最糟糕的作家。

    最后,也许最严重的罪犯是人文科学和社会科学领域的人(这些不是科学;仅仅因为你量化某些东西并不会让它变得更好,特别是如果它一开始就是垃圾)。 你可以在电视、网络和书籍上看到这些人试图对他们一无所知的东西进行写作、解释和武断。 真正可悲的是 CNN、MSNBC、ABC 或任何其他网络的谈话者没有意识到这些人不是真正的科学家,他们所说的几乎所有内容都是完全错误的。

    总的来说,这是一篇非常好的文章,我感谢您写这篇文章。

    (我今晚在网站上编辑时遇到问题;它一直跳来跳去;可能是我的浏览器的问题;昨晚我在页面顶部收到了一些奇怪的代码)

    • 回复: @Hippopotamusdrome
    , @Yan Shen
  11. Anonymous [又名“DJV梅尔维尔”] 说:
    @Yan Shen

    一篇精彩的文章!

    一些评论-
    言语上聪明的人已经认为那些理解和应用数学的人比自己优越。

    他们依靠那些试图跨越两个领域的人(比如你)将数学翻译成隐喻语言,这样他们就可以非常部分地理解它的含义,这比不理解要好得多。

    他们也许傲慢地认为一切事物,包括数学揭示的自然世界,都是他们语言磨坊的谷物。

    小要点:您可能是匆忙地“拼凑”而不是“抄袭”您的文章。
    “认知”很好,并且可能包含相当多的事实,类似于相信艾滋病的原因是常见的金龟子或花园金龟子。

    问候,
    戴安娜梅尔维尔

    • 回复: @Yan Shen
    , @WhiteWolf
  12. …除了 Spotify 或 Skype 等一些成功的应用程序以及诺基亚或爱立信等实体

    难道没有人注意到成功的欧洲公司的地理概况吗? 斯堪的纳维亚人。 这可以用智商偏差来解释吗?

  13. Dr. Doom 说:

    Steve Sailer 是一位数学奇才,但他的西方观点与环太平洋地区相去甚远,无法深入了解它的细节。 我恰好在亚洲度过了一段青春,然后沉浸在美国文化中。 我看到了两面,这对我来说并不神秘。 在远东,这不是虎妈,而是年轻男性成功的严肃压力锅文化。 他们希望你成为最棒的,因为家庭是第一位的。 在西方,这是一种悠闲和懒散的态度。 他们实际上给我服用了利他林,因为我指出低薪女学生在黑板上犯的错误。
    美国男人被告知要放松。 《回到未来》中的那个斯特里克兰实际上是个恶棍,因为他想让马蒂·麦克弗莱准时上学。 这个笑话一定会飞到远东观众的头上。 远东的孩子去测试教练。 他们不断钻孔。 美国男人正在偷懒,到处睡觉,被吸毒,但在这些测试中仍然表现良好。

    不要低估白人的遗传潜力。 他们生活在不利的条件下,并因不懈怠而被下药。 远东的孩子压力很大,找家教甚至作弊,但并没有比白人做得好得多,白人被灌输反学术宣传并被诱导不负责任。

  14. PandaAtWar 说:
    @Buzz Mohawk

    ……因为注重细节的制造业劳动力在那里很便宜。

    废话。

    例如,通过分析三星、华为和苹果等领先电子品牌价值链上的详细增值交钥匙高科技零件,您很快就会意识到,现在这基本上是东亚的事情,并且在可预见的时间内如果你看看最前沿的技术研发和突破在哪里,你就会发现未来。 即使是一些“美国”的贡献也主要包含源自东亚的技术。 在整个价值链中,欧洲唯一值得一提的贡献可能来自荷兰科技巨头ASML,这是由几家关键的德国交钥匙技术供应商实现的,这些供应商越来越多地面临来自一些财力雄厚的东亚初创公司的现状竞争尤其是来自中国的。

  15. Patrick82 说:

    我读到过,中国人在明朝的某个时候就远离了数学,而到了 19 世纪,大多数有学问的中国人并不精通这门学科。 许多在 1860 年代(鸦片战争之后)“洋务运动”期间质疑改革者的官员对“西方科学”持怀疑态度,此时,“西方科学”当然已经在所有领域超越了中国,也许除了医学。 比较正统的文人学者在讨论“夷人奇术”时,经常使用“诡计”、“魔法”等词语。

    在我看来,当时中国最优秀、最聪明的人(如果确实可以用这些词来形容士大夫的话)都致力于需要更多语言能力的事业……八足文、诗歌、历史和哲学。

    我不怀疑作者的结论,只是想把它扔在这里。

    • 回复: @Chrisnonymous
  16. 一篇非常好的文章(尽管它可以更短)并且符合我一直在思考的内容。

    只是,我认为作者混淆了他所写的各种类型的智力和领域。 或者也许我们需要想象力或其他术语&智力还不够?

    第一个反对意见:理论物理和数学或多或少不是“相同的”(我什至不会讨论实验物理领域)。 历史告诉我们,过去 300 年来最重要的物理学家在数学方面的表现远不如他们,反之亦然。 例如,爱因斯坦是历史上最伟大的三四位物理学家之一,但他并不具备可比的数学能力。 然而,是他提出了广义相对论,而不是比他更有数学天赋的大卫·希尔伯特(David Hilbert),后者可能是 3 世纪最伟大的数学家。 确实,希尔伯特在与爱因斯坦讨论后几乎立即得出了方程(https://en.wikipedia.org/wiki/Relativity_priority_dispute#General_relativity_3 )但足够现实/公平,承认爱因斯坦的首要地位。

    费曼在文章中的一个例子中。 我记得他说过这样的话:“物理之于数学,就像性之于自慰。” 显然,这是大多数物理学家的立场——我是一名理论物理学家——我们经常居高临下地对待我们的数学朋友。

    2. 数学的不同领域又如何呢? 难道不同类型的智能有可能在数论领域产生伟大的成就,但在拓扑学领域则不然吗? 或者选择其他数学领域。

    3. 在我看来,工程学和计算机科学是完全不同的领域。

    无论如何,这是一篇罕见的、我希望内容丰富的文章,它将取代对僵化的智商辩论的相当沉闷的崇拜。

    • 回复: @lavoisier
    , @Dmitry
    , @voicum
    , @utu
  17. PandaAtWar 说:

    严神,好作品!

    熊猫的2美分:

    1,熊猫也一直在争论这种空间/语言的分裂有一段时间了(笑),特别是怀疑为什么他们目前在总智商分数中的估值为 1:1,因为它在某种程度上暗示了这一点在执行这两组截然不同的心理任务时需要相同数量的智力和能量(或神经元,它们的连接,速度,一般效率......等,哈哈)。 当然,有些人试图通过相关性/多元回归等统计工具来解决问题,但鉴于相关性并不完美,因此还有很大的空间来深入挖掘和解释。

    2,正如你所说,有些人在空间部分和语言部分都表现出色,而另一些人则在空间部分和语言部分都表现出色。 然而,对于后者,熊猫怀疑这可能并不完全正确。

    因为人的大脑能量毕竟是有限的。 如果我们假设人们天生倾向于将更多的时间和精力投入到他们天生有天赋的地方,那么可以合乎逻辑地推断,一些拥有正常语言分数的人实际上通过将更多有限的时间和精力投入到空间上而牺牲了语言部分——从一开始就执行相关任务。 因此,事实上,根据测试的测量,其中一些语言本身不一定是规则的。 对他们来说这只是一个选择,是由于大脑能量分配而有意或无意的选择。

    由于 1 个空间 IQ 点和 1 个语言 IQ 点之间很可能不是 1:1 的智力和能量输入关系(空间任务消耗更多),正如 Panda 直觉推测的那样,相比之下,情况就小得多了那些在语言方面表现出色,同时在空间得分方面表现出色的人,由于选择的原因,实际上在空间方面也表现出色。

  18. anonymous • 免责声明 说:

    Yan,我想查看过去十年中进行的智商测试的数据,按宗教细分:印度教、新教、天主教、犹太教、佛教等(特别是针对 18-30 岁的年轻人)

    这些数据是否存在,如何找到?

    • 回复: @TelfoedJohn
  19. @Buzz Mohawk

    我的回应也是如此。 这里面有一些先有鸡还是先有蛋的问题。 制造业转移到亚洲后,那里的潜在人才得到了表现自己的激励和机会,这有什么奇怪的吗? 与此同时,在美好的美国,我们的就业市场青睐那些具有语言能力的人,但我们却把许多视觉空间和数学人才留在了人行道上

  20. Bruce 说:

    我在最大的国防公司工作。 东亚人在场,但大多数工程师,包括顶级工程师都是白人男性。 可能需要安全许可,但这对亚裔美国人来说不应该是问题。

    • 回复: @Stan d Mute
  21. 好东西!

    我正准备联系经济合作与发展组织 (OECD) 的安德烈亚斯·施莱彻 (Andreas Schleicher),因为 2015 年 PISA 分数显示,中国人的阅读能力几乎和美国人一样糟糕,而他们的数学和科学成绩远远领先。 你帮我(和安德烈亚斯)省了力气。 非常感谢。

    顺便说一句,对于费曼粉丝来说,没有什么比他在 YouTube 上的视频《Los Alamos From Below》更好的了。 它冗长而精彩,通过一个聪明孩子的眼睛介绍了 20 世纪物理学的所有巨人。

    • 回复: @anonymous
    , @TT
  22. Yan Shen 说:
    @Anonymous

    小要点:您可能是匆忙地“拼凑”而不是“抄袭”您的文章。

    哈哈,是啊。 这显然是我的大脑冻结了……太糟糕了,你无法在 5 分钟后编辑你的评论。

  23. 这是 严申?

    “我讨厌所有白人,因为他们抢走了所有最好看的亚洲女孩”严申?

    无论哪种方式; 乏味、迂腐的文章。 学习编辑。 为了简洁起见,有很多话要说。

    • 回复: @Anonymous
  24. 对此有一些注意事项:

    1.并非技术的所有部分都同样复杂。 在航空航天等领域,东亚已经落后了。 同样,军事和太空设备(如航空电子设备,传感器,复合材料,光学,电子产品)的设计问题,设计分析模型以及材料工程,计量学要比其他所有问题综合起来要先进得多。 美国,俄罗斯,以色列等在这些地区处于领先地位。 它比消费电子产品复杂得多,并且需要更大的单个障碍。 大型技术创新,例如全新的功能或进攻/防御层次(人体弹药,雷达,声学),都是并且都是在西方或俄罗斯制造的。 尽管中国将来可能会在这里取得领先,但其优势领域仍然有所不同。

    2. 并非所有科学领域都同样复杂或困难。 也就是说,东亚(几乎)没有对纯数学或理论物理学做出任何研究贡献,而西方有很多这样的研究贡献。 (参见:菲尔兹奖获得者。)事实上,几千年来东亚人甚至无法在数学上“发明轮子”。 他们没有提出基本的符号、基本的证明或任何东西。 他们所做的一切都是从印度偷来的。 近代东亚从未有过重大科学发现。 这也是一个神话,即更高级的纯数学仅基于数学智能(在简单的层面上是“形式的”而不是拓扑或抽象的); 事实上,言语方法、空间方法也常用于发现。 (G.波利亚用“语言”思考,庞加莱用“诗意”思考,大多数人不是用象征性的方式思考,而是使用“软”可视化;请参阅哈达玛关于“发明心理学”的书,研究数学家的可视化。)

    3.东亚人在文学、诗歌、哲学等方面并不逊色。这只是一个刻板印象,因为他们有自己的语标语言。 中国有着使用言语推理的悠久哲学传统,但它在于构建复杂和抽象的品质(以及禅宗佛教中一种品质如何转变为相反的品质),而不是复杂的对象。 中国仍然有与西方相媲美的“哲学家”,参见熊十力最近翻译成英文的《意识唯一性新论》。 中国没有哲学或复杂的概念化是一个神话:但它有。 此外,中国的理论在技术上也卓有成效(阴阳思想也应用于武术、战略、医学)。 此外,中国的白话四部小说也是战略论着(如《三国演义》)和阴阳脉络的哲学论着。 另请参阅弗朗索瓦·于连 (François Jullien) 和罗杰·T·艾姆斯 (Roger T. Ames) 关于中国哲学、语言学和符号学的著作,其中包括王弼等人物。 中国在自然语言之上发明了更广泛的“第二语言”(而西方只是尝试过,例如威尔金斯、达尔加诺、莱布尼茨的语言)。 西方发明了最严格的单一语言(形式数学)。

    4.中国的优势不是在科学上,而是在前科学实验上,而且中国在开发用于启发式的整个语言(基于阴阳级数)方面非常深入。 可以说,中药仍然是许多发现的源泉,比如“Gilenya”药物(FDA 批准)。 中国最大的单一进步——例如无化学的火药、磁罗盘、地震预报机、医学进步——都是基于高度复杂的语言模型,比如阴阳系统。 一些西方数学家(如格洛腾迪克)发现阴阳思维与发展启发式高度相关,因为这将发明问题转化为“在哪里”的问题,预测在哪个阶段应该在哪里注入相同的质量或方面。 前科学实验仍然与科学一样相关:热机在热力学之前制造,飞机在空气动力学之前制造,“机翼之谜”,计算机在计算机科学之前制造,等等。

  25. “地球号飞船”需要船员进行更新。
    书呆子的反冲进一步搞砸了。
    阿尔伯特·爱因斯坦很聪明,
    没有浮士德式的伪装。
    “认识你自己”? 火星机器人做不到。

  26. wayfarer 说:

    二十多年来,作为一名独立的 K-12 数学和科学导师赚取兼职收入。

    大部分工作都是基于互联网,利用高速数据连接、固定电话、join.me 以及由最好的作者编写的 PDF 教科书。

    一个自力更生的孩子可以通过“themathpage”掌握算术并建立良好的代数基础。

    来源: http://www.themathpage.com/

    然后,他可以借助“粉笔谈话”掌握更高级的主题。

    来源: https://www.youtube.com/channel/UC9SPN6qaM0DB455-DrWAdpA

    成为一名熟练的数学家甚至熟练的音乐家的一个简单秘诀是好奇心、渴望、获得高质量的教学材料以及坚持不懈的耐心练习。

  27. UnzReader 说:
    @Thought Criminal

    精彩的观察……呼吸新鲜空气真是令人心旷神怡!

    谈论使用毫无根据和愚蠢的二分法愚蠢地欺骗古老而光荣的人民和文明。

  28. 精通数学的书呆子是真正的价值创造者,而他们更外向、在社会上占主导地位、口齿伶俐的同行则将潜在价值转移给了他们作为企业高管的自己。 工程师擅长为他人创造价值以供消费。 另一方面,律师和商人似乎大多精通提取他人为自己创造的财富。

    非常棒的作品……直到上面。 将其归因于较高的数学/空间能力和缺乏语言推理能力。

    在有人看到商业应用之前,出色的公式或技术没有任何(经济)价值。 例如 – Xerox PARC 有 GUI,但不知道该怎么用它,直到一个高语言智商的黎巴嫩美国白人说:“我可以用这个狗屎改变世界!” 或者中国发明的火药,他们想不出比制造漂亮的彩色灯光更好的用途了。 我们都知道欧洲人用这个想法做了什么。 与之相反的是,犹太人对马克思主义的争论无休无止,而东亚人忘记了争论,并开始消灭他们自己的 50,000,000 人。 或者是犹太人无休无止地争论的“多样性”观念,非犹太欧洲人疯狂地采用了这种观念,并似乎用它来种族灭绝自己。

    拥有一个绝妙的想法,无论是数学上的还是口头上的,除非经过彻底的辩论然后推向市场(即“技术转让”),否则没有任何价值。 这才是 HBD 能够教给我们的真正教训,不是吗?

    • 回复: @TT
  29. Dwright 说:

    仍然无法解释为什么亚洲人的驾驶技术如此糟糕。 过度分析?

  30. @Bruce

    可能需要安全许可,但这对亚裔美国人来说不应该是问题。

    这不是自相矛盾吗? 你写“亚裔美国人”的事实只是强调了人类部落主义是非常真实的,并且对现实世界产生影响,其中包括安全许可。

  31. @Patrick82

    在我看来,当时中国最优秀、最聪明的人(如果确实可以用这些词来形容士大夫的话)都致力于需要更多语言能力的事业……八足文、诗歌、历史和哲学。

    是的,我也想知道这个问题。 所以我在考虑是否可以用按字母书写的语言和使用字符的语言之间的差异来解释。 (这是西方古典学(特别是希腊研究)学科内的整个研究领域。)我的想法是这样的:书面中文更注重概念理解,而不是口头细微差别和详细解释。 我认为除了书面文字之外,汉语语法也有这种倾向,尽管我真的在猜测,因为我不会说汉语。

  32. JSM 说:
    @Dr. Doom

    然而,正如那些最密切关注中国科技崛起的人所清楚的那样,中国对定量领域表现出明显的偏好,特别是物理、化学、工程、数学和计算机科学

    给我解释一下……中国有5000年的文明。 为什么是欧洲的白人发明和发现了严现在告诉我们的所有物理、化学、工程和数学,中国在这些领域的崛起是多么惊人?

    严会说:“哦! 白人,他只是摘了所有低垂的果实。 现在是亚洲人在做所有艰苦的工作。”

    那么,为什么白人要采摘低果子呢? 中国已有五千年文明史。 为什么他们不摘那么简单的水果呢? 啊? 过去五千年你去哪儿了,优秀的亚洲人? 啊?

    • 回复: @jim jones
    , @myself
  33. @Dr. Doom

    我认为你的评论和其他一些评论是一个森林/树木问题,我也在原始文章中看到了它。 看来,白人(以及南亚人,即次大陆印度人)处于极端边缘的比例高于接近平均水平的东亚人(如撒哈拉以南非洲人和女性)。 当我们看待例外论时,这非常相关,不是吗? 从总数来看,东亚人口将产生更多非常优秀的数学/科学从业者。 在最极端的水平上,按人均计算,他们的天才数量更少,但在这里,总人口数量脱颖而出,因此他们也可能产生更多的天才,除了菲尔兹奖获得者或改变历史的物理学家的水平。

    然后是从众心理问题,这种心理在白人和东亚人中似乎最为强烈。 在很大程度上没有创始人(犹太人)无休止的争论的情况下,我们采用了反例外主义的思想,压制了例外主义,而东亚人则不太赞同,尽管(或者可能是因为)盲目追随这种做法已经消灭了他们自己的 50,000,000 人。如果没有持续不断的争论,这种想法应该受到谴责,而这些争论在很大程度上阻止了犹太人自己大规模收养。

    • 回复: @myself
    , @denk
  34. PandaAtWar 说:
    @Thought Criminal

    那里提出了许多声音点。

    然而,您举的一些例子并不完全正确:

    当前消费类电子产品(如一些顶级智能手机)中最先进的半导体技术的复杂程度不会比任何国防技术有所下降。 相反,大多数高端国防技术都植根于这些半导体技术。

    航空航天是一台吞噬金钱的机器。 美国/俄罗斯在航空航天领域遥遥领先的主要原因(即使不是最决定性的原因)之一是50年冷战积累的巨额军事预算、研发人员以及他们被迫竞争生命的实践经验——要么死的优势,中国和日本都没有。 熊猫在某处读到,从1950年代到1980年代,中国用于航空发动机研究和生产的整个国家预算,包括技术、材料和人员成本,只有区区3万美元,还不到一个小美国的年工资支出国防实验室……如果把早期工业化基础的成果加起来,即使在100世纪1980年代,美国/英国/日本/德国在很多领域都比中国至少领先XNUMX年……

    • 回复: @JJ
  35. 我现在除了浏览之外没有时间做更多的事情,但有两点:

    (1)《Re Feynman》,这本关于他试图前往中亚国家图瓦的旅行的书描述了费曼试图将图瓦翻译成英语,但他的尝试相当薄弱,这似乎支持了他语言能力差的观点。 此外,他认为音乐从根本上来说就是节奏,这似乎表明他对大多数人认为是核心的音乐方面“没有耳朵”——那些可能被描述为音乐中最非 M 的方面。

    (2) 另一方面,费曼因其解释复杂物理的能力而闻名。 这与 M/V 划分无关吗? 这指出了沉的文章(尽管我还没有读透)可能存在的问题——未能充分定义术语。 认知能力真的可以分为 M 和 V 部分吗?如果是这样,这在日常生活中是如何发挥作用的? 难道中国只是拥有大量的高G人群,但当今生活的语言障碍和经济需求限制了其言语表达吗?

    最终,我很高兴看到沉严写的是认知而不是移民,因为他关于金钱是衡量一个人对美国(某种抽象的说法)价值的衡量标准的建议仍然令人厌恶。

  36. George 说:

    空间、3D、推理还重要吗? 在最高的智力水平上,我听过但不明白的理论是,维数比 3 多得多。在工程等较低水平上,CAD 可以用作许多不具备自然空间推理能力的人的拐杖。

  37. 1. 西方、俄罗斯、日本、中国都有着截然不同的价值体系:东亚之所以不“发明”,是因为它寻求的不是彻底的发明,而是其他形式的扩张。 俄罗斯也有自己的价值体系,它在文学、高端工程、数学等方面达到了顶峰。

    西方世界代表着爆炸:要么是一开始的爆炸(古希腊),要么是无尽的爆炸:在顶部(法国、意大利),要么是从内到外的爆炸(德国)。 快速、精湛的发明(如音乐)就是爆炸; 这是突然的夺取,从内到外,对一个伟大时刻的主宰。 渐进的建设、纪律和远见的水平(从罗马人开始,一直到意大利文艺复兴时期精确测量和视觉表现的发展),指导发明并防止其过度。 德国文化以理查德·瓦格纳为终结,因为他代表了内心彻底爆发的彻底满足,没有什么可以表达的:身体、心灵和精神的完整旅程到此结束。 (过去唯一另一个爆炸性的文明是古印度。)

    俄罗斯(我认为还有匈牙利)代表着汹涌的漩涡,漩涡,心理上的而不是哲学上的,精神上不清楚但实质上是干净的,是模仿和反映外在世界的内心世界。 是不断的发展,从内心的迷茫中恢复过来。 (因此,俄罗斯哲学的弱点是俄罗斯小说的长处。)这不是从内到外的“爆炸”。 它不会爆炸,它会“聚集”并调动各个方面。 俄罗斯的扩张是​​最高和最深的统治,它“弥补”了一些关键地区的内部混乱或封锁,弥补了周围北极地区的肮脏。 (因此,数学和理论物理是苏联式的,没有一丝一毫浪费的思想。)障碍是内部的,而不是外部的。

    东亚代表的不是发明(爆炸),也不是俄罗斯无休无止的“酝酿”,而是有机广度扩张的力量(虽然发明只是扩张的一种,但却是最快的)。 东亚有许多独特的深层建筑形式(绘画、文学、策略中的“辩证”结构),而西方有两种独特的深层建筑形式(纯数学、古典音乐创作)。 与奥斯瓦尔德·斯宾格勒所说的不同,中国或日本从来没有任何清晰的、不混乱的“内在意识”,因为真正的基本原则是概念形成的问题:一切——每一个概念、规则、目标、区别和内在意识。 -定向性——是对立面之间的妥协或“调和点”(因此儒家、道家和佛教的矛盾性,其中之一甚至在早期就向后看,其中之一在内在与外在之间“调和”,其中之一他们都很极端——而且每个人都处理不同领域的问题)。 另一种看待这一点的方式是,中国从未达到完全的文明和满足,也没有任何对向后看的满足的“硬化”,甚至没有任何形式的完整文明:它将这一点与它的对立面结合或永远混合在一起:这在每个时期的基本制度混乱程度(暴力在表面之下“酝酿”),以及后来发生的史诗文学和战略思维的巨大扩展(例如三个王国)。 日本也是如此:它实现了巨大的扩张,例如史诗般的武士文学(《平家物语》)。 它代表了一个有机体的广泛扩张,在每个时期都保持强大的力量,而不是在一个时期内完全革命性的。

    相反,中国和日本的文明观并不是一种完美的状态,而是一种长期有效的安排(必须是适度的,而不是完美的)。 西方基础科学的革命依赖于历史的某个阶段,就像古印度的进步(从头开始构建了整个符号和主题)。 不保证它与此之外的相关性。 东亚的进步(在前科学发现和纯工程方面)与每一个阶段都相关,但并不绝对主导任何一个阶段。 但在历史的某个阶段之后(例如,在基础物理学基本得到澄清之后,只有凝聚态物质、复杂系统等方面取得了进展),有充分的证据表明,俄罗斯和东亚模式将比西方模式占主导地位。

    2. 两国(中国和日本)从来都不是对手。 日本在系统层面帮助中国; 中国有能力的领导层是在日本接受训练的(如蒋介石、周恩来),没有日本,中国就不会尝试现代化。 日本征服满洲对于阻止俄罗斯帝国向中国进军是必要的。 日本征服中国不会从文化中减少任何东西,而只会增加额外的元素(与试图废除汉字的毛主义者相比)。 日本的“大屠杀”因其将鲁迅等中国精英文化人物从民族主义白色恐怖中拯救出来这一事实而得到平衡。 1970世纪90年代至XNUMX年代从日本到中国的技术转让是人类历史上最大规模的生产技术转让,涵盖了美国试图避免获得的极端高端生产技术(详情见埃蒙·芬格尔顿(Eamonn Fingleton),在他的书中)。 关键是,没有日本,中国就不可能有今天:反之亦然(日本满洲的工业化为其战后经济体系提供了典范)。 E. 亚洲国家之间的关系是模糊的——并非绝对消极或积极的关系。 西方往往会误解这一点,因为它只是停留在观察某种实践而不考虑它的不同方面。 例如,抱怨日本的中国人也购买日本的产品和娱乐,以游客的身份访问日本,而中国学生仍然将日本作为首选目的地。

    这是基于基本原则的。 中国和日本的文化(包括其哲学和形成观念的基本方式)都是基于中庸的原则:每一个信仰、规则、法律、目标、制度都不是绝对遵循的,而是有例外,或者有矛盾的方面。 日本和其他东亚国家也是如此:如果你了解他们的哲学和文化价值观,“中庸”是先于文明概念本身的东西。 也就是说,中国在历史上从来没有完全文明过(而是文明、野蛮、颓废的混合体)。 西方盛极而衰; 中国从未有过任何可比的黄金时代(只有白银时代)。 中国不支持完全自由或极权主义,而是支持某种中间妥协(摆脱两个极端的某些限制,例如今天很容易规避互联网限制)。 在中国几乎没有绝对的友谊或开放的自我(因此不喜欢狗)。 中国从来没有完全的教育,但也从来没有完全“黑暗”(有教育的“精神”)。 中国在历史上从来没有绝对安全过,但它避免了两个极端。 中国从来都不是绝对的种族主义者; 它把野蛮人称为劣等人,但它在整个历史上也鼓励与野蛮人的种族混合,而“中国人”的概念在种族上是模糊的(不像“罗马人”那样普遍,但严格来说也不是一个民族群体)。 公共观点和私人观点之间存在着急剧的缓和,从而滋生了矛盾。 中国从来没有绝对的公民精神,但其不统一的时期比西方(一个帝国永久分裂成较小的交战国家)持续时间短。 游击战是中国的特长,它实际上介于个人犯罪和有组织的公民战争之间。 也就是说,中国的犯罪行为并不比西方少,但它不像西方那样有任何生硬、随意的暴力,或者纯粹无端的暴力或酷刑。 这些都不能保证绝对优越或劣等; 这实际上取决于人们所谈论的历史阶段。

    (正是通过使用适度的属性,比如阴和阳不是绝对排他的,这使得品质变得更加复杂和结构化,如果推动得足够多,一件事就会转化为它的对立面;例如,通过尝试在每个领域保持温和,你将极端推向一些关键的问题——比如文学作品的巨大规模和总体寿命。此外,不存在从特殊到一般的“归纳”,因为每个特定的事实实际上都是完全“普遍的”;每个特定的信仰或看似简单的区别,是由矛盾的方面构成的。)

    必须承认,这具有某些长期优势:中国从来都不是绝对文明的(而是部分野蛮、部分颓废),这一事实使其不会像欧洲的出生率那样下降到极端颓废的地步。 它们集权但高度腐败的事实具有长期优势,因为中国可以采用许多在更严格的社会中不可能实现的“危险”技术(例如长期太空计划、人类基因工程)。 中国从来没有像罗马帝国、俄罗斯或德国在鼎盛时期那样军事强大,但也从来没有像它们在衰弱时期那样衰弱:这又使得(军事力量的长寿;也就是说,中国的游击战争能力总是好的,他们能够“逐渐”赶走入侵者)。

    但随后“长寿”本身就被缓和了:中国比印度、波斯或欧洲、在机构形式和特定知识“传统”方面稍微更连续一些(就主要和极具影响力的文学作品的持续流动而言)依赖于深层的语言建构,但不像日本那样连续。 它在许多方面(例如建筑的保护)不像欧洲那样连续。 不仅长寿被缓和了,而且对待长寿的态度也被缓和了:中国自诩古老,却破坏了过去实际的直接文物,并且还采用了外国的思想、技术等等。

    这也是中国和日本哲学的基本原则。 三岛由纪夫就是日本“禅宗”人物的一个很好的例子(他在日本国内仍然受到如此的尊重)。 通过清除自己的中国影响,他成为中国本身的特征人物(每个王朝末期的“革命思想​​家”,包括20世纪的鲁迅),而不是日本。 通过支持他自己想象中的日本,他使自己在面对真实的、现实的日本时显得无能为力。 作为一个西方式的个人主义者和尼采式的自我主义者(走向积极的肯定),他最终自杀了(虚无主义),这是自我的极端“空虚”。 通过拒绝佛教,他成为一个非常“禅宗”的人物。 通过将日本人塑造成内在个人主义者的错误形象,他麻痹了西方对日本的理解,从而摧毁了西方和个人主义。

    • 回复: @AaronB
    , @JJ
    , @myself
  38. jim jones 说:
    @JSM

    我们应该列出所有中国未能发明的东西:

    内燃机
    晶体管
    抗生素
    微积分
    喷气发动机
    聚合物
    核裂变与聚变
    氮肥
    疫苗
    电视(Television)
    广播电台

    • 回复: @Bardon Kaldian
    , @bjondo
    , @Anon
  39. 我自己对此的看法是,东亚的“视觉空间”智能对纯文学(缺乏这种成分)做出了更深入的贡献,而西方的言语智能(针对字母语言,而不是语标语言)刺激了数学的发展(例如,数学证明的思想源自言语辩证法,如“矛盾论证”)。 也就是说,更高层次的数学实际上需要数学以外的能力; 更高层次的文学受益于汉字提供的额外视觉内容(促进诗歌)。

  40. Joe Hide 说:

    致雅深,,
    很棒的材料,但是太长了。 您可以尝试在 3 篇文章中而不是 1 篇文章中提出如此出色的想法。
    尽管如此,智商写作的标准钟形曲线上的平均值至少有 3.5 个标准差……哈!
    请继续为我们语言认知高级且倾斜的读者写更多!

    • 回复: @acementhead
  41. AaronB 说:

    我打算客观地分析这一点——有很多内置的假设和历史错误! – 但后来我意识到神话的目的不是代表客观事实,而是提供灵感、鼓励,最重要的是 – 自我验证。

    更重要的是,像所有神话一样,这个神话表达了关于当前种族和国家联盟的更深层次的、多层次的真理,而“所有”历史时刻都试图证明自己是永恒的和不可避免的。

    亚洲人非常努力地试图证明自己并赶上西方的优势力量,他们放弃了传统的文学文化,将所有的精力投入到工程和技术上——这是西方不可阻挡的力量的源泉。 一个重大的牺牲。

    而西方确实已经丧失了权力意志,在“权力”纪律方面也有所懈怠。

    这个新的历史时刻需要一个新的神话来证实自己,就像19世纪需要“白人负担”的神话,犹太人的崛起需要“精英统治”的神话一样——两者都掩盖了事实,尽管它们揭示了白人的真实面目。他们要描述的历史情况。

    我说——让亚洲人拥有它。 他们应得的。 他们非常努力地“证明”自己,并取得了成果。

    亚洲人最需要的是与自己和世界和解,而今天许多亚洲人所感受到的痛苦和怨恨只有在他们确信自己不逊色于西方时才会消散——他们并不逊色!

    但他们需要在这一点上满足自己。 为了这个目的,他们牺牲了很多生活的乐趣。

    更重要的是,我似乎越来越清楚,白人不准备做出“证明”自己所需的牺牲,而是正在进入一个不同的阶段。

    这将是亚洲世纪——让我们一起庆祝它,并对亚洲天才将以一种新的、有趣的方式创造出什么东西感到好奇。 我自己也很好奇。

    我们距离“追赶”阶段的结束还有一两年的时间,很快就会进入亚洲世纪阶段。

    在不可避免的历史逆转中,西方现在可以进入一个轻松的阶段,在一个新的组织下培养“生活的艺术”,就像亚洲人曾经做过的那样,该组织将在当前混乱结束时出现,而当前的混乱是经济全球化的自然结果。权力意志阶段的结束,其最初的混乱是令人困惑的。

  42. 写一篇文章假设任何人都知道 HBD 是什么,而不是高智商的标志。
    谁能启发我?

    • 回复: @Daniel Chieh
    , @Alden
  43. Anonymous • 免责声明 说:
    @Johnny Smoggins

    事实恰恰相反吗? Vox Day 也承认这一点。

    http://voxday.blogspot.com/2011/05/mailvox-why-do-white-women-choose-black.html

    3. 低地位的白人女性往往被黑人男性高估。 只能吸引白色3的女人往往也能吸引黑色7。 我认识几对黑人男性和白人女性夫妇,其中黑人男性比白人女性更具吸引力。 这种现象也常见于白人男性与亚裔女性的情侣中。 当女性可以选择通过扩大可接受的男性范围来提高自己的性别排名时,许多人选择这样做就不足为奇了。

    任何生活在亚洲人口众多的地方的人都会认识到一个令人讨厌的事实,即在这些关系中的亚洲女性通常从丑陋到非常丑陋。 异族通婚很少具有审美吸引力。

    • 回复: @Johnny Smoggins
  44. AaronB 说:
    @Thought Criminal

    非常聪明和有趣的评论,我认为这基本上是正确的。

    西方不做温和的事情——极端的崛起,随之而来的是绝望和冷漠,因为希望未能实现,人类的状况未能得到改变。 这是一个不稳定的系统。

    中世纪更像是西方的温和时期,但到了14世纪逐渐陷入极端悲观主义,然后被极端的世俗乐观主义所取代。 19 世纪末,由于所有的征服和技术辉煌都未能带来“救赎”,绝望和冷漠与日俱增,这个世界的生活仍然一如既往地充满荆棘和不如意。

    看看亚洲是否真的从内部向西方方向转变——现在正处于“浮士德式”阶段——还是只是表面移植,一旦条件更加有利,就会像假皮一样脱落,这将是一件很有趣的事情。

    到目前为止,亚洲追赶西方的本质并不是浮士德式的——没有辉煌的爆发——而是中等水平的稳步积累的表现,再加上西方后浮士德式的冷漠,已经将重心转移到亚洲。

    但现在下结论还为时过早。

    如果日本的发展轨迹是亚洲试图追赶和超越西方的模板,那么我们也可以将中国排除在经历短暂的表现繁荣之后回归到调节不同元素的更传统的文化模式之外。

    看看浮士德式的西方是否最终毁灭了自己,或者最终学会了节制,这将是一件很有趣的事情。

    • 回复: @Daniel Chieh
    , @myself
  45. WhiteWolf 说:
    @Anonymous

    当我在学校时,我天生擅长数学。 我不明白为什么人们会遇到问题。 对我来说,这是一个简单得可笑的话题。 然而有一天,我敬畏地看到一名学生凭记忆画出了他看过的一部科幻电影中的场景。 当我对他的才华大加赞扬时,他只是耸了耸肩。 对他来说,这太简单了,他对此毫不在意。 我认为我们都倾向于低估自己的才能,而钦佩那些在我们所面临的困难中展现才华的人。

  46. @AaronB

    西方不做温和的事情——极端的崛起,随之而来的是绝望和冷漠,因为希望未能实现,人类的状况未能得到改变。 这是一个不稳定的系统。

    不确定不稳定,但是是的。 值得注意的是,西方似乎确实一直在黄金时代和黑暗时代之间摇摆。 就温和而言,我高度怀疑浮士德式的西方也将被“摧毁”——不安显然是有目的的,并且像所有周期一样,它可能会在几代人之后再次在黄金时代找到它的表现。

    • 回复: @AaronB
  47. @Thought Criminal

    2. 并非所有科学领域都同样复杂或困难。 也就是说,东亚(几乎)没有对纯数学或理论物理学做出任何研究贡献,而西方有很多这样的研究贡献。 (参见:菲尔兹奖获得者。)事实上,几千年来东亚人甚至无法在数学上“发明轮子”。 他们没有提出基本的符号、基本的证明或任何东西。 他们所做的一切都是从印度偷来的。 近代东亚从未有过重大科学发现。 这也是一个神话,即更高级的纯数学仅基于数学智能(在简单的层面上是“形式的”而不是拓扑或抽象的); 事实上,言语方法、空间方法也常用于发现。 (G.波利亚用“语言”思考,庞加莱用“诗意”思考,大多数人不是用象征性的方式思考,而是使用“软”可视化;请参阅哈达玛关于“发明心理学”的书,研究数学家的可视化。)

    虽然我同意基本上存在不同的“数学智力”,但我认为现在谈论东亚在数学方面取得的重大成就还为时过早; 至于菲尔兹奖,我们得看看未来20-30年会发生什么。

    顺便说一句,您对庞加莱、阿达玛(我们可以添加笛卡尔以来的大多数伟人)的深思熟虑表明“数学智能”是一个粗糙且可能过于矛盾的概念。

    也许“智能”这个词太狭隘了。 创造力、想象力、“视觉思维”、普罗提诺式的非话语“思维”又如何呢?http://gen.lib.rus.ec/book/index.php?md5=5BBF8C5AE74B97F18D4379819A512F84)?

    3.东亚人在文学、诗歌、哲学等方面并不逊色。这只是一个刻板印象,因为他们有自己的语标语言。 中国有着使用言语推理的悠久哲学传统,但它在于构建复杂和抽象的品质(以及禅宗佛教中一种品质如何转变为相反的品质),而不是复杂的对象。 中国仍然有与西方相媲美的“哲学家”,参见熊十力最近翻译成英文的《意识唯一性新论》。 中国没有哲学或复杂的概念化是一个神话:但它有。 此外,中国的理论在技术上也卓有成效(阴阳思想也应用于武术、战略、医学)。 此外,中国的白话四部小说也是战略论着(如《三国演义》)和阴阳脉络的哲学论着。

    在此我强烈不同意。 中国哲学,由于中国语言的性质和思想结构,根本无法与西方传统相比较,从阿那克西曼德、毕达哥拉斯、恩培多克勒、赫拉克利特、柏拉图、亚里士多德、斯多葛学派、普罗提诺、奥古斯丁、培根1、埃留根纳、笛卡尔、培根2、斯宾诺莎、莱布尼茨、休谟、卢梭、洛克、康德、黑格尔、谢林、叔本华、维科、尼采、马克思、柏格森、海德格尔、斯宾格勒、詹姆斯、维特根斯坦、卡西尔、弗洛伊德(作为心灵形而上学家)、波普尔..

    西方哲学在存在和不存在的所有领域都具有丰富的探究性,而这在中国(和印度)思想(伦理学,认识论,本体论,美学,政治哲学,社会哲学,早期心理学,宗教哲学,宗教哲学)中是根本不存在的。历史,非宗教形而上学,教育,科学哲学等。

    文学也是一样——诗歌是不可翻译的,而亚洲没有莎士比亚、陀思妥耶夫斯基或普鲁斯特的普遍性等同物。

    4.中国的优势不是在科学上,而是在前科学实验上,而且中国在开发用于启发式的整个语言(基于阴阳级数)方面非常深入。 可以说,中药仍然是许多发现的源泉,比如“Gilenya”药物(FDA 批准)。 中国最大的单一进步——例如无化学的火药、磁罗盘、地震预报机、医学进步——都是基于高度复杂的语言模型,比如阴阳系统。 一些西方数学家(如格洛腾迪克)发现阴阳思维与发展启发式高度相关,因为这将发明问题转化为“在哪里”的问题,预测在哪个阶段应该在哪里注入相同的质量或方面。

    这在一定程度上是正确的,但我还没有看到在将针灸等应用到现代医学方面取得任何进展。 事实上,在某些情况下,它被视为一种重要的安慰剂: https://sciencebasedmedicine.org/acupuncture-doesnt-work/ , http://skepdic.com/acupuncture.html

    • 回复: @AaronB
    , @Yan Shen
    , @TT
    , @Dmitry
  48. JJ 说:
    @PandaAtWar

    我同意航空航天分析。 除了航空航天之外,要产生有意义的成果,大多数研发本质上都是烧钱加上过去反复试验中积累的经验/数据的游戏(这也需要真金白银)。美国经济严重依赖美元霸权的铸币税,但作为国家债务增长,以往依赖政府巨额资金和补助进行研发的模式已经难以为继。 例如,为了验证某种复合材料的一些基本数据,美国实验室要进行数千次实验,而中国则需要数十次才能达到同样的结果。

    许多人将高科技想象为隐藏在实验室中的东西或外星技术。 他们只是忽略了这样一个事实:这些实验室技术只有在可供人们使用时才有意义。 如果智能手机可以做得更好而且更便宜,我会说这是巨大的高科技。

  49. 弱智的中国小克隆人说世界将是他们的——Hello Bot,我们会杀死这个克隆项目。 zhidaou bu——中国白痴——世界讨厌你们克隆人。

  50. Kartheek 说:

    1.有人告诉我,为什么实行内婚制又名选择至少2500年的上层种姓印度教徒(实际上是印度的所有种姓)并不比犹太人聪明

    • 回复: @Daniel Chieh
    , @Billy
  51. @jilles dykstra

    当然。

    过去两周,伦敦出现了种族隔离现象,令人惊讶的是,这应该不会引起任何担忧,甚至可能有助于让我们所有人了解人类生物多样性的奇迹。 让我解释。

    在百米世界纪录保持者博尔特的带领下,牙买加男子横扫伦敦奥运会短跑项目。 对于这个加勒比小国来说,这是一个惊人的壮举。 但作为更广泛趋势的一部分,这并不奇怪。 西非血统的跑步者是地球上跑得最快的人……

    …这就是群体遗传学 101。博尔特和他的牙买加队友是世界人口中的一小部分成员,他们是精英运动员,他们的祖先可以追溯到非洲西部和中部,他们的体型和生理机能经过数千年的进化而形成独特的形状跑得快。

    基因相关的、高度遗传的特征,例如骨骼结构、肌纤维类型的分布(例如,短跑运动员具有更多天然的快肌纤维,而长跑运动员天然具有更多的慢肌纤维)、反射能力、代谢效率和肺活量在人群中分布不均匀。 我们是否知道有助于在该领域取得成功的特定基因? 不,但这并不是反对遗传学在体育运动中的强大作用。

    https://www.forbes.com/sites/jonentine/2012/08/12/the-dna-olympics-jamaicans-win-sprinting-genetic-lottery-and-why-we-should-all-care/2/#49d0fa283fbe

    大脑是 显着地, 极大地 比肌肉纤维更复杂,但没有理由相信它完全不受选择压力(甚至纯粹的偶然因素)的影响。 更活跃的杏仁核会引发某些行为,更活跃的前额叶皮层会引发其他行为,即使通过血脑屏障的外来化学物质(例如酒精)会影响认知和行为,按理说,内部产生的化学物质也会影响认知和行为。我们在大脑的性别二态性中看到了睾酮对大脑的影响,即减少胼胝体连接性的“雕刻”效应和大概是某种效应导致海马体变大; 注射睾酮对女性的直接影响还增加了竞争感、性兴趣和空间智力。 与软件类似,化学物质会产生影响,但化学物质也会影响大脑的硬件。

    当然,这也让我发现智商有时是一个相当可疑的衡量标准,因为大脑如何工作以及认知的构成非常复杂。 但我可以接受它作为一个随意的、包罗万象的术语,但功能实用性有限。

  52. Truth 说:

    特里被指派写一篇关于家里发生的事情的故事,他从一个房间到另一个房间,列出了详细的内容清单。”

    听起来就像一桶猴子。

    如今,社会科学可能被更好地描述为社会正义,因为所谓的科学家通常只是加强其意识形态先驱,并宣扬政治上正确的,胡言乱语的胡说八道。 意识形态伪装成科学,唉。

    几乎没有物理那么多。 NASA是假的,“宇宙”是以地心为中心的。

    精通数学的书呆子是真正的价值创造者,而他们更外向、在社会上占主导地位、口齿伶俐的同行则将潜在价值转移给了他们作为企业高管的自己。 工程师擅长为他人创造价值以供消费。 另一方面,律师和商人似乎大多精通提取他人为自己创造的财富。

    这是毫无意义的。 财富不能从一系列数学计算中提取,甚至不能从车库发明中提取。 当这些产品在市场上找到它们的实际价值时,财富就被创造出来了。 这就是为什么爱迪生在今天比特斯拉更出名、更有影响力。

    不过,我的朋友,这是很好的处女作,比这里已经印刷的大部分内容都要好。 不过,有一点小问题,不要在你的工作上写下“结论”。 这是一篇专栏文章,而不是一篇论文。

    • 回复: @WhiteWolf
  53. Alden 说:
    @jilles dykstra

    这意味着人类生物多样性。

    长期以来,自由主义者声称所有种族和民族都是相同的,尽管在体型、肤色、骨骼结构上存在明显差异。

    现实主义者将物理生物差异称为人类生物多样性,这是抵制自由主义宣传的一种方式,自由主义宣传认为非洲人和其他种族没有进步的唯一原因是殖民主义和邪恶的白人

    自由主义者称 HBD 为种族主义、优生学等

    • 回复: @jilles dykstra
  54. JJ 说:
    @Thought Criminal

    我记得在这里读过一条评论,认为基督教价值观通过鼓励人们寻求新事物,在推动西方社会前进方面发挥了重要作用。 相比之下,中国有句话叫“活在当下”,我想,如果不是为了在鸦片战争后赶上西方、与西方竞争,中国会很乐意这样做。 如果我们想一想,我们真的比生活在没有电视、智能手机等的过去的祖先幸福得多吗? 现代技术基本上是一些奇特的玩具,它们将我们的注意力从某件事转移到另一件事。 到目前为止,我们还没有面临导致文明崩溃的核浩劫,但对独特、发明新事物的不懈追求很可能导致人类的灭亡。 一方面,人工智能取代人类并不遥远。

    • 回复: @Daniel Chieh
  55. Truth 说:

    总体而言,这个国家似乎是由一群律师和电视明星管理的,而相比之下,中国领导人似乎不成比例地拥有工程学位。 [42]

    好吧,如果有人假设净移民流量作为证据,我的问题是,“结果如何?”

    • 回复: @Daniel Chieh
  56. @Kartheek

    选择只会改善到环境所需的程度:现存的有袋动物在澳大利亚经过精心选择,但无法与其他地方引进的胎盘哺乳动物竞争。

    无论如何,这只是一种理论。

  57. sondjata 说:

    这可能是我今年以来最好的读物。

  58. 东亚人是一群高智商的克隆人,他们是 BOT 的人类近似,他们的灵魂中没有形而上学 – 什么都没有 – 他们现在不来自印度的宗教 – 佛教 – 现在不存在 –一堆骗钱的机器人,它们几乎是完全的机器——要小心这些机器人 UNZ commenteriat。 它们是最接近人类机器的——它们的灵魂中没有形而上学。

    • 回复: @TT
  59. 好文章,参考文献很多,哇! (尽管我承认我略读了部分内容)。 拼写错误警报:照片图例中的 Yamanka -> Yamanaka。

  60. @JJ

    一方面,人工智能取代人类并不遥远。

    而且它不需要。 正如卡辛斯基指出的那样,我们正在迅速将我们的思维外包出去。 因此,我们头脑中的想法可能很快就不再那么重要了。

    • 回复: @davidgmillsatty
  61. 请记住,UNZ 东亚人是机器人,他们的灵魂中没有形而上学。 只是近似机器人的无情实用主义者。

  62. Alden 说:
    @Dwright

    在美国出生或长大的亚洲人都很好。 问题在于来自许多国家的移民,而不仅仅是亚洲人。

    • 回复: @Truth
  63. bjondo 说:
    @jim jones

    发明了什么,发现了什么?

    • 回复: @Realist
  64. @Truth

    非常糟糕,但被阴谋论的其他成员所宣传,声称地球实际上并不平坦!

  65. 别误会我的意思,我娶了一位中国妻子,但这种文化缺乏形而上学是值得注意的,就像他们对现任总统宣称自己是终身暴君缺乏抗议一样。

    • 回复: @Daniel Chieh
  66. @Dwright

    作为人肉克隆,我们的思维必须外包给附近的母舰,并且需要几秒钟的时间才能做出反射性决定,因为越来越多的垃圾降低了近地轨道的信号完整性。

  67. AaronB 说:
    @Daniel Chieh

    这是一个很好的观点,正如历史所表明的那样,西方很可能在未来的某个时刻在黄金时代再次崛起。

    然而,总有一种可能性是,极端涨跌的“西方周期”相对于温和涨跌的“亚洲周期”,周期较少——因为每个周期都更加极端——并且可能有一个自然的结局。

    也可能存在一个“元周期”,其中整个人类世界现在正在冲向毁灭,接下来可能是巨大的冰河时代以及气候和地质灾难,只是为了一个新世界在灰烬上崛起——一个新物种甚至,或许,还是一个非人类的世界。

    我们所知甚少,世界没有理由需要对我们这个小物种有利。

    印度和亚洲的宇宙观暗示了如此巨大的世界毁灭循环。

  68. AaronB 说:
    @Bardon Kaldian

    中国哲学,由于中国语言的性质和思想结构,根本经不起与西方传统的比较,

    不幸的是,这是不真实的。 与严神和(可以理解的)新兴神话相反,西方独特的成就——其卓越的文明——是在科学、数学、工程和技术方面。

    正是在文学和哲学方面,西方尚未超越其他文明——尽管它竞争得很好。

    中国人在道家经典中达到了人类思想无法超越的哲学思想的顶峰和顶峰。 这种情况发生得相当早——西方传统花了 2,000 多年的时间才达到同样的微妙程度,并认识到语言的局限性和沉默的价值。

    你所看到的“丰富”的西方传统是向中国和印度更早达到的顶峰迈进的记录。 这样的记录会显得“丰富”——也许它的价值在于作为脚步记录。

    但在其他地方,高潮出现得更早。

  69. @Grahamsno(G64)

    我听说,第三次对巨魔来说是有魅力的。

  70. Anon • 免责声明 说:

    如果你是1970世纪80年代、90年代或XNUMX年代等时期的一名中国学生,想要去美国学习,并且你希望中国政府放你走,那么你必须是一名理工科学生。 政府认为让想要学习文学的学生出国没有任何好处。

    这种效应在其他亚洲国家也出现过,但原因不同。 最好的科学和工程学校都在美国,因此这类学生往往会去美国。 但如果你有语言天赋,国内的大学就很适合你。

    但在过去两代人中,这意味着在美国接受科学培训的亚洲学生最有可能在美国大学毕业后获得联系,从而找到美国科学工作。 他们中的一些人自然选择离开自己的祖国移居美国。 美国亚裔人口中科学家的数量不成比例,他们的孩子往往会搬到亚洲社会飞地,与其他亚裔科学家的孩子混在一起,并在他们中间结婚。 这有利于科学基因。

    此外,美国科技公司还安排向理科学生发放 HB-1 签证。 他们不把这些职位给文学专业的学生,​​因为他们认为没有理由从亚洲国家的人才库中招聘。

    美国亚裔人口并不是亚洲人的代表性样本。

  71. 赢得西方这个世界的不是智商,而是任何测试都无法衡量的大胆冒险精神。 谁想成为哥伦布或麦哲伦,海洋令人恐惧地领略我们所生活的无限——然而它们却通过这些无限的航海者航行——这是其他文化所没有的——这些中国机器人甚至没有发现这片巨大的大陆大小澳大利亚国家——就在驴子下面——然而库克船长才在这头克隆人军队的驴子下面发现了一些东西。

    • 回复: @jilles dykstra
    , @TT
  72. @Daniel Chieh

    LOL
    在斯大林元帅时代,他们只需要一名工人,斯塔汉诺夫同志,而不是1,500人。 显然,自过去的美好时光以来,事情一直在倒退,安德烈·马尔蒂亚诺夫、猎隼等人将证明这一点。

    • 回复: @Daniel Chieh
  73. 非常有趣的观点,而且可能是真的。

    那么创造力呢? 新奇? 数学能力和语言能力有什么区别吗?

    鉴于当今亚洲技术精英的明显优势,如何解释科学创造力和创新历史上几乎完全是西方的创造?

  74. @Alden

    他们不是
    梅尔维尔·赫斯科维兹(Melville J. Herskovits),《黑人过去的神话》,1941年,1958年,波士顿

  75. @Grahamsno(G64)

    只要有求生的意志
    Felipe Fernández-Armesto,《文明》,伦敦,2000

  76. BobX [又名“认为 YS prolix 的鲍勃”] 说:

    YS,给我们做了一顿美味的炖菜。 它的基础是数学,而东亚人在数学上偏向右边。 在这个相当健全的基础上,他添加了一种蔬菜混合物,从而污染了炖菜。 这里有丰富而丰盛的小吃。 一个国家的制造基地很重要,应该受到保护。 太糟糕了,它被如此多的掺杂的油腻所玷污了。

    YS 在沉浸在西方的过程中挣扎着,但似乎也因此讨厌自己。 他是为什么我们需要把移民政策调低,让熔炉运转的典型代表。

  77. @lavoisier

    我有一些理论,但根据我的研究,创造力与智商并没有特别相关。 有一些证据表明,它甚至呈负相关,因为患有多动症的儿童的工作记忆较差(与智商高度相关),但创造力较好。

    实际上我对此有更广泛的理论。 我确实觉得有点不幸,总的来说,实际上并没有太多关于创造力的研究,相反我们得到了很多轻率的说法。

  78. @Verymuchalive

    老实说,一旦你拥有了基础设施,很多壮举都是可能的。 我不喜欢“哦,这座闪闪发光的纪念碑”如此受欢迎的吸引眼球的一个特殊原因是,它通常忽视了拥有合适的人员、流程和其他需要更长时间才能开发的东西的脚手架。

    你可以在 5 分钟内构建一个小部件,但重要的部分是 50 年的知识和设备,以构建组装工厂、管理人员等。 不过,人们似乎非常关注最终产品小部件。

    • 回复: @Verymuchalive
  79. 一些抱怨:一个是偶然的——温伯格对哲学的评论不仅是庸俗的,而且是错误的——例如,看看爱因斯坦对认识论传统的参与及其对他思想的影响。

    其次,更重要的是,我担心仅基于当代世界的比较的论点。 如果一位分析师被问到同样的问题——比较 1800 年东亚和欧洲的定量和非定量智力,他会做出什么决定? 我相信他会得出相反的结论——欧洲日益增长的全球主导地位是基于其定量科学,而东亚的衰落是基于其对非定量/“文明”艺术和技能的关注。 但如果这种差异具有 HBD 成分,那么时代差异就不重要了。

    我怀疑社会学因素与遗传因素同样重要——因为数量最多的产业是与发展“追赶”最相关的产业。 但这只是替代假设的第一次尝试。

    • 回复: @AaronB
  80. @Truth

    尽管这个讽刺的链接可能很可爱,但它很能说明美国领导层几乎完全缺乏任何技术知识,导致了最近一些真正非最佳的结果。 我并不是主张国会应该有更多工程师的人,等等。 但我认为应该有更多技术上有能力的顾问和工作人员。 在各级决策中除了律师之外几乎完全缺乏任何人,这导致了一些非常奇怪的结果。

    它在很大程度上助长了整个“不是一个严肃的国家”的感觉。 这不是“白人法治国家需要律师”的事情; 即使在德国,拥有工程学或科学学位的政治家比例也更高。 我相信俄罗斯也是如此。

    • 回复: @Truth
  81. @lavoisier

    大滴定科学与中国东西部历史、哲学、经济社会

    李约瑟

    由Routledge发布

    中国的历史文明与印度文明、欧洲闪族文明并列为世界三大文明之一,但直到最近才开始对其科学技术成就进行探究。
    从一世纪到十五世纪,中国人总体上远远领先于欧洲,直到文艺复兴时期的科学革命才使欧洲领先。
    在这十五个世纪中,以及从那时起,西方一直受到来自中国和东亚的发现和发明的深刻影响。
    在这一系列文章和讲座中,李约瑟探索了中国早期领先、欧洲后来赶超的奥秘

  82. AaronB 说:

    鉴于当今亚洲技术精英的明显优势,如何解释科学创造力和创新历史上几乎完全是西方的创造?

    一种假设是,这是努力的结果。

    由于努力与智商密不可分,目前亚洲的技术优势以及空间智商倾斜似乎是为了应对西方挑战而将努力从传统文学追求上巨大转移的结果。

    同样,西方技术的优势也是努力转向主宰自然的结果。

    有证据表明,在 19 世纪——奋斗的伟大时代——西方人的智商更高,我想知道西方人的空间智商是否也更高,尽管可能没有那么倾斜。

  83. 所以基本上,如果你的智商为 125+,并且非常不平衡,那么就成就而言,天空就是极限。 然而,如果你既能加起来又能进行联合写作,那么你可能除了做一些世界上的基本工作,并可能抚养另一代无名小卒之外,不会去任何地方。

  84. Truth 说:
    @Daniel Chieh

    我的朋友,我们不需要“最佳结果”。 技术被高估了。

    “你”一天就建了一条地铁。 伟大的! 但工人们仍然买不起房子,也永远不会结婚。

    那么谁在乎呢?

    • 回复: @Daniel Chieh
    , @AaronB
  85. AaronB 说:
    @blank-misgivings

    如果一位分析师被问到同样的问题——比较 1800 年东亚和欧洲的定量和非定量智力,他会做出什么决定? 我相信他会得出相反的结论——欧洲日益增长的全球主导地位是基于其定量科学,而东亚的衰落是基于其对非定量/“文明”艺术和技能的关注。

    优点。

    这就是为什么所有这些基于上世纪最新数据的比较都存在严重问题。

  86. Eagle Eye 说:
    @anony-mouse

    是的,不错的作品,但需要认真的编辑和收紧(这恰好证明了数学与语言分裂的要点)。 特别是要摆脱对“我”的过度使用——这让人想起巴里·奥巴马(Barry Obama)的一次演讲。

    此外,本文还提出了一些后续观点:

    (1) 哪些其他能力子领域(除了语言能力)具有实际重要性并且与一般 g 有所不同?

    例如,面对公认的科学(或政治等)观点时的固执等特征? (想想司马迁、马丁·路德、马克斯·普朗克等)

    (2) 鉴于右尾语言和非语言能力的差异,将这些测量结果汇总为单个 g 值是否仍然有意义?

    PS:请更正标题中日本医生的姓名。 我是山真也_A_naka。

  87. @Verymuchalive

    是的,我倾向于认为这些都是愚蠢的无用功。

  88. @Truth

    我的朋友,我们不需要“最佳结果”。 技术被高估了。

    在平坦的地球上一切皆有可能。

    事实上,甚至公寓也是不必要的,因为它们需要技术来建造。 这是一场毫无意义的讨论,显然是在“互联网”上进行的,这也被高估了,也没有必要。

    • 回复: @Truth
  89. AaronB 说:
    @Truth

    让他们迎来胜利的一天。

    西方国家也在科技崛起的日子里欢呼雀跃。 当形势逆转时,拒绝向亚洲人提供这种服务是不慷慨和狭隘的。 他们为此付出了很大的努力。

    我非常乐观——甚至兴奋——西方开始看到“技术被高估了”,但如果我们真的决定将我们的努力从技术优势上转移开来,那么我们就应该慷慨地让亚洲人享受阳光下的日子。

    • 回复: @Talha
    , @Yan Shen
  90. j2 说:

    我认为这篇文章与一个人的智商概况及其大学的科学成就联系得太紧密了。 当然,有一定的联系,但不是那么强。

    我的意思是:
    如果中国政府对某些领域的科技成果进行投资,他们就可以期待这些领域取得成果。 他们有钱,他们可以做到。 欧洲人目前无法这样做,因为他们没有那么多钱,因为他们的公司在工作更便宜的东方投资。

    政府决定支持的领域与人们的智商状况之间没有根本的一致性。 如果你能给他们薪水和时间,你就可以从每一个超过1万人口的人中找到从事高水平科学工作的人才。 所以,如果你有很多钱并且想要在特定的科学领域达到顶级水平,你就可以做到,就像你可以在体育运动中做到这一点一样。 从事任何一项运动,对其进行投资,如果你为他们提供工作,你就会发现人才,然后这项运动就会上升。

    我们也根本不清楚智商特征(语言与视觉空间)有多少是遗传的,以及有多少取决于这些技能的发展。 由于东亚人比较内向,他们的社交沟通较少,语言能力也较差。 这并不一定意味着他们的遗传语言智商较低,只是由于遗传非智商相关的原因(内向),他们发展的语言技能较少,而空间技能较多。

    • 回复: @Yan Shen
    , @RudyM
    , @Dmitry
  91. Truth 说:
    @Daniel Chieh

    你没有 Twinkie 的智商,但你足够聪明,能够理解这一点。

    • 回复: @Daniel Chieh
    , @Truth
  92. anonymous • 免责声明 说:
    @Godfree Roberts

    我正准备联系经济合作与发展组织 (OECD) 的安德烈亚斯·施莱彻 (Andreas Schleicher),因为 2015 年 PISA 分数显示,中国人的阅读能力几乎和美国人一样糟糕,而他们的数学和科学成绩远远领先。 你帮我(和安德烈亚斯)省了力气。 非常感谢。

    我不知道这是不是因为中文书写系统的困难造成的? 台湾在阅读方面的得分也接近中国,但韩国和日本得分更高,新加坡和香港是得分最高的两个国家。

    • 回复: @TT
  93. J.Ross 说: • 您的网站

    你是对的,试图将所有东亚定量测试的成功归因于作弊是错误的。 如果你认为没有人知道东亚文化中作弊和不诚实的常态,包括许多关于受过高等教育的顶级人士将目的置于手段之上的轶事和谚语,那么你就是在浪费时间。 这种指责不能通过完善对 HBD 概念的理解来消除,因为它是一个文化和语言问题。
    我建议你读一些海明威的作品来获取风格指导。
    除此之外,这是一个非常好的专栏,有合理的论点和明确的证据。

  94. anonymous • 免责声明 说:
    @lavoisier

    那么创造力呢? 新奇? 数学能力和语言能力有什么区别吗?

    有谁知道创造力是否与“开放经验”的人格特质有关? 顺便说一句,我认为欧洲血统的人在这方面的得分高于东亚血统的人。 如果是这样,“对经验的开放态度”是否也与欧洲人对大规模移民、LGBT 和西方其他病态的热爱有关?

  95. FKA Max 说: • 您的网站
    @Dwright

    我怀疑这可能与近视有关。 但我认为它们不一定会导致更多事故。 所以,我猜,不好但相对安全的司机?:
    [...]
    种族在视力问题中发挥作用,研究建议

    在这项研究中,18.5% 的亚洲儿童近视——这是近视白人儿童数量(4.4%)的四倍多; 近视非裔美国儿童人数的三倍(6.6%); 近视西班牙裔儿童人数的一倍半(13.2%。)

    https://www.unz.com/isteve/kat-chow-on-npr-andrew-sullivans-model-minority-myth-ignores-poverty-of-bhutanese-americans/#comment-1842990

    夜间近视与夜间机动车事故的关系

    在视觉投诉调查问卷的结果或白天发生的事故数量方面,这些司机与该组其他司机之间没有统计学上的显着差异。 然而,近视度数 >0.75 D 的驾驶员在夜间发生的事故多于其他组的驾驶员 (p=0.044)。
    结论:
    在本研究人群中,夜间近视 >0.75 D 的驾驶员更有可能发生夜间事故。 这可能意味着应该对选定的驾驶员群体进行夜间近视检查。
    http://onlinelibrary.wiley.com/doi/10.1111/j.1600-0420.2006.00875.x/pdf

    视觉与驾驶

    https://www.ncbi.nlm.nih.gov/pmc/articles/PMC2975746/

    许多研究都表明,视力充其量与驾驶员安全(即碰撞事故)的联系非常微弱,因此对于识别未来有可能发生碰撞事故的驾驶员而言,视力是一个很差的筛选测试。 相比之下,很明显,视力与驾驶表现的某些方面(例如道路标志识别)有关。

    研究发现亚洲毕业生近视率高的原因

    一项新研究发现,亚洲主要城市高达 90% 的毕业生患有近视。 科学家表示,在受影响的人中,10%至20%患有高度近视,可能导致失明。 这项发表在《柳叶刀》医学杂志上的研究将眼睛损伤与亚洲学生长时间学习以及缺乏户外光线联系起来。

    哈西德派的亚历山大·拉帕波特(Alexander Rapaport)在布鲁克林和皇后区经营马斯比亚施粥所,他说,哈西德派认为,户外休闲时光会让他们受到街头的诱惑,尤其是衣着暴露的纽约人。

    http://www.nytimes.com/2012/06/29/nyregion/hasidic-jews-in-heavy-dress-bear-up-in-summer.html

    学习习惯对犹太青少年近视的影响。
    https://www.ncbi.nlm.nih.gov/pubmed/8254449 Zylbermann 等人。 (1993)

    我们发现一组193名正统犹太男学生的近视患病率和度数在统计学上显着高于其他学生的学习习惯。

    https://www.unz.com/jpetras/judeo-centrism-myths-and-mania/#comment-1849454

    那么,为什么我们的司机如此糟糕呢?

    当然,从轶事和科学角度批评女性司机并不是什么新鲜事。 无数研究证明,在导航和空间意识等因素方面,我们的能力不如男性。

    但这些研究都还没有考虑到一个将女性不良驾驶行为完全提升到另一个层次的变量:犹太人的身份。 这篇研究完全是我的——尽管我承认经验证据纯粹是轶事。 但相信我,这就足够了。

    https://www.thejc.com/comment/comment/so-why-are-we-such-crashingly-bad-drivers-1.65494

    以色列:欢迎来到罪犯和不良司机的土地

    美国准确地警告以色列,“攻击性驾驶很常见,许多司机未能保持安全的跟车距离或在变道前打信号灯……”,从而巩固了以色列令人不快的交通事故记录……澳大利亚警告居民,如果你开车,“可能会用石头砸你的车”它在安息日期间进入极端正统的犹太社区。

    http://www.independent.co.uk/news/world/middle-east/israel-welcome-to-the-land-of-criminals-and-bad-drivers-8101165.html

  96. @Daniel Chieh

    在距离我办公室不远的地方,道路工作人员在过去 10 年里一直在不断地拆除和重新修补同一个十字路口。 我没有骗你。 我不知道他们在那里做什么,但无论是什么,将人类送上月球所需的时间更少。 美国和中国的区别 C. 2018年,总结就在这儿

    唯一值得安慰的是,早在 2010 年,公路工作人员就清一色是墨西哥人。 现在他们几乎都是白人。

    • 回复: @Truth
  97. Yan Shen 说:
    @niteranger

    莱顿排名是基于已发表的前 10% 被引用论文的总数,但在某种程度上,这仍然是一种数量衡量标准,因为典型的东亚大学在所有已发表论文中进入前 10% 的论文比例较低。

  98. Yan Shen 说:
    @Bardon Kaldian

    关于中国哲学,黑格尔有一句著名的评论:孔子“只是一个具有一定实践和世俗智慧的人,没有思辨哲学”,这表明如果他的学说也许对他的声誉会更好。从一开始就从未被翻译过。 也许这并不太令人惊讶,因为即使按照西方标准,黑格尔也被认为是一位出了名的难以理解的元物理学家……

    https://books.google.com/books?id=bATIDgAAQBAJ&pg=PA48&lpg=PA48&dq=hegel+confucius+no+speculative+philosophy&source=bl&ots=3hBO6SX8V6&sig=-lmbweZxGQqfkQ0-DsEADs5kBSg&hl=en&sa=X&ved=0ahUKEwjDo92Lh9bZAhUhqFQKHf9rAusQ6AEISjAG#v=onepage&q=hegel%20confucius%20no%20speculative%20philosophy&f=false

    • 回复: @Bardon Kaldian
  99. @Bardon Kaldian

    关于爱因斯坦是否应该在狭义相对论或广义相对论上获得主要荣誉存在很多争论。

  100. Yan Shen 说:
    @j2

    当然,有两条论证路线。 一种认为,历史偶然的制度因素会严重影响一群人选择专业。另一种推理认为,如果没有像文化大革命这样的明显障碍,人们最终不可避免地会被他们有天然亲和力的事物所吸引为了。 这是标准的因果关系箭头问题。 非洲之所以贫穷是因为它被殖民了,还是非洲之所以被殖民是因为它贫穷而且没有齐心协力?

    我认为前一种推理在现代之前可能更有效。 我的观点是,我们越来越多地开始看到人们被他们天生擅长的事情所吸引!

    • 回复: @j2
  101. TT 说:
    @Stan d Mute

    或者中国发明的火药,他们想不出比制造漂亮的彩色灯光更好的用途了。

    聪明的中国人构想了他们的发明,和平利用采矿和烟花来丰富人类,但邪恶贪婪的西方却偷走了它,到处轰炸和杀戮,直到此刻。 谁有更好的目的?

    • 回复: @JSM
    , @Realist
  102. Anon • 免责声明 说:
    @jim jones

    是的,你忘记了所有用于大规模杀戮和毁灭无辜者的最好的战争机器。 温和的好恐怖分子..假旗911,假新闻宣传机器,金融诈骗,......孟山都的除草剂和转基因有毒食品,橙色剂,艾滋病,......。 人类的发明真是令人惊叹不已。 仍然有核裂变和核聚变炸弹可供烟花搭配,为文明的进步感到自豪。

    • 回复: @Realist
  103. j2 说:
    @Yan Shen

    你写了一个通用的答案,遗漏了我的两个观点,但让我只考虑第二个观点。

    我的第二点是:如果我们谈论智商概况,它可能是性格或其他问题的结果,这些问题很大程度上是由基因决定的,但与智商无关。

    也就是说,非洲人、犹太人,可能还有地中海人,其语言智商比空间智商更强。
    东亚人和北欧人的空间智商比语言智商更强。 这是基因不同造成的,也就是说智力有两种吗? 或者这是由其他遗传性特征(例如性格)引起的,因为我们很容易注意到非洲人和南欧人更具社交性,而
    北方人性格比较内向。

    支持这一观点的是,迄今为止,只有 4-5% 的智商变异是由智商基因解释的,而遗传率为 40-80%。 所以,智商可能不是直接遗传,而是导致智商的性状遗传。 同样,智商与大脑大小呈正相关,但只是微弱的正相关:女性的大脑小得多,但智商几乎相同。 如果大多数可遗传的智商不是由智商基因直接遗传的话,这是可以理解的。

    这很重要,因为你所说的自然优势只是在现在的情况下的自然优势。 中国人、芬兰人,实际上可能有和地中海人一样高的语言智商,但不能用同样的方式表达。 也许可以用另一种方式来表达。 为了更清楚地说明这一点:你可能无法在口头任务中表达口头智商,但可以用书面表达,这是相同的智商但不相同的表达。

    我不认为世界会按照你建议的方向发展:东亚人将主导数学话题,南亚人(是犹太人吗?)主导语言话题。 我们现在看到的并不是自然发展。 这是由一小群想要实现某些目标的人主导的全球化。 他们必须在某个时候被阻止。 那么世界就会以不同的方式发展。 我认为它将更加本地化,​​而不是全球化。 如果更本地化,话题的垄断性就会更少。

    • 回复: @Dmitry
  104. Anon • 免责声明 说:

    东亚人知道什么能填饱肚子。 把事情做好,然后卖掉它以获得回报。 所以他们追求制造实用的东西所需的东西。

    西部白人填饱肚子后,把时间花在做白日梦、密谋、计划、谈话、试图推销想法上。 他们推出了金融投资、华尔街、银行、脸书、谷歌……让他们的制造业陷入尘埃。

    我们将看看谁最终会挨饿。

  105. 我不确定费曼(或其他不平衡的天才)相对于他的科学能力而言缺乏语言能力是由于智力不平衡本身造成的。 就费曼而言,我相信他对智力/学习的语言部分完全不感兴趣。 我认为他大约 99% 的醒着时间以及大部分非醒着时间都花在思考数学和物理问题上。 据称,他与第二任妻子的离婚至少在很大程度上是由于这一特征。 对他来说,即使是敲鼓也是一项科学努力。

    OTOH,他的口语写作和演讲通俗易懂且有趣,无疑表现出了一定的语言能力。

  106. TT 说:
    @Grahamsno(G64)

    谁想成为哥伦布或麦哲伦,海洋令人恐惧地领略我们所生活的无限——然而它们却通过这些无限的航海者航行——这是其他文化所没有的——这些中国机器人甚至没有发现这片巨大的大陆大小澳大利亚国家——就在驴子下面——然而库克船长才在这头克隆人军队的驴子下面发现了一些东西。

    哥伦布、马格南利用明代郑和绘制的地图航海,库克发现了古地图上所示的陆地。 在那里,盎格鲁撒克逊白人尖叫,我们找到了,中国地图是正确的。 他们把罪犯运到那里,像害虫一样杀死当地的原住民,并在这片大陆上殖民罪犯。 他们发现了郑和为了探测这片大地有毒气体而留下的杂种狗,给它起了一个威武的名字塔斯马尼亚虎,..吠叫..它们的虎吼。

    现在豺狼在这里骄傲地嚎叫,我们找到了,我们找到了,中国地图是正确的。 就在他们的屁股下,我们从原住民那里偷走了他们让他们平静了数百年的东西。 我们的 DNA 是维京人,无论我们登陆何处,血流成河,烈火燃烧,我们杀死了男人,强奸了我们的女人,我们抢劫了宝藏,我们占领了土地。 嚎叫..嚎叫...

    • 回复: @Anonymous
  107. Truth 说:
    @Intelligent Dasein

    。 我不知道他们在那里做什么,但无论如何,把人送上月球花费的时间更少

    .

    我们花了六千年的时间才做到这一点,而我们还没有做到。

  108. TT 说:
    @Grahamsno(G64)

    东亚人是一群高智商的克隆人,他们是 BOT 的人类近似,他们的灵魂中没有形而上学 – 什么都没有 – 他们现在不来自印度的宗教 – 佛教 – 现在不存在 –一堆骗钱的机器人,它们几乎是完全的机器——要小心这些机器人 UNZ commenteriat。 它们是最接近人类机器的——它们的灵魂中没有形而上学。

    当我们堕落时,我们的智商下降,我们哭着说更好的人都是机器人。 不知道佛教虽然在被印度教摧残的印度土地上几乎消失,却在世界上蓬勃发展,在佛教国家泰国、缅甸、斯里兰卡、柬埔寨、老挝,在亚洲盛开。 连西方国家的智者都在品尝它的甘露。 下面的青蛙尝不到花粉的甜味,但远处的蝴蝶和蜜蜂却来采集它们。

    豺狼屁股酸痛,智商低下,正在嚎叫…… 他们是高智商克隆人,我们是弱智……嚎叫……嚎叫……

  109. Realist 说:

    “理查德·费曼之所以具有传奇色彩,不仅因为他作为理论物理学家的贡献,还因为他的智商被认为是 125。”

    虽然 125 的智商对于理论物理学家来说可能不太高。 对于普通民众来说,这当然是不小的。 智商 100 对于一般人来说是适中的。

  110. @Anonymous

    “任何生活在亚洲人口较多的地方的人都会认识到一个令人讨厌的事实,即在这些关系中的亚洲女性通常丑陋到非常丑陋”

    我想你必须告诉自己这样才能感觉好一点,但事实上,丑陋、愚蠢的亚洲女孩最终会和丑陋、愚蠢的亚洲男人在一起。 性感的亚洲女孩更喜欢白人男性。

    不过同意黑人的观点。 只有最低层、最绝望的白人女性才会变黑。

    我确实看到,至少在我住的地方,优质的白人女性和亚洲男性在一起,这没问题。

    • 回复: @myself
  111. AaronB 说:

    我只是在等待马拉和登克出现,这样真正的娱乐活动就可以开始了。

    • 回复: @TT
    , @wayfarer
    , @Daniel Chieh
  112. Realist 说:
    @bjondo

    清单上唯一可以被认为发现的项目是抗生素。

    • 回复: @TT
  113. JSM 说:
    @TT

    但邪恶贪婪的西方偷走了它,到处轰炸和杀戮,直到此刻。

    正确的。 我们用火药来清除印度人,这样我们就可以建立一个第一世界国家,让中国人进入,然后告诉我们什么。

  114. TT 说:
    @Bardon Kaldian

    在此我强烈不同意。 中国哲学,由于中国语言的性质和思想结构,根本无法与西方传统相比较,从阿那克西曼德、毕达哥拉斯、恩培多克勒、赫拉克利特、柏拉图、亚里士多德、斯多葛学派、普罗提诺、奥古斯丁、培根1、埃留根纳、笛卡尔、培根2、斯宾诺莎、莱布尼茨、休谟、卢梭、洛克、康德、黑格尔、谢林、叔本华、维科、尼采、马克思、柏格森、海德格尔、斯宾格勒、詹姆斯、维特根斯坦、卡西尔、弗洛伊德(作为心灵形而上学家)、波普尔..

    西方哲学在存在和不存在的所有领域都具有丰富的探究性,而这在中国(和印度)思想(伦理学,认识论,本体论,美学,政治哲学,社会哲学,早期心理学,宗教哲学,宗教哲学)中是根本不存在的。历史,非宗教形而上学,教育,科学哲学等。

    一个傻子根本听不懂中国或印度的语言和思想,声称他们五千年的文明根本无法与我们几百年的西方传统相比较。 你看,我们的埃菲尔铁塔比埃及金字塔还要高,我们的特朗普边境墙会比中国的长城还要长,我们的白宫比印度的泰姬玛哈还要现代化。

    什么易经、道家、道德经、儒家、佛教、孙子兵法、后黑学、孟子、百家、……。 吠陀经、阿育吠陀、印度教、印度哲学、佛教……我一个都不懂,但它肯定比我在英文西方书籍中读到的还要糟糕。 即使中国几千年前的占星记录表明太阳系地球是圆的并绕着太阳旋转,也无法与我们自杀的伽利略伟大发现相提并论。

    只有我们自己才是最伟大的,如果我们遮住自己的眼睛,没有人能看到我们。

    • 回复: @Intelligent Dasein
  115. TT 说:
    @AaronB

    马拉已经消失了一段时间,但丹克很快就会回来。

    • 回复: @denk
  116. Realist 说:
    @TT

    “聪明的中国人构想了他们的发明,和平利用采矿和烟花来丰富人类,但邪恶贪婪的西方却偷走了它,到处轰炸和杀戮,直到此刻。 谁有更好的目的?”

    一百多年来,火药一直没有被用作实用的炸药。

    “……贪婪的韦斯特偷走了它……”

    这他妈到底是什么?

    • 回复: @TT
  117. RudyM 说:
    @j2

    对我来说,内向与语言能力较弱有关是个新闻。 为什么作家中有这么多例外?

    • 回复: @j2
  118. wayfarer 说:
    @AaronB

    小心您想要的东西。

    如果登克出现在这个帖子上,他就会拥有它。

    只是开玩笑。

  119. @Truth

    “你和他打架”只对头脑简单的人有效。

  120. Dmitry 说:
    @Bardon Kaldian

    第一个反对意见:理论物理和数学或多或少不是“相同的”(我什至不会讨论实验物理领域)。

    在非常广泛和普遍的意义上——通常说,与大多数其他数学领域相比,你必须更聪明或更有才华才能在理论物理领域达到相同的专业水平。 以我对学科(物理学)非常短暂、肤浅但痛苦的了解,我可以相信这些说法。

  121. @Truth

    确切地!!! 奥尔登是一个堕落的RCCer。 西方没有亚洲人/黑人/犹太人。 如果没有这些比赛,我们白人/欧洲人也表现得很好。

  122. Dmitry 说:
    @TT

    在所谓的“数学”PISA 考试中,几乎没有任何最基本的数学知识。 整件事有点像个笑话。

    • 回复: @TT
  123. Dmitry 说:
    @j2

    我认为这篇文章与一个人的智商概况及其大学的科学成就联系得太紧密了。 当然,有一定的联系,但不是那么强。

    无论如何,来自世界其他地区(美国以外)的许多(甚至大多数)最优秀的科学家都在美国大学工作。 薪资差距的简单结果是让美国大学从世界其他地方吸纳人才。

    • 回复: @TT
  124. TT 说:
    @anonymous

    我正准备联系经济合作与发展组织 (OECD) 的安德烈亚斯·施莱彻 (Andreas Schleicher),因为 2015 年 PISA 分数显示,中国人的阅读能力几乎和美国人一样糟糕,而他们的数学和科学成绩远远领先。 你帮我(和安德烈亚斯)省了力气。 非常感谢。

    我不知道这是不是因为中文书写系统的困难造成的? 台湾在阅读方面的得分也接近中国,但韩国和日本得分更高,新加坡和香港是得分最高的两个国家。

    中国人读外语(英语)几乎和美国人一样糟糕……哈哈。 上海实际上排名第一(1 年),并且可能在 2010 年再次排名第一,但在与其他省份联合之后,中国的评级下降到接近美国。 另一个因素是考试是用电脑而不是纸笔进行的,一些农村学生对电脑不熟悉/没有经验。

    大多数华人的新加坡和香港(中国)超过了所有英语母语国家,甚至台湾和澳门、日本、韩国、欧盟也超过了美国和英国。 那么中国和东亚人的数学和科学就遥遥领先。 中国人和东亚人一定是作弊了。 欧盟一定是作弊了。 因此,罗伯茨先生如此仁慈,必须要求经合组织进行调查。

    • 回复: @anonymous
  125. TT 说:
    @Realist

    否则新加坡应该因发明碘作为防腐剂而受到认可。:)

  126. Dmitry 说:
    @j2

    东亚人和北欧人的空间智商比语言智商更强。 这是基因不同造成的,也就是说智力有两种吗? 或者这是由其他遗传性特征(例如性格)引起的,因为我们很容易注意到非洲人和南欧人更具社交性,而

    这种评论达到了荒谬和反证的程度,现实或多或少完全反驳了这种概括。 不难指出,北欧创造了人类历史上迄今为止最伟大、最复杂的文学作品,其中俄语达到了顶峰——东亚或许也不甘落后(源氏物语,作者: 11世纪的日本贵妇,被认为是世界上第一部小说)。 在视觉/空间艺术方面,最成功和成就最高的国家是南欧的三个拉丁国家(意大利、西班牙和法国)。

    • 回复: @Daniel Chieh
    , @j2
  127. Danand 说:

    Yan,感谢您为生成和分享此内容所做的努力。 我认为大多数人都会同意“构成智力的组成部分”在不同群体和个人之间存在很大差异。 但从总体上看,从过去和现在的人们的广泛范围来看,他们之间的差距非常大,这是一个艰难的论点。 即使它们是,特定领域/领域智力重要性的历史证据也是有限的(即——从更广泛的整体来看,一个群体在一个领域更有天赋,而在其他领域稍有缺乏,这重要吗?)。 整体平衡的智力,以及随之而来的特质; 内省、同理心、责任心……都很重要。

    历史预示着未来。 查尔斯·默里 (Charles Murray) 的《追求艺术与科学卓越,公元前 800 年至 1950 年》和弗朗西斯·福山信托基金 (Francis Fukuyamas Trust) 是其中的众多凝结之一,它们清楚地表明,由于缺乏更好的术语,“正确的东西”才是最重要的有效占据主导地位。

  128. Dmitry 说:
    @Bardon Kaldian

    在此我强烈不同意。 中国哲学,由于中国语言的性质和思想结构,根本无法与西方传统相比较,从阿那克西曼德、毕达哥拉斯、恩培多克勒、赫拉克利特、柏拉图、亚里士多德、斯多葛学派、普罗提诺、奥古斯丁、培根1、埃留根纳、笛卡尔、培根2、斯宾诺莎、莱布尼茨、休谟、卢梭、洛克、康德、黑格尔、谢林、叔本华、维科、尼采、马克思、柏格森、海德格尔、斯宾格勒、詹姆斯、维特根斯坦、卡西尔、弗洛伊德(作为心灵形而上学家)、波普尔..

    西方哲学在存在和不存在的所有领域都具有丰富的探究性,而这在中国(和印度)思想(伦理学,认识论,本体论,美学,政治哲学,社会哲学,早期心理学,宗教哲学,宗教哲学)中是根本不存在的。历史,非宗教形而上学,教育,科学哲学等。

    虽然我没有哲学方面的专业背景,更不用说相关语言的知识了,但许多伟大的哲学家认为古印度哲学是最受尊敬的传统之一,一些著名哲学家认为它比古希腊传统还要深刻。 要贬低受到叔本华和尼采等主要哲学家赞扬的印度哲学传统,这似乎是一个非常脆弱的立场。

    • 回复: @Bardon Kaldian
  129. Krzys 说:

    我对东亚自然定量智力而不是某些移民过滤器有一个疑问。 为什么尽管中国拥有如此多的数量人才,菲尔兹奖得主的代表却如此之少?

  130. TT 说:
    @Realist

    是的,是由那些想在西方申请专利的同性恋者发明的。 并通过美国疫苗计划用共用针头在非洲传播。 由西方“妓女”和恋童癖者作为礼物带到亚洲,现在由美国向全世界推销,作为一种美化的时尚新生活方式,LBGT也适合每个人,包括基督徒和天主教徒。

    制药巨头正在嘲笑银行,银行称全球化对美国不利。

  131. TT 说:
    @Dmitry

    还有可用的研究资金、最好的设施和可用的同行人才,使最优秀的天才能够实现他的梦想研究。

    当东亚加入并提供更好的条件后,浪潮就会回滚。 美国的研究经费正在枯竭,资金太少,竞争太多。 中国慷慨地提供,所以把奥地利的量子技术带到中国吧。

  132. @TT

    无法与我们伟大的伽利略发现相提并论,但伽利略却被害死了。

    您可能希望了解伽利略审判和软禁的某些事实,首先是 1)他从未被杀。

    不管你是谁,你确实是个挖屁股的白痴。 是的,西方哲学有着东方哲学无法比拟的风格历史、表达深度和创造力。 承认这一点确实没有什么错,所以你试图掩盖这一相当明显的观点并不能很好地反映你的情况。

    • 回复: @TT
  133. Dmitry 说:

    我发现严的文章的开头非常有趣,他在其中写到了费曼的低语文分数。 但他接下来的结论并没有得到支持,这就是我跳过文本其余部分的地方。

    因此,费曼似乎不会拼写并且犯了语法错误,这可以解释他为什么得到低分。

    但如果你在 YouTube 上观看费曼的视频,很明显他能够用语言方式非常聪明地表达自己。

    他在这个领域似乎远远高于你在大学听课的大多数数学家的水平(有时是高水平的研究,然后他们的演讲却如此令人失望)。

    结论并不是说他著名的数学技能和他的口头表达能力之间存在一些差异。 相反,智商测试本身并不能很好地衡量他的能力。 智商测试会惩罚那些不能正确拼写或使用语法或词汇量较小的人。 但这些事情都不是真正非常相关的。

    合理的结论是,费曼在这方面得分较低,因为他的写作/拼写/语法/词汇很差。 但这并不支持他言语上不聪明的观点。 它只是表明“智商”测试的“信噪比”很差,至少对于这种人来说是这样。

    • 回复: @Dmitry
  134. TT 说:
    @Dmitry

    谁在乎呢,拿到分数,加入最好的常春藤,哟……未来是光明的,生活是美好的。 绿卡是自动获得的,如果没有,在家里或您选择的任何地方都有一份很好的研究工作。 如果 PISA 不起作用,这些 PISA 分数高的学生将不会以最高成绩从伊夫里毕业。

    看看Spore、HK、亚洲人如何完美得分? 这是一项偏向西方教育体系的测试,经合组织肯定可以比非英语母语的亚洲人做得更好。 唉……年复一年地沉没,所以他们进口了更多的难民作为补救措施。

    • 回复: @Dmitry
  135. @AaronB

    说实话,你需要辛格真正完善这些评论。 如果你的帖子没有长篇大论地谈论萨蒂和斩首异教徒,说实话,写这些还有什么意义呢?

    • 哈哈: Talha
  136. Dmitry 说:
    @Dmitry

    例如,我发现他的口音和语法(我猜他犯了很多语法错误)很难理解。 再加上他糟糕的拼写能力和糟糕的词汇量,这就是他在口语测试中得分低的原因。 但与此同时,很明显他在言语上非常聪明。 这说明的是这种人的测试不充分,而不是他有某种智力差异。

    • 同意: CanSpeccy
  137. TT 说:
    @Intelligent Dasein

    一个需要称自己为智慧的人通常是非常可疑的。

    您可能希望了解伽利略审判和软禁的某些事实,首先是 1)他从未被杀。

    哎呀,对不起。 怪我童年的故事书是在西方印刷的。 如果我有宽带接入和时间,我会挖掘出2000年前中国占星家已经发现了太阳系的完整细节,太阳和行星的质量,它们的轨道半径,以及如何使用一根绑在两个线上的绳子将饼图导出到小数点后10位用锋利的大头针画出完美的圆,分割成 1024 等份。 算盘还没有发明。 以及他是如何提出所有微积分的……哦……韦斯特发明了微积分。 哈哈。

    伽利略被发现“强烈怀疑异端邪说”,即认为太阳位于宇宙中心一动不动,地球不在其中心并且在运动,并且人们可以持有并捍卫一种可能的观点。在它被宣布违反圣经之后。 他被要求“放弃、咒骂和厌恶”这些观点。 [50]
    根据宗教裁判所的意愿,他被判处正式监禁。 [51] 第二天,他被改为软禁,并度过余生。
    他的违规对话被禁止; 并且在审判中未宣布的一项行动中,禁止出版他的任何作品,包括他将来可能写的任何作品。 [52]

    • 回复: @jilles dykstra
  138. @Dmitry

    这种评论达到了荒谬和反证的程度,现实或多或少完全反驳了这种概括。

    说实话,这就是我对智商争论的普遍感受。 我对大脑研究得越多,就越觉得一个简单的数字比最基本的整体能力衡量标准更重要。

  139. Anonymous • 免责声明 说:
    @TT

    明朝【提督】郑和……

    我们的 DNA 是维京人,

    对于伟大的海军上将和世界探险家郑和来说,可悲的是,中国的习俗使他无法在非洲留下任何DNA痕迹。 郑和小时候被阉割,被卖去当宫廷太监。

  140. Dmitry 说:
    @TT

    谁在乎呢,拿到分数,加入最好的常春藤,哟……未来是光明的,生活是美好的。 绿卡是自动获得的,如果没有,在家里或您选择的任何地方都有一份很好的研究工作。 如果 PISA 不起作用,这些 PISA 分数高的学生将不会以最高成绩从伊夫里毕业。

    看看Spore、HK、亚洲人如何完美得分? 这是一项偏向西方教育体系的测试,经合组织肯定可以比非英语母语的亚洲人做得更好。 唉……年复一年地沉没,所以他们进口了更多的难民作为补救措施。

    虽然我完全同意你第一句话所信奉的人生哲学,但重点仍然是 PISA“数学考试”不包含数学。 您可以下载考试副本并亲自查看。

    问题的部分原因在于它是在 15 岁的孩子身上进行测试,当时大多数年轻人还没有开始学习数学。 这就像测试孩子们的网球技能,看他们上下弹球的能力。 嗯……你需要等到他们真正会打网球,然后结果可能会非常令人惊讶(那些不擅长弹球的人,在他们开始真正学习真正的网球后可能会让你大吃一惊)。

  141. @Realist

    好吧,我们被可靠地告知,“他们将成为康”,但对于卑鄙的白人和他们的中央情报局发明艾滋病等。

  142. 为什么中国人直到20世纪末才开始成为科学强国?

    • 回复: @Stan d Mute
  143. Dmitry 说:
    @Dr. Doom

    史蒂夫·塞勒 (Steve Sailer) 是一位数学奇才

    他显然是一位非常有才华和机智的作家(读起来很有趣),但他不知道如何进行基本统计。

  144. TT 说:
    @Realist

    事实上,在发明枪这种射弹武器之前,古代中国就已经将火药用于各种武器,包括火焰喷射器、火箭、炸弹和地雷。 但中国人用它来防御蒙古人。 蒙古人利用它入侵日本。 欧洲人用它来入侵中国和世界。

    告诉你要保持沉默,巨魔总是使用滑稽的手柄。

    维基百科上的数据

    曾公亮于 1044 年撰写的中文《吴经总要》提供了各种混合物的百科全书参考资料,其中包括石化产品以及大蒜和蜂蜜。 提到了使用虹吸原理的火焰喷射机构以及烟花和火箭的缓慢匹配。 学者们认为,中国人在将火药应用于战争方面几乎没有浪费时间,在发明枪支作为射弹武器之前,他们制造了各种火药武器,包括火焰喷射器、火箭、炸弹和地雷。 [21]

    • 回复: @Stan d Mute
  145. @TT

    巨魔总是使用有趣的手柄

    这里有 1,160 条评论和 170,000 多个字。 几乎所有内容在语法上都是正确的。 据我所知,孔子在这里首先指责我是一个巨魔。 多么讽刺啊? 我很高兴他忽略(未能理解)所提出的观点,而是专注于中国庞大的军事力量(忽略了 50 万名英国士兵用鸦片麻醉剂打昏了这个国家),但我们巨魔很容易被逗乐,不是吗? ? 至少我们不会玩“捡棍子”(儿童游戏)或盯着咖啡渣寻求智慧。

    除此之外,再加上 Unz 的白人“忽略评论者”的魔力 – Sayonara。 (是的,出于善意,我没有提到日本人对他弱小的祖先做了什么)

    • 回复: @TT
  146. @Peripatetic commenter

    为什么中国人直到20世纪末才开始成为科学强国?

    忙着追龙? 屠杀五千万自己的同胞? 让英国人开着帆船向他们扔驴子? 让弱小的日本把屁股交给他们? 躲在墙后? 等待尼克松? 等着俄罗斯教他们怎么做? 摧毁基础设施来建造后院炼钢炉? 读茶叶? 玩智慧棒? 用小针互相戳戳还叫药?

    以上全部? 至少他们一直很忙吧?

  147. 中国人的智商如此之高,为什么白人民族主义者仍然对中国的威胁如此无动于衷?

    希望看到有关印度智商趋势的最新更新信息。

    redpillindian.blogspot.com

  148. myself 说:
    @Thought Criminal

    关于第2点)“近代东亚从未有重大科学发现”。

    好吧,如果你把现代算作从过去几个世纪开始,比如说从公元 1500 年到现在,那么这是显而易见的,因为东亚人自己也完全承认和承认。 “现代”与“西方统治”时代重合是有充分理由的。 可以公平地说,现代=西方时代。
    毫无疑问,东亚确实受到陈旧过时的思想体系的阻碍——或者换句话说,受到非常落后的文化的阻碍。 事实上,他们自己认为应该放弃的文化,无论好坏。

    你们将“文化挪用”白人的思想。

    还有第2点)“事实上,几千年来东亚人甚至无法在数学上“发明轮子”。 他们没有提出基本的符号、基本的证明或任何东西。 他们所做的一切都是从印度偷来的。”

    不,在这一点上我谨表示不同意。 我不了解广义的东亚人,但中国和印度之间几乎没有数学交流,中国和欧亚大陆西部之间更是如此,至少在公元 200 年左右是这样。 即使从那时起,借鉴的想法也很少。 实际上,随着 16 世纪耶稣会传教士的到来,明显优越的“现代西方数学”思想才相当席卷了中国和整个东亚。

    从本质上讲,中国独立发展了数学——直到被现代时代超越。

    关于第4点)“中国的优势不是在科学上,而是在前科学实验上。”

    正是因为在大约 1500 年之前,任何地方的文化都没有任何可以谈论的“科学”,甚至它的最终发展者——西北欧的文化也没有。 (注意,也不是整个欧洲,我们谈论的是欧洲内部一个非常小的、特定的文化群体)。 例如,古希腊人和古罗马人没有“现代科学”——他们有探究和结构化思维,其他一些古典文化也是如此。

    关于第3点)“东亚人在文学、诗歌、哲学等方面并不逊色”

    我不得不同意,但我必须指出显而易见的事情,那就是在对一系列令人难以置信的学科进行广泛、深入和严格的考察时,特别是对中国作为一种“祖先”文明进行考察与欧亚大陆东部的其他地区相比,它们往往是从非常不同的起始假设开始的,因此得出了截然不同的结论,并颁布了与欧亚大陆南部和西部/北非截然不同的范式、观点和细微差别。

    对于西方人来说,结论和范式往往不仅看起来“陌生”,而且完全“陌生”。

    顺便说一句,印度次大陆思想家的探究方向和结论对于南亚以外的思想家来说也很陌生。

    • 回复: @Dmitry
  149. myself 说:
    @Stan d Mute

    东亚人的智商比白人和印度人的智商更接近这一概念很可能有一定道理,但我从未见过或遇到过这样的适当研究。

    相比之下,研究人员注意到亚洲人的总体智商和数学/语言分裂已经有一段时间了,但他们都没有提到智商聚类现象。

    也许亚洲人的创造力不那么高,并不在于智商遗传,而是在于从众的文化范式?

    • 回复: @Stan d Mute
  150. myself 说:
    @JSM

    任何人都毫无疑问地认为过去500年属于西方。

    亚洲人自己也这么说。 正是他们接受了这个事实,而不是抱怨和抱怨,才让他们得以发展。

    但我们不应该骄傲自大。 历史永远不会停滞不前,永远不会。 我们统治了过去 500 年。 过去五千年我们并没有占据统治地位。

    而我们目前的主导地位是。 。 。 看起来很脆弱。

    • 回复: @JSM
    , @Stan d Mute
  151. Svigor 说:

    加藤! 今天不行,加藤!

  152. JSM 说:
    @myself


    允许高深莫测的亚洲人进入我们的大学和行业,他们可以窃取军事和工业机密,然后带回 Chicom 中央委员会……这看起来很愚蠢。

  153. @myself

    也许亚洲人的创造力不那么高,并不在于智商遗传,而是在于从众的文化范式?

    东亚的从众心理与欧洲的从众心态(例如清教主义或犹太民族主义)有何不同? 我对平均值的聚类可能是错误的,我想我已经看到它被研究和证实了,它符合我用我的谎言眼睛看到的,但也许不是。 不管怎样,我的目标是激发更多的思考,我很高兴看到我至少在一个想法上取得了成功。 文化无疑是一个重要因素,但我的问题始终是:“种族是一种社会建构,还是社会是一种种族建构?” 因果关系的箭头究竟指向哪个方向?

  154. myself 说:
    @Thought Criminal

    我会同意的。

    中国宇宙观的核心,如果你深入挖掘的话,就会发现任何思想、体系、信仰、哲学或一系列行动,如果推得太远,都将不可避免地产生相反的原则这将迫使它重新与更大的宇宙和谐相处。 人有代理权,但宇宙(“现实”,如果你愿意的话)是最终的仲裁者。 人不会违背现实。

    按照这种观点,尽管人的本性是有能动性、有意志和奋斗,但最终的结果并不完全由他决定,只是部分由他决定。 他的行为是他自己的,结果可能不是——这没有理由感到内疚或羞耻。 《易经》中说:“行义无罪”。

    我喜欢你对中华文明的描述——总是在野蛮/高级文明/颓废的光谱上——一个光谱,永远没有终点!

    为了达到扬升,“神化”就是停滞,因此进入一种遗忘状态,没有任何人、任何人、任何种族、民族或文明都无法从中恢复! (确实没有一个文明能够从灭亡中恢复过来——孟菲斯、底比斯、摩亨佐-达罗、雅典、罗马或拜占庭现在在哪里?)

    永远不会坠入地狱(和遗忘),也永远不会升入天堂(和遗忘),而是将宇宙视为无限和永恒,并将凡人在其中的旅程视为无限和永恒。

    这是中华文明的核心思想,也许早于中国所有其他思想流派。

    现在,这是一种好的甚至有用的生活方式吗? 如果我知道的话见鬼。

  155. TT 说:
    @Stan d Mute

    抱歉,我把你误认为是巨魔,开枪打错人了。 你真是个该死的pt,那些清朝的混蛋……你的把柄看起来确实很有趣,不是吗? 平常心,别小气,我们也被别人骂过。

  156. bjondo 说:

    生活中的重大苦难和毫无意义、毫无价值的发明是高智商拥有者的礼物吗?

    似乎大多数高智商拥有者都会花费、浪费税款来发明对其他高智商拥有者的发明的修正。 一个永远持续下去的过程。

    木工、管道工、农业智商是如何测量的。

    谁对美好、体面的生活更有价值:费曼一家、沃尔福威茨一家、罗伯特鲁宾一家还是优秀的木匠/水管工/马铃薯种植者。

    爱因斯坦能找到走出卫生间的路吗? 对于享受日常生活来说,他的抄袭是否比百吉饼更有价值?百吉饼是他所谓的高智商邪教偷来的食品,并声称是自己的。

    智商测试对于筛选那些应该被淘汰的人来说可能是最有用的。

    • 回复: @Daniel Chieh
  157. Svigor 说:

    任何人都毫无疑问地认为过去500年属于西方。

    亚洲人自己也这么说。 正是他们接受了这个事实,而不是抱怨和抱怨,才让他们得以发展。

    但我们不应该骄傲自大。 历史永远不会停滞不前,永远不会。 我们统治了过去 500 年。 过去五千年我们并没有占据统治地位。

    而我们目前的主导地位是。 。 。 看起来很脆弱。

    我不在乎统治力。

    统治地位是一个相对的事物; 如果鲍勃有 1 亿,汤姆有 100 亿,那么鲍勃就有“统治地位”。 如果汤姆再赚十亿美元,他现在就拥有了“主导地位”。 但是WGAF,除了那些痴迷于这些事情的人?*两人的财富都远远超出了他们的需要。 汤姆并不因为拥有的东西比鲍勃少而贫穷,鲍勃并不因为拥有的东西比汤姆多而富有,反之亦然。

    当然,如果人们需要国家主导地位来激励他们为你提供最好的、伟大的、更多的权力,那很好。 我个人不相信全国范围内攀比的事情。 我相信设定我们的国家目标并实现它们。

    仅中国就有超过十亿中国人。 人们非常希望一个人口如此众多、平均智商高于欧洲平均水平的国家,能够在 GDP 方面超越美国。 我们这个国家只有大约 200 亿左右的白人,他们必须拖着一群 NAM 冲过终点线。

    *我防御性地引用西方记录,作为告诉种族灭绝者滚蛋的一种方式; 我们不需要你们的“认知精英”。 否则ICGAF。

    ***

    我希望中国能真正与我们竞争,特别是在医学、遗传学、替代能源、太空探索等重要领域。他们创造的创新将造福全人类; 同样,犹太人也从冯·布劳恩的工作中受益。

  158. utu 说:

    智商还是数学/言语分裂?

    错误的替代方案。 答案是:都不是。

    日本、中国、韩国现在已经具备了领导的一切条件。 即使欧盟和美国从地球表面消失,世界也会生存并顺利进步。 他们不再需要西方了。

    • 回复: @Truth
  159. @myself

    但我们不应该骄傲自大。 历史永远不会停滞不前,永远不会。 我们统治了过去 500 年。 过去五千年我们并没有占据统治地位。

    是……又不是。游戏还远没有结束。 但是,当我们发现、征服并驯服了这个星球时,他们的屁股却被两个小小的岛屿交给了他们,并在他们玩游戏和吸鸦片的时候建造了一堵墙躲在后面。 现在,首先在俄罗斯的帮助下,然后在我们的帮助下,他们确实正在迎头赶上,并且很可能很快就会让我们黯然失色。 在这一点上,我们似乎比其他任何事情都更加狂热地决心种族灭绝自己。 我的最佳实践解决方案是无耻地窃取对方最擅长的东西。 如果游戏需要一个失败者,那就让这个失败者成为那些对我们物种甚至他们自己的亚种的进步没有做出任何贡献的人(我不需要说出他们的名字)。 我们还有很多东西要向东亚特别学习,他们也有很多东西要向我们学习。 即使我们在一起,距离理解我们的宇宙还有很长的路要走。

  160. @wayfarer

    我同意——那些有很大差异的领域,特别是与遗留系统相比,需要个人判断的领域将是最难实现自动化的。 事实上,我认为我们不会很快看到人工智能带来的“失业”; 更令人担忧的是当前大数据趋势的加速,以一种以前不可能的方式集中信息。

  161. @bjondo

    木工、管道工、农业智商是如何测量的。

    我不知道,您是否喜欢管道系统的冶金、通过复杂的物流渠道获得木材或通过大规模、优化的农业项目获得现成的食物?

    解决现代性的许多缺陷很容易,但它也给社会带来了客观的、可衡量的好处,而且在引入互联网的过程中,可以说创造了现实的全新方面。 在这样做的过程中,我们最终也可能会创造出我们的替代品,但这难道不是所有生命的最终目标吗? 如果我们要创造宙斯,成为克罗诺斯有那么可怕吗?

    • 回复: @wayfarer
  162. myself 说:
    @AaronB

    根据中国的传统观点,在过去的五百年里,是西方而不是其他地方更和谐、更符合宇宙的流动,因此西方在物质乃至哲学和艺术上取得了全面的成功。

    更进一步说,随着中国逐渐恢复与历史要求的和谐与平衡(历史是永恒宇宙流动的外在表现——道家思想),它经历了重大进步。

    这种想法认为,西方所谓的萎靡不振是由于过去几十年来与宇宙的错位造成的,但与中国无关——西方正在走自己的轨道,无论它是什么。

    至于日本,他们有一个适合特定时代的获胜公式,但后来他们变得死板,不愿改变,他们坚持自己的尝试和真实的公式。 我在中国采访过的人都应该有意识地避免这种错误。

    不过,有一项观察,我认为是有效的。

    日本近年来并未从内部发起重大变革。 明治维新是对西方的反应,而二战后时代又是对盟军失败的反应。 毫无疑问,反应非常出色。

    相比之下,中国虽然为了适应现代化的需要而抛弃了帝制,但也发生了一场巨大的内战,摧毁了旧的社会秩序,并给予共产党极端的控制。 然后,它再次从内部经历了改革开放时代,使意识形态人士失去了威信,并将其赶下了权力。

    日本在现代化方面取得的更大成功意味着它没有经历中国认为必要的革命。 另一方面,尽管经历了许多失败,中国却表现出更愿意进行自我发起的彻底变革。

    • 回复: @jilles dykstra
  163. 我不认为欧洲大陆的工程人才普遍下降。 在这方面落后的国家是忽视制造业并大幅削减关税的盎格鲁国家。 欧洲在设计和制造机床、重型机械、农业设备、船舶、武器系统等方面遥遥领先。日本在制造价格实惠、可靠的汽车方面可能处于世界领先地位,但欧洲在生产卡车、货车和汽车方面仍然非常有竞争力。创新的豪华汽车。

    然而,您可能是对的,西方在设计和制造电子硬件方面落后了。 消费电子产品现在完全由东亚主导,而设计精良的欧洲汽车往往因不可靠的电子产品而令人失望。

  164. bjondo 说:

    我不知道,您是否喜欢管道系统的冶金、通过复杂的物流渠道获得木材或通过大规模、优化的农业项目获得现成的食物?

    嗯,木材来自一个简单的运输系统——附近的森林和孟山都等公司的各种高智商科学家进行的大规模、优化的农业项目,产生了大量吨垃圾食品。 也减少了品种。

    解决现代性的许多缺陷很容易,但它也给社会带来了客观的、可衡量的好处,而且在引入互联网的过程中,可以说创造了现实的全新方面。

    互联网作为隐藏在高智商人群中的信息和交流的提供者固然很好,但取代社交互动则不然,没有互联网,生活也很美好。

    在这样做的过程中,我们最终也可能会创造出我们的替代品,但这难道不是所有生命的最终目标吗? 如果我们要创造宙斯,成为克罗诺斯有那么可怕吗?

    笨蛋

    • 回复: @Daniel Chieh
  165. @bjondo

    您始终可以选择退出婴儿死亡率为 10-30% 的美好世界。 也许人类前进方向最清楚的迹象是,推动这种加速前进的不是力量,而是力量。 这是个人集体的选择。 没有人坚持认为我们都有一部智能手机,现在可以几乎 24/7 跟踪我们 - 但它的采用已接近普遍。

    人类有一定的趋势。 不管我们怎么想,这些都是可以预测的。 机器在预测分析方面比任何人类都要好得多。

    • 回复: @bjondo
  166. Talha 说:
    @AaronB

    正是,让他们在物质的巅峰享受时光; 然后他们可以决定是否喜欢那里。 我们可以有很多新发明,有中文的原始名称。 我们将从他们的发明中受益; 好吧,假设他们不创造人工智能驱动的杀手级纳米机器人云。

    我只是跟一个弟兄说,“现代”建筑在中国的大城市里开始兴起,真是太可惜了——与风格丰富、风格独特的中国古典建筑相比,它是那么丑陋、没有灵魂。

    有趣的线程。

    和平:

  167. wayfarer 说:
    @Daniel Chieh

    一个熟练的木匠,就是一个熟练的几何学家。

    关于液压长笛演奏机的设计和构造的论文。 这篇论文的作者是“木匠和几何学家阿波罗尼乌斯 (Apollonius the木匠和几何学家)”,有些人已经确认了他的身份。

    https://en.wikipedia.org/wiki/Apollonius_of_Tyana
    https://www.britannica.com/biography/Apollonius-of-Perga
    https://www.qdl.qa/en/archive/81055/vdc_100023698323.0x000003

    • 回复: @wayfarer
  168. myself 说:
    @Johnny Smoggins

    亚洲的亚裔男性并不在意当地的亚裔女性“变白”。

    我认为是因为2个因素:

    1)亚洲当地的顶级女性,就像世界各地的顶级女性一样,只想与自己种族的顶级男性在一起。
    你会看到一些漂亮的当地女孩和白人男人在一起,但真正的淘汰者总是和他们自己的同类在一起。 这关系到他们和他们的孩子的基因传播(生殖成功,而不是真正的性)和社会地位。 既然是亚洲,他们自然会被自己的男人所吸引。

    2) 嗯,她来自的地方还有很多! 如果白人想要一些,也不会减少供应一丁点。

    • 回复: @Truth
  169. Yan Shen 说:
    @AaronB

    我的观点并不是说任何特定群体必然会脱颖而出,而是一旦障碍逐渐消除,不同群体的人最终会专注于他们自然倾向的领域。

    尽管我确实主张数学能力高于语言能力,但预测英语圈将仍然是生命科学和医学领域的总体领导者并不是一件微不足道的事情。

    我确实认为硬件胜过软件,制造业胜过服务,但这也许是后续文章的内容。

    你将我的文章描述为为了验证目的而写的神话,但我主要认为自己做出了一组相对保守的预测,这些预测植根于 HBD 并得到清晰可观察的数据点的支持,例如我引用的莱顿排名等。

  170. anonymous • 免责声明 说:
    @TT

    2015 年美国 PISA 阅读

    https://nces.ed.gov/surveys/pisa/pisa2015/pisa2015highlights_4f.asp

    白人:526
    亚洲人:527

    2012 年美国 PISA 阅读

    https://nces.ed.gov/surveys/pisa/pisa2012/pisa2012highlights_5e_1.asp

    白人:519
    亚洲人:550

    在阅读测试中,亚洲人相对于白人来说似乎并不处于劣势。

    • 回复: @TT
  171. myself 说:

    所有这些关于文化和文明兴衰的哲学讨论,我们所有人(包括我)都未能指出一些摆在我们面前的事情:

    在我们人类文明有记载的历史中,过去五个世纪由一种充满活力的文化“西方”完全占据主导地位是一种反常现象,而不是常态。

    压倒性的常态是几个合作/竞争的文明同时充满活力和进步,每一个文明都充满活力和活力,尽管各自独立,但它们都在推动人类社会的发展。

    在漫长的历史时期,整个欧亚大陆以及北非、东非、中美洲和南美洲都是活跃和充满活力的。

    对于西方的崛起时期,我的看法是“必须有人成为第一”——首先进入现代性时代,首先发展科学,首先质疑过去的古板观念。 有人是西方,我们已经从中受益。

    首先,但不仅仅是。

    成为第一并不意味着我们将永远垄断权力和现代性。 英国是第一个工业国家,但并不是唯一的工业国家。 一个好的想法最终会被全人类吸收和内化。

    西方文明的有用思想也是如此。 西方的方式不可避免地被全世界直接“采用”。

    如果西方应该“失去”其迄今为止的特权地位呢? 那又怎样呢? 作为众多种族中的一个强大种族有什么问题呢? 是因为别人多了,我们就少了?

    宇宙是零和游戏吗?

  172. Truth 说:
    @utu

    这件事最好快点发生,因为他们都没有孩子。

  173. Truth 说:
    @myself

    亚洲的亚裔男性并不在意当地的亚裔女性“变白”。

    这不是我读过的。

  174. @TT

    这与太阳或地球无关,耶稣会士的威望受到威胁。
    耶稣会士对太阳系或平坦的地球根本不感兴趣

    彼得罗·雷东迪(Pietro Redondi),加利利(Galilei),凯特(Galilei),凯旋(De politieke machtsstrijd rond het proce tegen)伽利略·加利利(Galileo Galilei),1633年,1989年,阿姆斯特丹(Galileo eretico,1983年,1989年,都灵)。

    'politieke machtsstrijd' = 政治权力斗争,'proces' = 审判,'ketter' = 异端

  175. @Dmitry

    我并不是“打折”它。 我读过 Radhakrishnan 撰写的精彩阐述(http://gen.lib.rus.ec/book/index.php?md5=B21E1496C0CDE537858B7E5AF99D9C4F )和达斯古普塔(http://gen.lib.rus.ec/book/index.php?md5=7567AAE467409E9A3EA24D1DB4986946),以及 Eliade、Coomarswamy 等人的学术著作。 在各种主题上,我想说印度思想在形而上学方面是深刻的,而在大多数其他领域却是狭窄的。 与希腊和西方哲学不同,它主要仍然是对宗教经典的阐述,充满神话隐喻,从未作为一个独立的领域脱颖而出。

  176. @Yan Shen

    我不知道你是否读过它(至少部分地读过),但我会向任何对中国思想感兴趣的人推荐陈永捷的调查: http://gen.lib.rus.ec/book/index.php?md5=A13880637E7E9F98EF712AE179E38891

    我知道有更新的展览(Van Norden 等),但这部 1960 世纪 XNUMX 年代的经典作品仍然是我的最爱。

  177. 顺便问一下,“东方”和“西方”之间或许还有更深刻的差异呢? 我不知道这是否已得到证实,但是......

    https://www.newscientist.com/article/dn7882-westerners-and-easterners-see-the-world-differently/

    西方人和东方人看待世界的方式不同

    从字面上看,中国人和美国人对世界的看法是不同的。 美国安娜堡密歇根大学的心理学家表示,美国人关注照片的中心对象,而中国人更关注图像的整体。

    “有大量轶事证据表明,西方人和东亚人有着截然不同的世界观,”进行这项研究的理查德·尼斯贝特 (Richard Nisbett) 解释道。 “美国人会分析性地分解事物,专注于将物体分类并找出它们应该遵守的规则,“ 他说。

    相比之下,东亚人有一种更整体的哲学,看待事物与整体的关系。 “打个比方,美国人看到的事物是非黑即白的,而东亚人则看到更多的灰色阴影,”尼斯贝特说。 “我们想设计一个实验,看看这是否会转化为他们实际看到的东西的字面差异。”
    ......

    • 回复: @Daniel Chieh
  178. bjondo 说:
    @Daniel Chieh

    机器在预测分析方面比任何人类都要好得多。

    机器预测在多大程度上依赖于人类的操纵、宣传?

    是的,婴儿死亡率是真正的心弦拉动者。 堕胎也与贫铀、疫苗和现代食品一起导致婴儿死亡。 有时,死亡,现代的、高智商造成的死亡,来得更慢、更痛苦、更丑陋。

    选择退出大自然的自然世界,而不是一个由贪婪和自私的高智商创造的世界。

    • 回复: @Daniel Chieh
  179. j2 说: • 您的网站
    @RudyM

    我发现你有点怀疑,所以让我们参考 unz 评论中的 James Thomson
    https://www.unz.com/jthompson/lapps-finns-cold-winters-and/
    “这些数据表明,萨米人与世界上大多数人具有相同的特征,即相对于高加索人标准,他们在空间测试上的表现比在语言测试上的表现更好。 两个群体的非语言技能都比语言技能更强,这可能是在野外寻找猎物的猎人所期望的。”

    通过少量文献检索可以发现,黑人和犹太人的语言智商较强,空间智商较弱。 我认为这对于安纳托利亚农民的后裔来说是正确的,因此对于地中海国家来说也是如此。 上面你已经证实,在北方,空间智商比语言智商强。 汤姆森发现这种智商特征在世界上最常见,因为东亚人也有这种特征。

    毫无疑问,东亚人和北欧人更加内向,因此存在相关性。 相关性并不一定意味着因果关系,但在这种情况下可能存在因果关系。

    有些评论声称我做了一个荒谬的推理,并认为北欧人的语言智商不可能低于空间智商,因为北欧人有这么多优秀的作家。
    人口。 这并不会使众所周知的智商概况无效。

    内向的人既可以成为优秀的作家,也可以成为优秀的科学家。 即使平均智商更倾向于空间,也不意味着你无法从这些国家找到语言天赋的人。 言语智商通过阅读而提高,而不仅仅是通过说话,因此您可能会发现许多作家都是内向的人,并且言语智商很高。

    从文学成就来看,北欧和东亚的语言智商似乎特别高,但事实证明,这些人群的平均语言智商比空间智商要弱。 语言智商仍然很高,因为这些人群的智商很高,但出色的文学成就并不意味着该人群的语言智商处于平均水平。

    特别是,它并没有使所提出的论点(未经证实)无效,即平均语言智商可能因内向文化而较低。

  180. j2 说:
    @Dmitry

    当您提到北欧时,您指的可能不是相同的人口。 俄罗斯幅员辽阔,只有上部是北欧。 我的意思是最北欧,那里的智商概况就是我所说的。 这并非荒唐可笑,而是智商测试中已知的事实。 事实上,来自卡雷利亚的俄罗斯人在基因上与大多数北欧人相似,并且可能具有相似的智商特征。

    人群的平均智商与人群的杰出成就之间没有直接联系。

    有关更多详细信息,请阅读我写给 RudyM 的内容。

    顺便提一句。 我继承了俄罗斯作家的整个系列并阅读了所有书籍。 它们并不是真正的人类文学的顶峰。 《战争与和平》就像亭子文学,只是更长,但托尔斯泰不是北方人,他的家人从切尔尼戈夫搬到了莫斯科。 那就是中东欧。

  181. @bjondo

    证据可能就是现实本身。 有些社会试图拒绝现代性的某些方面,我研究了其中的一些,但最终,它们基本上以这种或那种方式消亡。 例如,你能想象一个拒绝计算机的社会吗? 它们存在于许多地方,这些地方的电子产品仍然昂贵,超出了大多数人的承受范围。 但随后精英阶层开始使用它们,它很快就变得既实用又成为地位的象征,而且随着使用价格的下降,至少整个下一代都会追求它。 如果在当地遭到拒绝,他们就会从任何渠道移民出去。

    在很多方面,我认为这就像拒绝枪支,因为它们对依赖敏捷和力量的种姓社会产生负面影响。 甚至是基因缺陷。 当然,你可以尝试假装枪支暂时不存在,但随后其他使用枪支的社会就会把它强加给你。

    重要的是有效的方法。 我们不再生活在马克斯·韦伯的神秘和距离的“魔法花园”中。 现在,每一项优势都在全世界范围内传播,任何不接受它的社会就会落后、成员流失、本质上枯竭。 除非你有办法进入朝鲜你的社会(及其随之而来的成本),否则就不可能避免技术给世界带来的变化,并被迫采取红皇后策略。

    人们可以哀悼已经过去的事情,但即将发生的事情是不可避免的,就像宇宙的热寂一样。

    • 回复: @bjondo
  182. @Bardon Kaldian

    请注意,他的论点基本上是萨皮尔-沃尔夫假说的扩展版本,尚未得到现代研究的支持; 在理查德·尼斯贝特的书中 思想地理他的测试发现,华裔美国人的思维方式与欧洲人一样“分析性”,而对于一生在东亚长大的欧洲公民来说,反之亦然,这高度暗示了文化的结果; 他相信这几乎完全是通过语言实现的。

    • 回复: @Hacienda
  183. TT 说:
    @anonymous

    是的,事实上,东亚人在偏向于经合组织教育体系的测试中占据了所有最高排名,而教育强国美国/英国/澳大利亚都排名较低,这是怎么回事?

    事实上,2012 年上海在所有三门科目上再次名列前茅,2009 年数学成绩也提高到 613 分,相当于比普通学生领先 3 个学年。

    中国参加2012年测试的地区仅限于上海、香港和澳门作为单独的实体。 2012年,上海第二次参加,三科再次名列前茅,各科成绩较2009年有所提高。 上海的数学成绩为613分,比平均分高出113分,比一般国家的学生领先约3个学年。 教育专家们争论这一结果在多大程度上反映了中国整体教育体系的质量,并指出上海比中国其他地区拥有更多的财富和更高的薪水。 [32] 香港在阅读和科学方面排名第二,在数学方面排名第三。

    PISA 是否变得过分强调国家教育政策,尤其是在东亚? 这些东亚裔人再次主导了智商排名。

    PISA 数据“越来越多地塑造、定义和评估国家/联邦教育系统的关键目标”。 [7] 这意味着PISA测试的制定者——例如选择评估和不评估的内容——拥有相当大的权力来制定教育辩论的条款,并指导全球许多国家的教育改革。 [7]

    • 回复: @anonymous
  184. @myself

    在我看来,中国保留了帝国制度,就像俄罗斯保留了沙皇制度一样。
    只是统治者的称呼不同而已。
    文化几乎没有改变。
    正如一位荷兰政治家所说:“法国是君主制国家,君主由选举产生,荷兰是国家元首世袭的共和国”。
    马克龙的绰号是木星。
    默克尔的行为难道不是德皇吗?
    正如宣言签署时一样,美国仍然由少数富人组成的寡头统治。
    比利时,一团糟,正如一个具有该国籍的人所说的“比利时,它不存在”。
    佛兰德斯人和瓦隆人仍在互相争斗。
    西班牙仍处于法西斯统治之下,正如我们在马德里对待巴塞罗那的方式中看到的那样,内战又重演了。
    意大利就像中世纪的城邦国家一样混乱。
    等等等等。

    • 回复: @Daniel Chieh
  185. Hacienda 说:
    @Daniel Chieh

    你读中文吗? 汉字阅读似乎比阅读需要更大的焦点区域
    和英文单词。 阅读英语只需在流利后浏览字母顶部即可。 在我看来,中国读者即使很流利,也必须上下左右扫视才能理解一个中文单词。 这会成为眼球看事物的习惯吗? 这就是为什么“直视中国人的眼睛”似乎很困难吗?

    • 回复: @Daniel Chieh
  186. @Hacienda

    我是这么做的,虽然我是在学了英语之后才学汉语的。 这是可能的,尽管有趣的是尼斯贝特博士研究中的一些分组,例如日本观众如何倾向于将“兔子”与“草”、“鸡”与“种子”联系起来,而他的美国小组则将“兔子”联系起来与“鸡”和“草”与“种子”。

    顺便说一句,我正在读尼斯贝特博士的 思维软件 现在。 他仍然是少数几个主张环境影响智力的原则性倡导者之一,他承认智商是真实的,并且遗传具有重要组成部分,同时也关注文化和环境对智力的影响,因为整个童年时期的神经发生和大脑重构。

    • 回复: @utu
    , @Truth
    , @Hacienda
  187. Joe Webb 说:

    所有这些都有助于解释东方专制主义。 中国人和亚洲人普遍不能用数字来思考社会现象。 因此,正如我们常说的红色中国或朝鲜是专制国家。

    亚洲人在大学里的作弊率大约是白人的 5 倍……我想这在 Unz 的某个地方有报道过。

    把世界变成东方专制主义,无论如何,这就是它的趋势,从中东到南美洲的混血儿到西亚,等等。 唯一真正人道的人是白人,他们在这方面走得太远了。 我们都是平等的,首先是白痴的救世主心理。

    亚洲人不能不关心民主标准和言论自由。 他们似乎对金钱非常感兴趣,比白人更感兴趣。

    我询问了两位不同的中国年轻女孩,她们对白人男性的明显偏好。 在门洛帕克和帕洛阿尔托……在不同的事件中,他们都说了同样的话。 白人对你更好,对亚洲男人来说这都是生意。

    黄祸。

    乔·韦伯

    • 哈哈: Daniel Chieh
  188. anonymous • 免责声明 说:
    @TT

    事实上,2012 年上海在所有三门科目上再次名列前茅,2009 年数学成绩也提高到 613 分,相当于比普通学生领先 3 个学年。

    我并不觉得上海有什么令人惊讶的,因为它是中国杰出的教育强国之一。 此外,我相信他们在 PISA 上的缺席率也很高:

    https://www.unz.com/isteve/pisa-which-countries-not-to-trust/

    就我而言,我实际上并没有非常认真地对待智商测试,而且我相信它们的用处非常有限。 几周前,我开始研究林恩和许多其他智商研究人员的智商数据,我开始注意到智商分数的收集方式、平均分数的指定方式以及结果如何不按地区加权等各种奇怪的现象。人口得出“全国”智商分数。 还有一个问题是,朝鲜的智商在世界上经常被列为105-106。 在我看来,智商分数应该是测试分数; 朝鲜什么时候测过智商? 朝鲜农村贫困人口接受过检测吗? 所有这些让我对智商统计数据非常怀疑,尤其是老年人的智商统计数据。

    最近的国际标准化测试,如 PISA、timss 等。 看起来好一些,但同样,也只是那么有用。 尽管如此,我确实发现有趣的是,PISA 和 PIRLS 等测试并未显示亚洲和欧洲国家之间的阅读成绩存在任何显着差距。

    如果人们正在寻找亚洲人和欧洲人不同属性的可能解释,那么在我看来,可能值得研究智商以外的平均特征的差异,例如性格、气质等。这并不是假设是否或这些特征并非源于环境、情境、社会、文化或遗传。 只是为了看看它们是否存在以及它们是否有任何解释力。 只是我的两分钱。

  189. bjondo 说:
    @Daniel Chieh

    需要除草的原因。 为了拯救人类。

  190. utu 说:
    @Daniel Chieh

    文化和环境对智力的影响

    幸运的是,我们在这篇文章中几乎没有评论试图将注意力转移到文化上,该文章的作者因被简单的 HBD 概念引入歧途而错过了讲述西方和东方有趣故事的机会。 但至少他拒绝了基于 IQ/g 的最原始版本,并承认至少必须调用基于数学和语言的二轴空间,这相对于基于 g 的还原论来说是一个适度的改进。

  191. 乔“阿纳托利是烤肉串”韦伯。

    您的预测能力已经过测试,并且……发现存在不足。

  192. denk 说:
    @TT

    我寄居去参加我的会议 周公 现在。

    除非出现不可预见的情况, 可能 回到这里,也许作为旁观者,

    h

  193. Jeff77450 说:

    我不明白为什么每个人都对中国人不屑一顾。 有没有人怀疑没有西方投资和技术,包括超过1万亿美元的知识产权盗窃,他们是否还会在下一次水稻收获时大汗淋漓? (夸大效果)。

    • 回复: @TT
  194. Truth 说:
    @Daniel Chieh

    我愿意,虽然我学了英语之后又学了汉语

    丹尼,你刚刚毁了我。 一直以来我都以为你是合法的。 你出生证明上的名字可能实际上是“丹尼尔”,而不是 Xan-Chol 之类的。

    • 回复: @Daniel Chieh
  195. @Truth

    我的名字显然不是丹尼尔。 我不是一个自我认同的白痴。 和许多中国人一样,我的本名是有意义的,蕴含着父母对我未来的希望和祝愿。

    我在学说英语的同时说中文(基本上从出生起),但作为一种书面语言,中文更难学,尤其是在我不沉浸其中的环境中。

    • 回复: @Truth
  196. Santoculto 说:

    言语智力有两种类型,或者说它的两个重要目的:

    – 正确理解和应用概念; 创建新的[希望有用且基于事实]

    – 记住/掌握书写[拼写/语法]的“正确方法”和词汇量。

    第一个似乎更实用、基于启发式、客观有用,而第二个似乎更有助于表明社会和/或意识形态地位以及掌握社会命令,例如,用那种或这种语言“正确”书写。

    语言智商测试的一个大问题似乎是它们简单地忽略了每种语言的特性,因此从先验角度来看,如果你擅长罕见的母语或者如果你无法正确学习另一种语言,那么这似乎是相对的,例如法语。

    但是,如果将一种语言与另一种语言进行比较以象征现实[及其各个元素],分析一种语言的精确/事实和全面性也许会很有趣,因为我们可以拥有一种非常复杂的古代和稀有语言,但不是那么有用,甚至很复杂了解沟通的根本目的是否是尽可能以最佳/干净/互惠/有效的方式进行。

  197. Truth 说:
    @Daniel Chieh

    所以你不像陈康生那样使用“Bobby”或“Jackie”等可爱的盎格鲁昵称之一?

    https://notednames.com/Movies/Actor/Jackie-Chan-Birthday-Real-Name-Age-Weight-Height/

    • 回复: @Daniel Chieh
    , @Talha
  198. utu 说:
    @Bardon Kaldian

    然而,是他提出了广义相对论,而不是比他更有数学天赋的大卫·希尔伯特(David Hilbert),后者可能是 20 世纪最伟大的数学家。 确实,希尔伯特在与爱因斯坦讨论后,几乎立即得出了足够现实/公平的方程,足以承认爱因斯坦的首要地位。

    爱因斯坦想要表演他在狭义相对论中所做的同样的伎俩,他颠倒了洛伦兹的推理,并从洛伦兹变换中得出了一个假设,即光速不变,再加上一个简单的假设,即物理定律在惯性系统中得到维持。 他于 1905 年出版了这篇文章,没有作为单一参考文献,大概声称他不知道洛伦兹,更不知道庞加莱工作。 直到第一次世界大战后,当 1905 年论文的英文译本出版时,洛伦兹才被承认,但没有被认为是灵感来源,这意味着爱因斯坦仍然声称他不知道洛伦兹的工作。 诀窍在于,通过假设光速不变,以太的问题就变得多余了。

    他想利用引力完成类似的壮举,并使其变得多余,这样就不会再有人问起它的性质和远距离的怪异作用。 他利用等价原理将问题转化为空间的几何问题。 他无法用数学方法解决这个问题。 因此,马塞尔·格罗斯曼(Marcel Grossman)前来救援并完成了大部分数学计算,但随后他们陷入了最终方程的困境,他们无法制定或推导出这个方程。 于是有人联系了希尔伯特,他确实解决了这个问题并决定发表。 他向爱因斯坦展示了一些结果,爱因斯坦决定在不承认希尔伯特的情况下发表它。 然后历史学家开始创造伟大爱因斯坦的神话,并决定将希尔伯特从历史中抹去,就像他们抹去洛伦兹和庞加莱一样。 一些手稿被销毁,一些手稿被更改。

    http://www.znaturforsch.com/aa/v59a/s59a0715.pdf
    希尔伯特论文的印刷校样的一个关键部分被人剪掉了,这一事实在科里、雷恩和斯塔切尔的论文中没有提及,科里、雷恩和斯塔切尔得出的结论是站不住脚的,没有证明价值。 我宁愿表明,证据的被切断部分表明某些不知名的人试图伪造历史记录。

    奥威尔的“控制过去的人就控制了未来”。 谁控制了现在,谁就控制了过去。” 用在这里非常贴切。 那些控制现在的人正在重述历史叙述,以便通过改变过去来控制未来。

    就像大英帝国发现从怪人艾萨克·牛顿中构建一个终极天才的神话非常有用一样,20世纪初的犹太人也从熟练且不择手段的抄袭者阿尔伯特·爱因斯坦中创造了一个终极的难以理解的天才。

  199. @Truth

    我的中文名字与英文同音,所以没有什么真正的原因。 我确实有一个相当讨人喜欢的绰号,是我帮忙的一个黑人给我起的,这很有趣; 他将我的姓氏与一个敬语押韵,似乎很惊讶以前从未有人想到过这一点。 从“智商”的意义上来说,黑人的声音似乎确实非常圆滑。

    • 回复: @Truth
  200. jjj 说:

    软件编程的语言加载与视觉空间加载有何不同?

    @严申

  201. Factorize 说:

    智商的基因增强可能已经在中国发生。
    (据我了解,这在美国是非法的,或者至少在美国是不可接受的。)

    东亚人的 M/V 倾斜,更具体地说,东亚男性的 M/V 倾斜可能会随着智商的提高而变得更加倾斜。 这对东亚社会的影响现在应该引起广泛的研究兴趣。 想象最大化数学智力可能会导致明显的自闭症行为,并可能导致对繁殖完全缺乏兴趣,这似乎并不牵强。 随着人类智商估计的理论最大值接近 1500 左右,这种风险可能会大大增加。

    • 哈哈: Daniel Chieh
    • 巨魔: utu
  202. Joe Webb 说:

    我不明白你的阿纳托利烤肉串密码

    也许那是因为你们的英语和汉语想要诗歌,或者英语中所谓的感性。

    有趣的是,我今天去看医生,中国医生说,哦,你很容易说话……当谈到你们自己的女人时,白人远远领先于你们中国男人。 这如何让你感觉……更加极权主义?

    如何解释中国的东方专制主义等永远不会改变的现象?

    不如直接回答,而不是歪曲中国话。 另外,与字母表相比,您的图画行话是如此落后。 这可能有助于解释你们的种族在数字以外的思维方面的缺陷。

    我们很快就会把你赶出去。

    乔·韦伯

    • 巨魔: utu
    • 回复: @Daniel Chieh
  203. @Factorize

    我认为你不应该从模因中学习神经科学。

    • 回复: @Factorize
  204. @Joe Webb

    感谢您对 Unz 幽默的贡献。

    • 回复: @TT
  205. @Factorize

    无论如何,值得进一步讨论:

    1)目前,我们知道一些基因组合起来可能会对智商产生一些影响。

    2)我们还没有达到能够对这些东西进行 CRISPR 改造的程度。

    3)“智力”很复杂,虽然智商是一个不错的广泛指标,但它无法解释许多影响最终表现的问题。

    一个很好的例子是,执行功能问题在智商测试中没有得到很好的考虑——早些时候,有人提到过一位女性,尽管她在其他方面很聪明,但在定时压力下的考试却让她感到压力重重。 我们可能认为她大脑中处理信息的部分结构良好并且能够产生结果,然而,涉及压力的大脑边缘部分压倒了她调动相关神经元产生考试结果的能力。

    另一个例子是被定义为患有多动症的人——他们可能拥有执行任务所需的所有心理部分,但无法很好地集中注意力。 另一方面,尽管工作记忆较低,但他们似乎具有轻微的创造力优势。 自闭症本身似乎涉及大脑“系统化部分”的压力,可能是通过破坏连通性并使其进一步脱离大脑的社交部分。 它与性兴趣无关,尽管参与或识别社交线索的能力下降显然会对正常生活(包括浪漫追求)产生负面影响。

    应该指出的是,尽管它们可能被视为“障碍”,但它们都是正常功能的一部分,在某些情况下可能是有利的。 您不能简单地升级某些东西并期望更高的性能;您必须这样做。 神经化学物质增加/泛滥是许多严重精神障碍和外来药物使用的一个特征。 一般来说,它不会提高整体表现:服用 LSD 的人可能在某种意义上更具创造力,但不能将其视为功能上的全面、一致的改善。

    按照您的建议,我们距离“改善大脑”还有很长的路要走。

    • 回复: @Factorize
  206. Talha 说:
    @Truth

    同意——它几乎就像漫画书中的秘密身份戒指一样带有头韵。

    白天,他是 Leroy Li——谦逊的高智商遗传学家。

    晚上他就是“小熊猫”!

  207. Factorize 说:
    @Daniel Chieh

    感谢您的回覆。

    我对你从亚洲角度对此的看法感兴趣。
    我意识到这是一个很难谈论的话题,尽管它很重要,值得诚实地尝试。

    M/V 倾斜也适用于男性和女性,尤其是在成就最高的时候。
    即使在 0.01% 的能力水平上,倾斜之间的性别差异也非常大。 0.01%仅为3SD;
    预计 IQ 为 100 SD。

    考虑到研究证据,提高智商会合理地带来男性和女性基本上无法相互沟通的风险。 如果这些问题现在很困难,那么将来可能会在某个时刻变得不可能。

    智力的遗传增强显然是可能的。 美国公司特别指出,他们认为这种增强不符合社会规范。

    在硅谷,高智商的人繁衍后代所带来的自闭症风险是显而易见的。 它被描述为一种流行病。 这只是通过选择和随机重组产生的。 关于高智商人群所涉及的生殖风险的诚实对话
    尤其是那些有数学倾向的人似乎完全合适。 可悲的是,正是沟通能力的缺乏才导致了这个问题的不断发展。

    据报道,中国已接受大规模PGD。 在这些风险变得太明显之前考虑一下可能涉及哪些风险似乎是非常明智的。

    • 回复: @Yan Shen
  208. Yan Shen 说:
    @Factorize

    我们是否可以提高现有个体的智商,而不仅仅是编辑潜在未来人类的基因组? 如果是这样,我很乐意看到智商 1500 增强的塔内西斯·科茨 (Ta Nehisi Coates) 与智商 1500 增强的约翰·德比郡 (John Derbyshire) 正面交锋。 这就像看到现实生活中的两辆终结者 T-3000 为争夺霸主地位而决斗……

  209. Yan Shen 说:
    @Factorize

    老实说,中国大陆有1.38亿多人口,我认为长期来看,人口减少几亿也不算太糟糕……

    • 回复: @The Scalpel
  210. Factorize 说:
    @Daniel Chieh

    目前最先进的 IQ GWAS 揭示了人类智力 10% 的差异。
    智商被认为是 100 SD。
    10% 是 10 SD 或 150 IQ 点。

    大多数 IQ 的 SNP 相当常见,因此简单地选择胚胎(而不是 CRISPR)就会导致 IQ 的大幅提高。

    最近的 GWAS 发现自闭症与智商呈正相关。
    我发现有趣的是,即使在智力曲线的顶端(没有基因工程),类似自闭症的特征也开始出现。 如果这就是 +5-7 SD 时发生的情况,那么有理由怀疑在 +10-20 SD 或更高时会发生什么。

    值得注意的是,天才精子库显然不得不关闭,因为当人们实际上有机会生育具有天才基因的孩子时,他们认为所涉及的风险(例如自闭症)超过了好处。 这可能就是我们的未来。 人们可能会再次选择增强智商以外的其他功能。

  211. WhiteWolf 说:
    @Truth

    特斯拉为我们带来了交流电,而爱迪生则在推动直流电,而直流电在长距离传输时效率要低得多。 特斯拉用他的天才做了很多实际的事情,值得比他更多的认可。

    • 回复: @jilles dykstra
  212. TT 说:
    @Jeff77450

    如果中国不这样做,以加速其崛起和繁荣,那将是愚蠢的。 当发展中国家在美国的压力下严厉打击版权时,它付出了巨大的代价,阻碍了进步。 我的政府使IT进步和业务现代化如此严格的版权执法成本很高。 是什么促使企业以$ 10的销售价格和$ 5的盗版软件价格进行升级?

    西方提出了一个名为专利和知识产权的系统,他们详细公布了他们的调查结果,以确保他们有最好的理由起诉任何侵犯其主导领域的人。

    其他人则通过这些巨大的宝库进行筛选,并从那里进行改进。 日本,韩国和美国的盟友可能在美国的某些祝福下迅速崛起。

    我怀疑美国不滥用这个制度在道德上更善良或更诚实,而是主张只保护自己的利益,因为它有更多的损失,进一步以它的 5eyes 7/24 监视每个人。

    曾经有一篇文章告诉法国,美国总是在法国发明发布之前几天就发表文章,这太巧合了,所以他们决定每种高科技只能通过硬拷贝进行交流,并且行之有效。

    中国只是开始发表他们认为对他们有商业利益的论文,因为他们正在深入全球化网络。 但是我相信许多顶级技术都不会发布,尤其是在军事上适用的一种技术,直到条件对它们有利为止。 他们还生产许多具有版权失效的优质便宜产品。

    小国只是松散。 闪存是由一家名为 Trek2000 的小公司开发的。 直到今天,每个人都公然抄袭。 在过去的 2 年里,Trek 一直在到处起诉,并认为它对巨头是徒劳的,所以所有的新发明都卖给了东芝。 他们是所有东芝闪存的 OEM。

    20年前,当闪存驱动器只有16〜32MB时,两名年轻的中国工程师试图说服我们支持他们的2GB闪存发明,这将彻底改变商业HDD(最大的是Seagate 2GB?)和PC存储(juz插件可在任何地方运行任何虚拟PC)闪存中的所有软件)。 后来,大闪存mem开始推出,今天是2GB,但我仍然看不到这两个家伙的公司或名称。

    Creative 还到处起诉其 Sound Blaster 声卡、MP3 IP……苹果最终同意以 100 亿美元进行庭外和解,条件是用于起诉其他公司。 另一款产品在类似的 iPad 和 Android 平板电脑出现之前十年就已经在 WinCE 上运行了用于物流和医疗用途的开发 WiFi 触摸屏平板电脑。 这些只是冰山一角。

    大吃小,丛林法则。 所以不需要假装占据道德制高点,很快西方就会开始复制中国FSR铁路(一些中国专利技术令他们沮丧)、量子、超级计算机、电动汽车、Pebble核反应堆、人工智能等等。

    据我了解,西方在中国的外国直接投资一直微不足道,特别是美国数十年来一直故意劝阻外国直接投资以阻止其债务扩张宣传的中国升值,同时错误地大肆宣传印度的潜力与中国背道而驰。 香港,新加坡,台湾,亚洲是外国直接投资始终保持领先的国家。 更重要的是这些亚洲国家的技术转让,尤其是孢子是中国获得公认技能的圣地。

    2017年52.57-3.26月对华投资排名前十位的国家和地区(按实际投入外资计算)如下:香港(2.81美元)、台湾省(1.84美元)、新加坡( USD1.78b)、日本(USD1.75b)、美国(USD1.36b)、韩国(USD1.24b)、荷兰(USD890b)、德国(USD610b)、英国(USD68.1m)和丹麦(USD94.4m),其中6.7b美元,占全国实际使用外资总额的XNUMX%,同比下降XNUMX%。

  213. TT 说:
    @Daniel Chieh

    我总是对你对愚蠢的巨魔永远简短而幽默的回复感到好笑。

    • 回复: @Daniel Chieh
  214. 未来最重要的问题是先进社会中高等数学/语言类型的繁殖率。 在原始社会,这个数字很高,但随着社会的进步,这个数字就会下降。 日本就是一个例子。

    至于奖励高等数学/语言类型,为什么? 作为人类,我们为什么应该鼓励更先进的武器、更多的智能手机、更多的人工智能?

  215. @Factorize

    大多数 IQ 的 SNP 相当常见,因此简单地选择胚胎(而不是 CRISPR)就会导致 IQ 的大幅提高。

    尝试了 IIRC 胚胎选择,但由于某种原因没有导致显着增加; 无论如何,胚胎选择还不够普遍,不足以造成如此大的人口差异——它只在人口的某个社会经济部分的范围内,而“自然”受孕则更为普遍和有趣。

    自闭症似乎与大脑部分与另一部分之间的交流减少有关,与其他交通的“隔离”表明智力的提高。 然而,这在改进方面是有限的,你只能让信息变得如此清晰,而过多地减少意义的“语义网络”可能是破坏任何发散思维/创造力的绝佳方法。 由此看来,大脑的连通性在一定程度上是创造力发挥作用的方式——信息存储在大脑的许多位置,洞察力通常是通过松散的关联而不是沿着髓鞘良好的神经元传播而发现的(这可能就是 LSD 提高创造力的方式,通过放松对逻辑和抑制的通常检查)。

    我认为人类的大脑根本没有得到优化,但我也不认为我们目前的理解太差而无法利用它做太多事情,至少不能用出生后的外源因素做很多事情。

    • 回复: @Factorize
  216. @TT

    我玩了太多英雄联盟,再也不会被巨魔困扰了。 我不知道你是否熟悉这个游戏,但中国还有一个关于它的笑话:

    “一开始你有五个敌人。 尽量不要在玩游戏时增加这个数字。”

    我认为这也是《王者荣耀》游戏玩法的基础,但我对那个游戏不太熟悉。

  217. 为什么你认为人口下降会就此停止,特别是如果生孩子并不能“加起来”?

  218. @Yan Shen

    之前的评论是针对严沉的。 对困惑感到抱歉。

    “为什么你认为人口下降会就此停止,特别是如果生孩子并不能“加起来”?

  219. Anonymous [又名“先生”] 说:

    我完全不相信这种“分裂”的胡言乱语。 我为沉先生写了这么多文字试图理解这种非理论感到遗憾。 我给你一个词:学校。 在东亚国家,他们将数学视为学校的一门严肃的学科。 这不是选修课。 学生必须学好数学才能进步。 很多练习可以练习你的数学能力。 还有很多考试。 而在美国,人们已经享受了太久的美好时光,所以老师和学生都只选择容易让他们兴奋的科目,功利性见鬼去吧。 所以你会看到他们谈论恐龙、木乃伊、探险、法庭辩论等等。数学很无聊,除非有好老师教,而且需要付出很多努力。 然后它甚至不需要你的学术进步。 作为一个非白人,我观察到现代数学、计算机科学、密码学、形式语言等都是欧洲/美国白人发明的。 那么你怎么能说他们数学不好呢?

    • 回复: @Da_Han
  220. @WhiteWolf

    特斯拉确实发明了可以用交流电运行的电动发动机。
    这为长距离、高效率、高张力的电力输送带来了巨大的突破。

  221. denk 说:
    @Stan d Mute

    “我们压制例外论”

    布哈哈哈哈

    蜜糖,
    别让我的脚趾笑!

    福库斯 定义 例外论,

    从来没有听说过'福库斯例外论'呃
    介意进行搜索吗?

    福斯咒语
    “照我说的做,而不是我做的”

    ‘屠杀五千万自己的人民? '

    引文?

    “他们玩游戏、吸鸦片”

    当你为自己那段历史感到自豪时
    在枪口下种族灭绝了一半的中国人,呃,
    真是个迷人的家伙!

    呵呵呵

    • 巨魔: Stan d Mute
    • 回复: @Stan d Mute
  222. Hacienda 说:
    @Daniel Chieh

    尼斯贝特博士的研究表明,日本观众如何倾向于将“兔子”与“草”、“鸡”与“种子”联系起来,而他的美国小组则将“兔子”与“鸡”、“草”与“种子”联系起来。

    对我来说,很明显,兔子和鸡是相配的。 不太清楚草籽是在一起的。
    所以我知道日本人从哪里来。 我不知道尼斯贝特博士采取的更精彩的做法。 我必须阅读这本书并了解有关日本 K-12 教育体系的更多信息。

  223. Joe Webb 说:

    尝试中国和极权主义。 尝试言论自由的白人。 尝试用 ju-ju 来非洲人。

    尝试犹太人的统治。

    在硅谷,用谎言、欺骗、偷窃、金钱来尝试中国人和亚裔印度人。 这是我从其他自由派白人那里听到的。

    试试白人,带着救世主义……我们的高利他主义商数对于个人生存和种族生存来说都是病态的。

    尝试将中国人和其他所有种族视为完全达尔文主义(第一组),犹太人在这方面占据优势,并且是阿拉伯人的表兄弟......更多的是相同的部落主义......闪族人。

    整个世界,从种族上来说,除了白人,都是部落的。 你赢了,我们白人输了。

    就那么简单。 然而我们白人正在觉醒。 正如几周前《美国文艺复兴》报道的那样,即使是我们的孩子,他们中也有大约一半(年龄?)感觉到移民正在伤害他们。

    新黄祸将会流行,可能是在黑色和棕色之后最后流行,因此你们中国人等有足够的时间来安排回国的时间。 钱袋可以随身携带,至少现在是这样。

    哦,还有你的中国ju-ju,叫做风水,或者其他什么……神奇的思维……也带着它,别忘了你的孩子。

    乔·韦伯

    • 回复: @anon
  224. Joe Webb 说:

    只是在上面失去了更多。

    所以。 几年前,帕洛阿尔托的华人努力游说学校(白人纳税人)为他们的孩子支付中文沉浸式(!)行话的费用。

    现在这就是犹太人所做的事情……想象一下,犹太人要求白人在学校支付希伯来语费用。 事实上犹太人不这样做,但这只是因为希伯来语只适用于俱乐部成员。 “肆无忌惮”的中文单词是什么?

    或多或少这只是贪婪/金钱,而不是文化等断言。

  225. Da_Han 说:
    @Anonymous

    “巴比伦人发明了XY和Z。那你怎么能说他们不擅长XY和Z呢?”

    亚洲人可以做得更好。

  226. @denk

    引文?

    你是个愚钝的人吧? 好吧,好吧,请称呼我为伟大的老师。 我会称你为可悲的巨魔。

    https://www.independent.co.uk/arts-entertainment/books/news/maos-great-leap-forward-killed-45-million-in-four-years-2081630.html

    一般接受的范围是 37 万到 65 万之间。 微不足道的日本重创了约 300,000 万。 第一次鸦片战争中,20,000万英国人和印度人屠杀了约200,000万中国人,但仅伤亡69人。 第二次鸦片战争见证了 20,000 名英法士兵屠杀了另外 200,000 名中国人。

    自卑感有多大? 伟大的老师完成后,现在你在我的“忽略”名单上。

    • 巨魔: TT
    • 回复: @TT
  227. Dmitry 说:
    @myself

    如果不带偏见地阅读,古希腊思想对我们来说也是非常陌生的——也许与印度和中国的古代传统一样。 古希腊思想的异域陌生性,是尼采在其著作中经常强调的事情之一。

  228. denk 说:

    那个 顽固的白化石 连我的评论都不敢刊登的媒体, 索赔 ...... ..
    你管这叫证据吗,孩子?

    我还教过一些像你这样的白痴 日报101,可惜你太厚了,学不到什么东西……

    现在注意了,这就是我所说的 吸烟枪 !

    除了对中国的鸦片种族灭绝之外,
    我在此指控福库斯于20年在印度尼西亚策划了1965世纪最严重的种族灭绝事件之一,杀害了约3000000万印度农民和华人。

    我怎么知道呢?

    从马嘴里说出来的!

    外交部回答说:“我们当然不排除任何无法归因的宣传或心理战活动,这些活动将永久削弱印共。 因此,我们同意[上述]建议…… 合适的宣传主题可能是……中国对特定武器运输的干预; 印度尼西亚共产党作为外国共产党的代理人颠覆印度尼西亚”。 它继续说道:“我们希望在印度尼西亚人仍处于失衡状态时迅速采取行动,但治疗需要微妙......

    请告诉我们您在这方面可能有的任何建议,我们可以在这方面提供帮助”。

    http://markcurtis.info/2007/02/12/the-slaughters-in-indonesia-1965/

    所以你现在可以自豪地庆幸“我们 重击 1965 年又出现了数百万中国人,这就是事实。

    呵呵呵呵

  229. Factorize 说:
    @Daniel Chieh

    感谢您的回应。
    我很想听听您对近期智力增强前景的印象。 不是

    我的猜测是,这种增强在有限的意义上已经是可行的。 计算表明,1-2M规模的GWAS将解锁整个IQome。 对于这些变异,这只是胚胎选择的问题,胚胎选择已在临床使用了近 50 年。

    如果每条染色体有 1000 个变异,并且染色体链之间有 50 个好的变异差异,那么在为整个集合选择最高的 PGS 时将存在 10 个 SD 差异。 这将需要大量的选择,尽管显然十分之一的选择仍然会给出 1 SD。 人们可能需要一段时间才能充分认识到通过简单的选择可以产生多大的增强效果。

    这显然将对人类产生巨大影响。 有些人可能只是想暂时解决这个问题。 我们可能只能假装未来一段时间内还没有到来。

    我发现值得注意的是,消费者基因组学服务已经推出,提供多基因性状选择。 然而,他们明确排除了智力增强。 给出的理由是这种增强在政治上不被认为是可接受的。

    身高现在可能是迄今为止得到最好表征的多基因表型,其遗传学在许多方面被认为与智商相似。 现在看来,多基因选择完全有可能使选定胚胎的成年身高增加 2-3 个标准差或大约一英尺。

    • 回复: @Daniel Chieh
  230. @Factorize

    我很怀疑。 数百年来,我们一直拥有培育身高的方法,但我们从来没有像对待动物那样有意识地选择人类。 是的,即使在中国,所涉及的成本仍然令人望而却步。 假设某个不道德的亿万富翁选择以这种方式选择他的孩子,我认为这不会是一个令人惊叹的规模,而且之前培养天才的努力并没有取得明显的效果,例如:

    https://en.wikipedia.org/wiki/William_James_Sidis

  231. Factorize 说:

    我同意将他人客观化为配子供应商并不会给人一种“他们从此过着幸福快乐的生活”结局的感觉。 多基因选择的重要部分是,优生理想的婚姻将不再适用。 任何一对都可以使用现有的常见变体并设计所需的极端表型。 就成本而言,我怀疑它会像任何技术一样,当大众市场发展时,规模经济将压低价格,低于想象。 如果染色体选择变得可用,那么可能就像从每个亲本中选择最好的 23 条重组染色体一样简单。 我想如果需要的话,他们也可能拥有一般最佳的染色体,或者有人可能想成为第三个遗传父母。

    商业和金融潜力似乎是巨大的。 这可能是21世纪的产业。 当然,智商的提高可能会持续几个世纪。

    我在生殖方面看到的不道德行为是,人们随机地滚动基因轮盘赌,当它不起作用时,他们就会责怪孩子。 这一定是人生最大的悲剧之一:爸爸妈妈不爱你,因为你没有触犯红线 7. 一个人们可以控制孩子表型的世界一定比一个父母有爱的世界更好对于他们的孩子来说,这是以随机事件为条件的。

    应该记住,直到几个月前结果发表之前,智商的有效遗传增强是不可能的。 智商与格林威治平均值(100)之间存在的微小差异是由几个世纪甚至数千年的选择压力造成的。 单代有效的胚胎选择将导致智商大大超过目前认为的高智商。

    • 回复: @Daniel Chieh
    , @res
  232. @Factorize

    我只是不认为它像你希望相信的那么容易 - 我不认为你可以只选择你想要的大脑特征并得到“更聪明的人”。 你可以选择特定的等位基因,例如 BDNF 的更大表达,但它必须与其他等位基因结合使用,我不知道我们是否接近能够预测结果表型的点:得到“这会让我们的人整体上更聪明”,而不是“这让我们的人患有精神疾病,但是嘿,他对各种金属的气味确实有很好的工作记忆,这也是他唯一真正的东西”兴趣:嗅金属。 ”

    你不能轻易地对人类进行实验。 在某种程度上,中国的优势在于他们更加渴望使用接近人类的模型,例如黑猩猩,但归根结底,人类的大脑非常独特。

    我想我们会看到的。

    • 回复: @Yan Shen
    , @Factorize
  233. anon • 免责声明 说:
    @Joe Webb

    尝试白人的种族灭绝和对无数土著种族和文化的全球破坏。

    有趣的是,像你这样的另类右翼似乎只能从积极的角度看待白人,甚至试图声称白人是受害者,而对过去几个世纪普遍存在的白人背信弃义却视而不见。

    请省去那些令人厌烦的借口,例如:

    “bb-但是白人只是想帮助并c-教化这些土著野蛮人!”

    对于任何有头脑的人来说,几个世纪以来白人对世界其他地区的行为都带有剥削和恶意的意图。 只有像你这样的另类右翼人士才会真正相信白人是神圣而完美的人,从未做过任何错误或不公正的事情。 事实上,自那时以来几乎没有什么变化,西方国家仍然忙于破坏和剥削世界其他地区,总体上造成混乱。 也不要为此责怪犹太人。 我与许多白人保守派交谈过,他们一致认为他们有权以他们认为合适的方式与世界打交道,无论这个星球上其他人的愿望或考虑如何。 在我看来,尽管殖民主义早已不复存在,但今天的白人仍然存在这种傲慢、专横和剥削/机会主义的倾向。

    此外,我还注意到,认同另类右翼的白人通常往往是白人种族中最糟糕的典范。 我遇到过许多富有同情心、真诚和善良的白人,但他们中没有一个人认同另类右翼分子喜欢宣扬的这种短视、幼稚、美化、肚脐眼、完美的白人形象,尽管这个形象完全不一致与遥远的过去白人的真实且广泛记录的行为有关。 白人并不完美,和其他人一样,他们也会做出一些非常可怕的事情。 说真的,抓紧伙计

    • 回复: @Joe Webb
  234. Yan Shen 说:
    @Daniel Chieh

    https://www.usatoday.com/story/tech/science/2018/01/24/cloned-monkeys-scientists-china-first/1062649001/

    中国科学家周三宣布,他们成功克隆了两只长尾猕猴,这一壮举引发了人们对人类克隆可能有多接近的质疑。

    长期以来,人们一直认为灵长类动物的克隆从根本上比马、羊和其他哺乳动物更困难,因此距离更远。

    “这是一个重大进步。 以前没有人能够克隆出非人类灵长类动物,”旧金山加州大学干细胞中心主任阿诺德·克里格斯坦(Arnold Kriegstein)说。

    • 回复: @Bukephalos
    , @Daniel Chieh
  235. @Factorize

    天才精子库显然不得不关闭,因为……他们认为所涉及的风险(例如自闭症)超过了好处。

    没有

    天才的精子库不再存在。 著名的Germinal Choice信息库于1999年关闭
    ...
    阿道夫·希特勒(Adolph Hitler)…种族纯洁…雅利安人种…纳粹…雅利安人…纯血统…理想的雅利安人…种族主义者…纳粹德国

    这就是为什么。

    • 回复: @Factorize
  236. 威廉·彭伯·里维斯(William Pember Reeves),《长长的白云,AO TEA ROA》,1898年,1924年,1998年,米德尔塞克斯(Middlesex)
    菲·曼宁(FE Maning),《旧新西兰》,1887年,1980年,奥克兰

    有趣的书,早在白人到来之前,毛利人就有能力互相消灭。
    他们并不是唯一的人。
    在非洲,同样的故事。
    印加帝国建立了大约一百年,有人想象这是一次和平的运作吗?
    与白人的区别只是我们拥有更先进的武器和船只。
    并让人们描述我们所做的事情。
    西班牙有一篇关于玛雅文化被摧毁之前的描述。

    • 回复: @anon
  237. TT 说:
    @Stan d Mute

    现在我非常确定你不是你声称的那样的巨魔,而是一个白痴的仇恨中国的煽动巨魔。

    巨魔总是使用有趣的手柄,没错。

    这里有 1,160 条评论和 170,000 多个字。 几乎所有内容在语法上都是正确的

    但只是废话。 更加努力。

    中国人发明了你们鄙视的烟花火药,但现在他们吸取了鸦片战争的教训,所以它将用在高超音速核武器导弹DF上来撞击侵略者的屁股。 俄罗斯斯基也了解到,美国北约是一群只懂得一种语言的说谎豺狼,即暴力。

    • 回复: @denk
  238. anon • 免责声明 说:
    @jilles dykstra

    早在大规模移民和多元文化主义因全球化而根深蒂固之前,白人就有能力相互取代和毁灭(具有讽刺意味的是,这一过程是在西方殖民主义和西方技术(即飞机和互联网)的框架下有机实现的),但现在的不同之处在于,西方由于更好的技术和全球支持全球化的情绪(有利于移民友好政策的实现)以及西方国家内部强大的支持移民派系的影响,各国正在涌入大量移民。

    那么你的意思又是什么?

    • 回复: @Joe Webb
  239. bjondo 说:
    @Bukephalos

    我想我读到中国将使用中央情报局和以色列对俘虏和其他俘虏和使用的人进行实验。 这两个人的实验比中国人愿意做的任何事情都要多和疯狂。

  240. @Dr. Doom

    我服用利他林是为了阅读整本书而不是指定的章节。 我只能假设这是因为老师没有完成书并且不想让我破坏它。 它没有用,我把药片装进口袋,并在今年余下的时间和明年阅读所有内容。 鼓励女孩们阅读,而我则因此而被吸毒。

    • 回复: @res
    , @ThirdWorldSteveReader
  241. denk 说:
    @TT

    罗恩经营一个开放论坛,我全力支持。
    这是我们仍然可以发泄的少数几个网站之一,
    但也有一个缺点,就是会招来很多苍蝇。

    我通常遵循黄金法则……
    不要喂巨魔!

    但是,当它变得过于大胆时,我们需要狠狠地拍打它,以保持空气新鲜!

  242. @Yan Shen

    是的,这令人印象深刻。 前额叶皮层不会给我们太多的帮助,但仍然相当不错。

    我确实认为,随着进展的进行,它将来自这样的步骤,这些步骤看起来相当离题,但可以作为执行学习过程的必要构建块。 中国科学本身也需要改进——有很多质量差的研究,控制不够好,仓促的研究等等。我确实认为它在改进,但那里的很多学术文化需要改变,例如发表或灭亡,走向极端。

  243. @Bukephalos

    进行不道德的实验以从监狱对象中产生超人听起来像是在当前现实模拟中体验 V 字仇杀队的好方法。

    • 回复: @Bukephalos
  244. res 说:
    @Factorize

    多基因选择的重要部分是,优生理想的婚姻将不再适用。 任何一对都可以使用现有的常见变体并设计所需的极端表型。

    我不认为这对于选择来说是正确的。 以身高为例。 如果父母处于+2 SD,则起始胚胎分布平均值应约为+1.5SD(由于向平均值回归),并且与处于平均值的父母相比,通过等效功效选择(例如1中的10)实现的可能结果应为大致保持这一优势。 我怀疑如果选择多个特征,优势可能会更大。

    实际的基因工程当然是不同的。 但即使如此,我认为减少所需的编辑数量也可能有好处。

    • 回复: @Factorize
  245. res 说:
    @CraigAustin

    鼓励女孩们阅读,而我则因此而被吸毒。

    欢迎来到 Harrison Bergeron 的世界。

  246. Joe Webb 说:
    @anon

    你的正义行为在这里进行。

    我从来没有像犹太人经常声称的那样声称白人是神圣的人

    我们已经杀死了很多其他人,正如其他人所做的那样。

    我们占领美洲是因为从根本上来说,美洲印第安人是石器时代的人。 (他们应该感谢他们的肮脏之神,因为中国佬没有在我们之前到达这里……他们都会消失,等等。)

    我们把他们送到学校等等。没有任何作用,因为他们今天的智商是83,而且他们的性情非常不稳定和暴力,就像黑人一样。

    我们征服了,就像其他民族/种族征服了其他人一样。

    您说您曾与认为白人有权控制世界的“另类右派”交谈过,等等。我认识其中许多人,我们的总体立场是每个种族都有权留在自己的祖国。 但不是我们的。

    你还说白人认为他们可以控制世界。 你一定是在和犹太人说话,而不是白人。

    最后你说另类右派没有同情心,等等。你的“好心”白人是自由主义者。 他们向那些屎人敞开了大门(由犹太人领导,他们一心要把白人变成自己国家里绝望的少数派),不仅摧毁了我们的种族士气,而且摧毁了对聪明和人道文明的任何希望……意思是意识到事情比如言论自由、法治、隐私等等,更不用说像登月这样的高智商了,而是高敏感性,这使得令人愉快的人际关系成为可能,尤其是两性之间的关系。

    白人发明了浪漫的爱情。 对于其他人来说,按照我的中国女孩的故事,或者黑人的阴沟性行为,这只是“生意”。

    白人是最好的全能种族。 我们确实发明了文明。

    至于我们的统治倾向……被像你这样的娘们所哀叹……这就是为什么你在这里,我的朋友。 你站在镜子里的自己面前……如果你有真正的大脑,你就会意识到你在这里是因为你的祖父等杀死了敌人,无论是个人的还是政治的。

    如果你想消灭白人,就从自杀开始吧,你这个傻瓜。 抓紧一点,种族叛徒……你们孩子们在哪里?……我猜是零。

    我刚刚看到报道说奥普拉正在等待上帝给她一个竞选总统的信号。
    阿弗雷卡!

    或者选择阿兹特兰! 或者拿中国来说! 穆斯林! 请。 一切专制。 顺便说一句,美洲印第安人来自亚洲,表现出相同的基因……集体主义等。

    白人是唯一个人主义的种族。 太过分了。 我们需要部落化来驱逐底层生活,并将像你这样的自由主义者关进监狱,直到你清醒并……承诺生孩子。

    有没有注意到所有的极限运动都是白人主导的? 冒险、勇气、权力意志。
    我们正在失去像你这样的傻瓜。 (我曾经有点像你……左派。但我很聪明,勇敢……这意味着我有勇气改变!正如前总统所说。)

    内战即将到来,因为很多白人都有这种“救世主”情结。 拯救智人的每一条垃圾DNA,无论客观上多么非人类。

    加州刚刚花了 18 万美元收治了一名患有某种奇怪疾病的墨西哥非法儿童。 除了你之外,这就是杀死我们的原因。

    乔·韦伯

    • 回复: @GammaRay
  247. Joe Webb 说:
    @anon

    顺便说一句,白人自由派/犹太颠覆者正在用卡车运载非法移民去投票。 这一点甚至在福克斯新闻频道上也被直白地指出了。

    这是自杀行为。 自由主义者变成了共产主义者……为共产主义拉皮条的小姐妹。 早在托洛茨基主义“共产主义世界主义者”时代,这就是字面上的“世界革命”。

    斯泰林的《一国社会主义》终结了这一点。

    今天,又是世界革命。 旧的革命现在! 1789 年和人类救世主。

    从基督教的意义上来说,拯救任何人是不可能的。 这是一个达尔文主义的世界。 但这包括部落主义,所以你的团体可以被拯救。 也就是说,如果你认同它并为之奋斗,就像犹太人等以及所有其他种族群体所做的那样,除了被病态利他主义诅咒的白人之外,它就可以被拯救。

    这一切都始于斯多葛主义,它融入了基督教等。

    移民是种族自杀。 自由主义者无法忍受这个世界……达尔文主义……对他们来说,世界变成了全有或全无。 一切都是救世主义,没有什么是死亡。 也许这就是他们想要的……让我帮助他们……自杀吧,别再用你们的虚无主义来殴打我们其他人了。

    乔·韦伯

  248. Factorize 说:
    @Hippopotamusdrome

    非常感谢您的链接和更正。

    我自己也不完全确定关闭天才精子库的真正原因是什么。 有时真相可能是模糊的。

    我仍然觉得奇怪为什么天才精子库会关闭。 您链接的文章接着讨论了当前的精子库如何竭尽全力描述其捐赠者的表型。 基因选择不就是遗传选择吗? 选择长得好看、运动能力强的人就可以了,但不要太聪明?

    这些诊所清楚地表明了对本次论坛上提到的增强功能的需求。 我认为,鉴于目前对智商遗传学的了解,文章中提到的增强效果不大应该不足为奇。

    此外,由于这是一篇最近的文章,我很惊讶在讨论供体的表型分析时没有提到完整的基因组序列。 在使用选定的配子供体和胚胎选择的第二代遗传选择中,看到智商的大幅提高也就不足为奇了。

  249. Factorize 说:

    中国是否做过与西方国家类似的心理测量研究? 是否有随机抽取的中国人参加过智商测试? 或者我们是否真的接受种族间智商差异是未经科学证据证实的说法的可能性?

    • 回复: @Talha
    , @res
  250. Talha 说:
    @Factorize

    这是一个好主意。 我其实很想看看,在没有西方文化影响的情况下,中国人会想出什么样的智商测试来衡量智力。 也许他们现在想出一些可能完全不同的东西已经太晚了,因为这种影响已经无处不在,但我想知道他们在与西方接触之前过去测试过什么。

    和平:

    • 回复: @myself
  251. Bukephalos 说:
    @Daniel Chieh

    如果不是令人反感的同性恋宣传,我几乎会发现这部电影很有趣。 但是沃卓斯基姐妹不得不把他们的“信息”塞进某个地方,我猜

    • 回复: @Daniel Chieh
  252. res 说:
    @Factorize

    有关一些中国智商研究论文的列表,请下载 David Becker 的电子表格,其中提到 https://www.unz.com/jthompson/the-worlds-iq-86/ 并查看收集表。

    • 回复: @Factorize
  253. GammaRay 说:
    @Joe Webb

    非常可悲的是,你花了这么多时间写这个词“沙拉”,只是为了回避我提出的非常简单的反驳观点。 很明显,你缺乏诚实地承认我的观点的正直,反而试图粉饰任何有思想的人(包括大量白人)理所当然谴责的事情。 对不起,伙计,但无论你如何看待,入侵、种族灭绝和将人们赶出自己的祖国都是错误的。 那么你为什么还要继续试图为此做出脆弱的道歉呢? 只要承认这一点。 不管是谁做的,都是错的; 仅仅因为“其他人都在这样做”,并不能免除白人的罪责。 我们是否应该改革我们的司法系统并采用同样的逻辑?

    “世界各地每天都会发生谋杀案,因此将谋杀定为刑事犯罪是没有意义的,因为其他人都这样做。”

    “世界各地每天都会发生强奸事件,因此将强奸定为刑事犯罪毫无意义,因为其他人都这样做”

    “世界各地每天都会发生盗窃,因此将盗窃定为刑事犯罪是没有意义的,因为其他人都会这样做”

    白人似乎有一种独特的文化倾向,忽视和践踏他人的权利和考虑。 这种普遍的文化心态与白人的先进技术相结合,使他们能够征服整个世界。 是的,这是一项令人印象深刻的壮举,但仍然不道德。

    为什么当谈到技术发展等积极事物时,WN/另类右翼总是想吹嘘白人的独特性和不可或缺性,但当谈到大规模种族灭绝、文化破坏和抢劫等消极事物时,突然之间WN/另类右翼人士想谈论白人与其他人有何不同? 为什么不一致? 与另类右翼谈论欧洲殖民的道德问题就像试图与黑人谈论 OJ 辛普森的刑事罪责一样。 另类右派对此有一个巨大的盲点,他们几乎总是求助于 WN 版本的“we wuz just good boyz and dindu nuffin”,或者更常见的是“we wuz controrz”。 当涉及到这个话题时,你们都缺乏承认任何形式的内疚或错误行为的能力。 这对你们所有人来说都是相当糟糕的视觉效果,而且它只能让其他另类右翼人士信服。

    话虽这么说,按照 HBD 世界观; 如果每个种族都有某些内在特征,这些特征会影响一个种族以特定的方式行事并具有特定的优势(和劣势),那么为什么 WN/另类右派总是回避均匀地应用这种 HBD 种族决定论概念并且从不接受它得出其逻辑结论? 很明显,白人(一般来说)有一些特别之处,这使得他们能够如此成功。 但同样明显的是,白人(总体而言)有一些特质,使他们倾向于对外群体表现出专横和剥削的倾向。 如果您打算应用 HBD 推理,请始终如一地这样做; 不要挑选那些听起来对你来说方便的东西。 这只是自私而已。

    我们占领美洲是因为从根本上来说,美洲印第安人是石器时代的人。 (他们应该感谢他们的肮脏之神,因为中国佬没有在我们之前到达这里……他们都会消失,等等。)

    事实上,有大量证据表明,在哥伦布发现美洲之前,中国文化与美洲印第安文化之间存在着某种和平的接触。 虽然这些说法的有效性尚无定论,但有许多令人信服的论据和证据支持其可能性。

    老实说,如果你要把中国人带入这场争论,你就表现出对这个话题严重缺乏认识和了解。 中国人并不是暴力的殖民者(值得赞扬),历史记录也证明了这一点。 中国人在历史上的许多时刻都有能力殖民和破坏,但在大多数情况下,他们似乎只是满足于贸易和行使软文化/经济霸权。 听说过郑和吗? 明朝本来有能力用庞大的舰队侵略全世界,但他们却选择了和平的探索和贸易。 此外,由于中国文化向内求索的本质,明朝最终决定闭关锁国,整个舰队停飞并停止了任何进一步的远征。 这很能体现中国人的普遍心态。 如果他们有暴力的帝国主义倾向; 它的数量级小于白人。

    客观地说,我认为中国人会和平、平等地对待美洲人,而不是像欧洲人那样把他们视为征服者和被征服者。 你试图提出中国人会消灭美洲印第安人文化的论点,这只是你的一种公然、明显的投射形式。 当今发生的事件也支持了我的论点; 西方国家特别是美国继续试图对抗和破坏世界其他地区的稳定; 与此同时,众所周知,中国人表现得更加温和,并大力尊重其他国家的主权和完整,特别是在涉及其内政时。 虽然中国人并不完美,也有自己的缺陷(实际上有很多缺陷),但侵略性、帝国主义和与他人对抗(尤其是在全球范围内)并不是其中之一。

    我们把他们送到学校等等。没有任何作用,因为他们今天的智商是83,而且他们的性情非常不稳定和暴力,就像黑人一样。

    你怎么样? 你们不仅进行种族灭绝并将美洲原住民赶出他们的土地; 但你也将你的文化和生活方式强加给他们。 你能想象如果非洲或穆斯林入侵者给予欧洲原住民同样的待遇,另类右翼会强烈抗议吗? 哈哈。 但由于你缺乏同理心的能力; 你无法意识到你所说的是绝对荒谬、残酷和不合理的。 如果您认为您所说的是合理的,那么请告诉我您是否会容忍假想的非洲/穆斯林征服者对欧洲土著的同样行为😉

    您说您曾与认为白人有权控制世界的“另类右派”交谈过,等等。我认识其中许多人,我们的总体立场是每个种族都有权留在自己的祖国。 但不是我们的。

    是的; 这是我从大多数alrighters那里得到的总体印象。 他们过去纵容殖民主义,这强烈表明,如果条件合适,他们将来也会纵容殖民主义卷土重来。 我关注这个运动很长时间了,伙计,我知道你们实际上是怎么想的

    比较幽默; 为什么你试图诉诸道德/伦理论据来解释白人“应得”的东西,而就在早些时候你还基于实际考虑为美洲印第安人文化的种族灭绝和流离失所辩护? 在我看来,当道德/伦理论据服务于特定目的时,你喜欢使用道德/伦理论据,但只要适合你,你就可以毫无问题地恢复基于冷酷、严格的实用性和“就这样”心态的论据。 我的朋友,这被称为不一致,这强烈表明你缺乏诚信。

    所以根据你的推理; 白人有权利拥有家园,但美洲原住民没有权利拥有自己的家园,只是因为。 在你看来,当白人暴力地取代土著人民并在他们的土地上定居时,这是公平和公正的,但当移民在西方国家和平定居时,这是不公平的。 哇,多么令人惊奇的虚伪和双重标准,掩盖着脆弱和自私的外表。 此外,多元文化主义和大规模移民是你无法控制的,就像外国入侵和征服你的国家一样; 因此在这种情况下,强权就是公理。 不可阻挡的、强大的、全球和国内的力量迫使西方转型并采取多元文化政策,就像不可阻挡的力量迫使本土文化进行转型一样。 你可以毫无疑问地告诉美洲原住民克服它,那么为什么你自己不克服它呢?

    也要记住这一点; 白人是他们自己行为的受害者。 全球化有机地产生于作为西方殖民主义遗产的政治和经济网络。 此外,由于飞机和互联网这两项西方发明的技术进步,全球化已经发展和加速。 另类右派喜欢把西方的问题归咎于其他人(移民、犹太人等),但他们不知何故没有意识到,西方实际上只是在收获其所播下的种子的自然结果。 西方强行、暴力地启动了全球化工程,现在却屈服了。 如果你真的如此讨厌全球化和移民,那么你应该讨厌你的西方祖先造成了这种状况,而不是崇拜他们并为他们感到自豪。

    你还说白人认为他们可以控制世界。 你一定是在和犹太人说话,而不是白人。

    不,我非常了解犹太人,当我看到它时,我可以识别犹太人的表型和一般的犹太人心理。 与我交谈过的所有“白人”都认为自己可以控制世界,有权践踏其他人,为所欲为,但他们一直都是真正的、温和的、非犹太人的白人。 通常具有保守倾向,但并非总是如此。

    一般来说,历史上白人对世界各地土著文化造成的损害最大,而且事实上在今天仍然如此。 无可否认这一点; 白人非常擅长这样做,并且对此毫无愧疚。 很明显,白人是世界上不同文化的首要破坏者,甚至比“犹太人”更甚。

    最后你说另类右派没有同情心,等等。你的“好心”白人是自由主义者。 他们向那些屎人敞开了大门(由犹太人领导,他们一心要把白人变成自己国家里绝望的少数派),不仅摧毁了我们的种族士气,而且摧毁了对聪明和人道文明的任何希望……意思是意识到事情比如言论自由、法治、隐私等等,更不用说像登月这样的高智商了,而是高敏感性,这使得令人愉快的人际关系成为可能,尤其是两性之间的关系。

    白人发明了浪漫的爱情。 对于其他人来说,按照我的中国女孩的故事,或者黑人的阴沟性行为,这只是“生意”。

    白人是最好的全能种族。 我们确实发明了文明。

    肚脐凝视其最美。

    如果你想消灭白人,就从自杀开始吧,你这个傻瓜。 抓紧一点,种族叛徒……你们孩子们在哪里?……我猜是零。

    我不想消灭白人,我也没有在任何地方说过这一点。 典型的另类右翼组织夸张。 我很高兴能与白人和平共处,事实是,大多数白人与不同种族的人和平共处,并将不同种族的人视为朋友和爱人。 有趣的是,我知道另类右翼讨厌这样一个事实:大量白人实际上对不同种族的人都很好,而且不讨厌他们。 只是为了澄清一下; 我从一开始就只是提倡合理、客观、平衡地看待所有种族。 就像其他种族一样,白人也有优点和缺点。 我写这篇原创文章的唯一原因是纠正另类右翼的普遍误解,即白人是完美无瑕的,不会做出不道德的行为或做任何错误的事情,当你考虑到西方殖民的血腥和不公正的历史时,这显然是不真实的剩下的世界

    最可恶的是; 你实际上并不代表所有白人。 另类右翼也没有占据令人垂涎的“所有白人的代言人”地位。 事实上,大量白人积极反对像你这样的人,而我认为绝大多数其他白人完全无动于衷。 只有在互联网上,另类右翼才喜欢将自己任命为白人的统治者和代表; 与此同时,在现实世界中,另类右翼分子被大多数白人回避和忽视,哈哈

    • 回复: @Santoculto
  254. Factorize 说:
    @res

    是的,从严格的最大测量意义上来说这是正确的,尽管可能会出现天花板效应。

    由于有大量的 SD 可供任何配对选择,人们可能会简单地选择爱情和问题较少的行为特征,而不是最佳表型。 如果 10 SD IQ 变得广泛可能,15 或 20 SD 似乎不会提供令人信服的边际效益,特别是如果男性和女性之间出现巨大的社会/沟通障碍等成本。

  255. Factorize 说:
    @Daniel Chieh

    你的评论让我开始仔细思考提高智商的风险是什么。 你当然是对的,在没有对大脑有任何特殊了解的情况下简单地选择智能变体可能会导致意想不到的结果。

    然而,当然,目前的方法是举手投降,
    掷骰子似乎更不负责任。 考虑到自然繁殖的巨大遗传风险,我们很难相信深思熟虑的基因工程不能做得更好。

    鉴于科学进步的速度,可能需要几十年甚至几个世纪才能了解如何安全地增强智力。
    许多人可能会因等待而感到沮丧。

    与此同时,一些人尝试的一个策略是列出一系列特征/疾病,他们将尝试共同优化,避免可能增加风险的更极端的基因组合。

    • 回复: @Daniel Chieh
  256. Factorize 说:
    @res

    谢谢你的链接!

    我不确定中国是否是那些正式认同非心理测量世界观的国家之一。

    您是否知道五万年前基因变化可能将人类推向行为现代性的联系? 事实上,我更赞成人口统计学的观点,即人口密度创造了心理测量上增强的人类。 然而,从这些旧石器时代早期社区的许多消失中可以看出,这种影响非常不稳定。

    我相信您知道全新世的文章,其中讨论了大约 10,000 年前发生的更强大的遗传效应。 这可能是亚洲 g 边缘和 M/V 倾斜出现的时间。

    • 回复: @PandaAtWar
  257. res 说:

    您是否知道五万年前基因变化可能将人类推向行为现代性的联系?

    这是一个有趣的问题。 这是一个具有挑衅性的可能答案: https://www.amazon.com/Madness-Adam-Eve-Schizophrenia-Humanity/dp/055299930X

  258. @Factorize

    当然,目前这种举手投掷的方式似乎更不负责任。

    并不是那么可怕——等位基因的主导表达通常是那些问题较少的表达,严重的损害会导致以卫生形式自发终止(例如,男性出生更容易流产)。

    根据定义,如果你试图做出一些特别的东西,那么你就是在尝试构建一些异常的东西,所以你很可能会得到意想不到的结果。 但是,是的,我同意,与此同时,最有可能的是,尝试解决明显的遗传问题会容易得多,包括通过 CRISPR 解决严重自闭症等行为问题。 你现在在研究中看到的是这样的:

    https://www.technologyreview.com/s/546036/first-monkeys-with-autism-created-in-china/

    • 回复: @Daniel Chieh
    , @res
  259. @Daniel Chieh

    这里值得补充一下:很多医学都是这样,顺便说一下,它可以有证据表明我们知道它有作用,但机制未知。 例如,我们知道噻奈普汀具有抗抑郁作用,但对其作用机制却知之甚少。 甚至我们用于分析人脑化学活动的工具仍然很原始,因为我们不可能在 t=10 分钟时牺牲人类受试者来提取大脑来提取化学物质样本(例如,我们对克隆白鼠所做的事情),并且 FMRI 给出了我们的血液活动,但不提供化学信息,并且一直存在缺陷(著名的笑话论文提供“死鲑鱼显示情绪状态”)。

    我认为这很令人兴奋。 它在很大程度上是一片值得探索的荒野,也是我们理解的前沿,具有真正的收获并有能力造福人类。 增加恢复健康和赋予生命的能力,独立于任何增强功能,似乎是真正人道和仁慈的追求之路。

  260. Joe Webb 说:

    五万年前不存在行为现代性。 不用去看书本,也不看新石器时代等等,五万年是北方成熟的开始,它选择了亚洲和欧洲更聪明的人。 参见哈彭丁和科克伦的《万年爆炸》。 他们有些拐弯抹角,但……这是在寒冷气候下经过五万年自然选择后的最后一万年,才讲述了这个故事。

    林恩和拉什顿现在其他人已经概述了北半球与南半球之间的种族差异,尤其是在智力方面。

    没有冰河时代,就没有大脑。 非洲等地区最愚蠢的人,仅次于行为上的现代自由主义者,是最黑的,俾格米人和澳大利亚原住民垫底,智商低得惊人,只有 50 多岁。

    黑非洲的平均水平是 70,就像现在的海地一样。 小脑袋等于小大脑,等于低智商。

    智力像楼梯一样从北到南下降。

    想象一下刚从船上下来的非洲奴隶的智商。 1700等7o。 在美国宪法时代,五分之三的人是相当准确的,至少在智力方面是这样。

    这份清单上的行为/操作愚蠢令人惊讶。

    唯一真正愚蠢的人是那些上过大学并了解到没有什么区别可以带来改变的人。 彩虹魔法……梦见彩虹尽头的那一锅屎。

    • 回复: @Factorize
    , @bjondo
  261. res 说:
    @Daniel Chieh

    根据定义,如果你试图做出一些特别的东西,那么你就是在尝试构建一些异常的东西,所以你很可能会得到意想不到的结果。

    这是个好的观点。 我认为需要特别注意的一个情况是明显阳性(例如较高智商)等位基因流行率低。 我们可能应该非常小心地选择低流行等位基因的纯合病例(即自然界中不常见的病例),因为可能存在杂合子优势和/或隐性遗传病(例如泰萨克斯病)。

    也就是说,一种理论认为,遗传负荷至关重要,对人类基因组进行“拼写检查”,以便我们选择纯合的高频等位基因,这也可以最大限度地提高研究中的性状,这似乎可能是一场胜利。 尽管这种对遗传多样性的修剪似乎对这个群体来说存在风险,因为它倾向于单一栽培。

    我想知道尝试优化可能的杂合子优势(例如镰状细胞性状导致疟疾抗药性)的情况是否明智,如果下一代自然繁殖,这会增加遗传疾病的风险。

  262. Factorize 说:
    @Joe Webb

    谢谢你的回复

    是的,你关于一万年时间框架的建议是正确的,因为它是最重要的。 当时,随着人们试图应对所有的问题,基因进化爆发了。
    正在发生的变化。 特别值得注意的是此时的人口密度,尤其是在东亚。 在 10 kya 之前,我看不出人口压力(即人口密度)如何足以迫使基因发生变化。 如果我生活在那个时候,各种新的规则和规定开始出现,我就会开始走路。 所以10 kya确实是人类走向现代的关键时刻。

    有趣的是,直到近现代,非洲仍然拥有刀耕火种、非定居的农业系统。 只是现在他们已经没有空间了,需要做出社会变革来适应。

    我也没有完全意识到东亚有两个主要分支: 来自 2kya 的古老的
    另一个则不那么古老,包括 40 年前的欧洲人。 有谁知道是否做过心理测量GWAS来看看两个东亚分支是否可能存在差异? 研究欧洲人的 3 或 4 个部落也很有趣。 了解“农民”或“骑手”部落的后裔会产生哪些遗传差异,这将是一件很有趣的事情。 有没有什么地方可以上传你的基因组来找出答案?

    然而,大约 50,000 万年前的时间也被认为标志着一个变化时期:旧石器时代晚期的过渡。 所谓的行为现代性的研究结果出现在大约 150 至 30,000 年前的很长的时间范围内。

    我想知道是否可能存在与这种转变相关的基因变化。 然而,我现在赞成人口理论。 PMID:19498164 一旦人类住区达到一定的密度,那么上面引用的文章表明社区的平均技能水平会提高,因为会有高技能的人来模仿(根据社会学习理论)。 显然,非洲有很多地方在大约十万年的时间里失去了现代化。 欧洲的人口在大约100,000年前才达到所需的密度。 下图模拟了高(左)和低(右)人口密度,圆圈大小代表平均技能水平。

    我对理论的修改是遗传学而不是社会学习理论是驱动力。 当你有密度时,你自然会期望有一些智商更高的人,只是偶然的。 如果您的定居点为 10,智商平均值为 70,则可能有 1 个人的智商为 85。如果定居点为 100,则某人的智商可能为 100。不需要长期基因改造,并且更高端随着人口减少,智商会不稳定。 随着数万年人口的潮起潮落,微型文明的建立和消失会反复循环。

  263. Factorize 说:

    res,你能帮忙吗?

    我不确定“等高线图”是什么意思。 什么的轮廓? 等值线应该等于什么? 文章似乎没有澄清这一点。

    检测混合图中的多基因适应

    • 回复: @res
  264. bjondo 说:
    @Joe Webb

    发明回旋镖需要什么智商?

    • 回复: @PandaAtWar
  265. Anonymous [又名“约翰”] 说:

    什么是HBD? 谷歌一下,你会发现十几种可能的含义。 您应该始终在一开始就定义您的首字母缩略词。

    • 回复: @bjondo
  266. GammaRay 说:

    为什么管理员不让我的回复通过? 我认为让“意识形态正确”的发帖者(例如“joe webb”)回复我的一条评论是不公平的,但是当我尝试回复他留下的评论时,管理员不会让我的评论通过。 这样做是一种疏忽之罪,并且会产生一种错误的感觉,即我无法(代表我)反驳另类右翼的谈话要点,而实际上我可以毫无问题地解决他的论点并发表令人信服的回应。 你太无耻了 unz.com,这不是一个真正自由的论坛,另类右翼尽管对言论自由等西方价值观大肆宣扬,但实际上并没有实践它

    如果我写的是明显的垃圾邮件,那么我不会抱怨,但如果我写的是合法的回复,而我的回复最终被扔进了垃圾桶,那么很明显, unz.com 实际上并不相信这是一个能够容忍不同意见的地方; 尤其是有充分理由并有事实支持的替代意见。 它说了很多关于 unz.com 像乔·韦伯这样的评论者可以不受阻碍地吐出他们的胆汁,人们不可以合法地挑战那些如果没有审查制度就无法独立存在的观点。

    • 回复: @GammaRay
    , @Yan Shen
  267. GammaRay 说:
    @GammaRay

    之前有问题的评论(#277)在发布此评论后终于发布了,很高兴模组最终决定发布我的评论,问题解决了。

  268. Yan Shen 说:
    @GammaRay

    我的理解是,史蒂夫·赛勒是唯一在这里审核评论的人,其他所有内容都会通过自动过滤器。 也许您之前评论的长度是导致批准延迟的原因。

  269. Santoculto 说:
    @GammaRay

    但是但是

    “中国人”也不像地球表面的天使……

    是的,大多数保守派,无论他们的种族[以及他们的异常版本,不法分子],往往对他们的个人和集体行为很少进行自我反思。

    大多数来自极右翼的白人“刚刚”开始攻击犹太人引发的“白人罪恶感”,这是一种精心设计的道德剥削和勒索,以说服白人投降并庆祝自己的消失。 一种二元策略,是的,在智力上也很懒……

    • 回复: @GammaRay
  270. Santoculto 说:

    宏观种族之间的男性气质表现特征。

    情感/心理:自尊和支配力[或并非如此]

    一般

    非洲黑人

    认知:非语言/空间和定量

    一般

    东亚人

    人口统计:心理变异[“典型男性化”]……

    一般

    欧洲白人

    • 回复: @PandaAtWar
  271. Joe Webb 说:

    在使用“姓名”和/或“真实或虚构的电子邮件”框时遇到一些问题。 我已经和乔·韦伯(Joe Webb)或乔·韦伯(joe webb)一起发帖多年(!),并且一直收到消息“已在使用”或类似的公告。 所以我尝试了一些其他的事情,包括将第一个框留空,并将我的名字放在下面的框中。 这似乎有效。

    所以,需要明确的是,上面的“匿名者”是我,乔·韦伯。

    ----

    我们可以开始解析新石器时代的各种年代等等,以至无聊。 更一般地说,青铜时代等等让我们对北方民族的进步有了更扎实的认识。

    种族、进化和行为的基础……请原谅我的拉什顿行话……只是北半球及其严酷……迫使更聪明的人进行自然选择。 并无情地忽视了南方国家。

    南方国家的傻瓜已经输了。 就是这么简单。

    那么,个人主义与集体主义的因素也是深刻的。 东方专制主义是除白人和撒哈拉以南非洲人之外的所有种族的特征,他们只是混乱的,不属于个人主义或集体主义类别。

    乔·韦伯

    • 回复: @Anonymous
  272. res 说:
    @Factorize

    我认为等高线图本质上是数据点(单个 SNP)的密度函数。 我认为重点是尝试量化典型的效应大小,并了解哪个区域对于给定性状具有更大的选择压力和/或表型值。

    观察轮廓的峰值,我们发现 CHB(汉族人)倾向于导致比 CEU(来自犹他州的北欧人)更高的 EDU 和一字眉但身高更低的等位基因。

    另请参阅此评论: https://www.unz.com/jthompson/genetics-of-racial-differences-in-intelligence-updated/#comment-1897271
    图 4 和图 5 提供了查看数据的不同方式,得出了类似的结论。

    PS 如果其他人想要该图的放大版本或在上下文中查看它,请参阅第 78 页 https://www.biorxiv.org/content/early/2017/06/04/146043

    • 回复: @Factorize
  273. GammaRay 说:
    @Santoculto

    但是但是

    “中国人”也不像地球表面的天使……

    这里不存在矛盾或双重标准。 无论我在这里发布什么 unz.com,我会始终坚持自己的观点。 我不能容忍别人的虚伪或双重标准,尤其是我自己。

    话虽如此; 这是我写的关于中文的完整内容,它没有经过任何编辑或操纵:

    客观地说,我认为中国人会和平、平等地对待美洲人,而不是像欧洲人那样把他们视为征服者和被征服者。 你试图提出中国人会消灭美洲印第安人文化的论点,这只是你的一种公然、明显的投射形式。 当今发生的事件也支持了我的论点; 西方国家特别是美国继续试图对抗和破坏世界其他地区的稳定; 与此同时,众所周知,中国人表现得更加温和,并大力尊重其他国家的主权和完整,特别是在涉及其内政时。 虽然中国人并不完美,也有自己的缺陷(实际上有很多缺陷),但侵略性、帝国主义和与他人对抗(尤其是在全球范围内)并不是其中之一.

    中国人并非无可指责,也并非完美无缺,我一定要明确这一点。 你有责任在没有彻底阅读我所写内容的情况下试图找出我的论点中的缺陷。

    根据我的经验,大多数极右翼分子(我近十年来一直关注另类右翼)一致希望将西方所取得的所有美好成就归功于自己(有很多,我毫不犹豫地给予赞扬)功劳应得的地方),同时又希望摆脱西方所造成的所有负面影响,或者将其合理化。 好吧,你不能两全其美,你需要把一切的功劳都归功于自己,无论是好的还是坏的,或者什么都不归功。

    目前关于西方目前的困境; 我认为这只是西方过去不道德行为的自然结果。 西方创造了一个傀儡(全球殖民),现在它已经复活并威胁着西方。 虽然我同意,从非常有限的角度来看,西方似乎确实是受害者,但如果你稍微缩小一点,就会发现西方基本上正在自食其果。

    如果极右翼愿意对西方对世界各地传统文化造成的损害采取更加严肃和情感成熟的立场,我会更愿意同情某些极右翼的政治立场。 极右翼哀叹西方传统文化的丧失,同时庆祝/试图为西方对世界各地无数土著文化的破坏辩护,这是多么愚蠢和虚伪? 这有什么意义呢? 为什么我要对那些绝对拒绝对其他人表示同情或理解的人表示同情或理解呢?

    • 回复: @joe webb
    , @Santoculto
  274. Factorize 说:
    @res

    资源,谢谢!

    我不喜欢打扰别人做作业,尽管我不清楚这些等值线图。
    我不确定为什么文章中没有对它们进行解释。 似乎甚至没有关于颜色含义的传说。 然而,正如您所指出的,红色区域对应于东亚人的高教育水平和欧洲人的身高这一基本解释符合直觉。

    也感谢您提供人类家谱的网址。 我没有意识到澳大利亚原住民和新几内亚原住民在基因上与人类其他分支有多么遥远。
    他们几乎与非洲人一样独特。 您是否有一棵类似的树,其垂直轴上有年份?

    我还点击了提到 SNP IQ 结果的 pubmed 文章的链接。 当你阅读上下文中的评论时,很难将其标记为反驳。 我注意到您之前链接的身高文章使用了与智商论文类似的方法,但同行评审评论却非常负面。 即使作为粗略的计算,我也不确定该方法实际上是否存在缺陷。

    《一万年大爆炸》是一本很棒的书。 我并没有完全意识到人类的分歧可能更多地是关于种族,而不是关于物种。 该论坛上的大部分评论都致力于强调种族之间的差异。 然而,其他人提出的观点似乎有相当多的优点,即人类进化的时间尺度根本不足以让种族获得有意义的分化。 然而,我们可能更多地是通过不同原始人类谱系的基因渗入人类而不是种族来分开的想法改变了谈话。

    社会正义界真的相信尼安德特人-欧洲-东亚混血儿在认知上可能与那些没有这种基因渗入的人相似吗?

    资源、智商和智力是目前值得关注的一个令人兴奋的话题。 这里就是宇宙的中心! 在我以前的生活中,我可能会在奇怪的遗传学或人类学讲座中打瞌睡。 看起来就像是在捕捉蝴蝶。 然而,现在这一切却如此鲜活。

    桌面上的财富创造潜力是巨大的! 寻找智商的变异并对其进行选择对于人类来说将是一个巨大的时刻。 中国可能会发布 200,000 个智商 GWAS,总的基因解锁可能指日可待。 现在这不被视为全球紧急优先事项是没有道理的。 投资回报太大,不容拖延。

    尽管如此,遗传学革命已经开始了! 我一直在网上冲浪,已经有公司在营销这些产品。 知情人士无
    不再踢轮胎了。 比赛开始了。 这对于高 g 的人来说是显而易见的; 我希望那些可能落后一步的人不要以为自己可以打瞌睡。 落后一步很快就会变成落后一光年。

    自然繁殖是婴儿诞生方式的整个观念需要彻底重新思考。 鸟类和蜜蜂需要退休。 我们现在正处于一个世纪之初
    复制只能在实验室中进行的旅程。 对于多基因学,需要使用大量的基因技术。

    • 回复: @PandaAtWar
  275. joe webb 说:
    @GammaRay

    要求我们白人民族主义者道歉是没有意义的。 可以肯定的是,一些传统文化已被破坏。 这是一件好事。'

    1900 年末,阿根廷几乎消灭了所有印第安人。这即使不是一件好事,也是不可避免的。 印度文化已经臭气熏天,而且仍然如此,不断的战争,永不放弃的武士精神(很大程度上是在白人的帮助和怂恿下给他们带来了马。(我猜他们太愚蠢了,不会消灭美国的马,就像许多其他灭绝的哺乳动物一样)这可能已经被驯化了),这种酷刑堪比中国佬被千刀万剐而死。

    任何平均智商低于90左右的种族,基本上都是一种暴政的“文化”。 给我看一个不是的。

    就我个人而言,我从未见过值得拯救的土著种族,而不是“文化”,尤其是经常处于战争状态的美洲印第安人。

    向我展示一个相对“民主”的土著种族,具有相对女性平等、合法权利(忘记言论自由)等等。

    非洲……总计为零。 Amrerindiand…半全零。 亚洲……有一定价值,但不多。

    当冰雪在大约一千年后回归时,如果白人还在这里,我们将不得不向南迁移到非洲。 再见,非洲人。

    乔·韦伯

    • 回复: @Santoculto
    , @GammaRay
    , @Truth
  276. joe webb 说:

    “客观地说,我认为中国人会和平、平等地对待美洲人,而不是像欧洲人那样把他们视为征服者和被征服者”

    主观上你是个该死的傻瓜,而且很可能是个中国佬。 看看中国现在如何殴打少数民族或持不同政见者,然后就有了新的中国人终身独裁者。 说到生活,就买一个。

    乔·韦伯

    • 回复: @GammaRay
  277. PandaAtWar 说:
    @bjondo

    徒步旅行。

    它落后于前王朝埃及文明的智商。

    澳大利亚原住民仍然无法弄清楚尿液流的空气动力学原理,更不用说回旋镖了。

    • 回复: @bjondo
  278. PandaAtWar 说:
    @Factorize

    ......找到智商的变体并对其进行选择将是人类的一个重大时刻。 中国可能会发布 200,000 个智商 GWAS,总基因解锁可能指日可待……

    没那么乐观……

    人脑是地球上最复杂的机器。 不要以为 200,000 个 GWAS 就能解决这个问题。 另外,潜在的遗传副作用即使不是同样巨大,也应该是巨大的,因为无论是智能设计还是进化,毕竟没有免费的午餐。

  279. PandaAtWar 说:
    @Factorize

    我相信您知道全新世的文章,其中讨论了大约 10,000 年前发生的更强大的遗传效应。 这可能是亚洲 g 边缘和 M/V 倾斜出现的时间。

    熊猫能想到的唯一原因是:这可能是由于阿努纳奇人或他们当时的竞争对手。

    g的关键是大脑的大小!

    超膨胀灰质的额外立方体不会无缘无故地存在,这给所有智商研究人员带来了一个重大的礼物和明确的未回答的问题:

    德系犹太人(以及一般犹太人)的平均大脑大小是多少?

    综上所述,也许看似数学/语言的分裂(正如严慎在这里所说的)也只是一种自我感觉良好的错觉。

    • 回复: @TT
    , @Factorize
  280. PandaAtWar 说:
    @Santoculto

    宏观种族之间的男性气质表现特征。
    情感/心理:自尊和支配力[或并非如此]
    一般

    废话。

    非洲黑人在西方社会所表现出的“男子气概”,是在其他文明主要种族的纯粹善意、自责和自我克制之下表现出来的。 也就是说,说实话,确实多半是基于别人的怜悯之心。 当剩余的善意和/或耐心消失后,一旦受到严重的面对或挑战,他们就会退缩到所谓的“自尊”和“主导地位”。

    顺便说一句,人类男子气概的真正含义远远超出了野生动物世界中所表现和定义的简单暴力程度。 例如,与重量级疯牛相比,一位 80 岁的软太极拳练习者可能会在更多方面(无论是广度还是深度)表现出男子气概。

    此外,所谓“自尊”的心理概念和测试在某些方面存在严重缺陷。 学术界对它的定义如此迟钝,这开始惹恼了熊猫。 这个概念常常是在一种幻觉上进行检验的。 因此,在这种情况下,最好将其称为“错误的自尊”。

    • 回复: @Santoculto
  281. TT 说:
    @PandaAtWar

    熊猫,你听说过没有大脑还能活着的人吗? 所以即使大脑也没关系。

    剑桥大学的一位物理学家曾经告诉我,在世界大战期间,有记录表明,当士兵的头皮被炸开时,有些人里面实际上没有大脑。

    时至今日,仍有一些人没有大脑而活着,他们生活正常,但到医院检查后才发现。

    一些记录的病例:一名法国男子、一些孩子、婴儿、一名中国女士……还有许多未记录的病例。

    http://mysteriousuniverse.org/2017/07/miraculous-cases-of-people-who-lived-without-a-brain/

    http://www.dailymail.co.uk/femail/article-3293259/Baby-born-without-brain-amazes-doctors-celebrating-second-birthday-says-Mummy-time.html

    佛教解释说,我们的心是非物质的。 它是一种非物质的意识,依赖于我们心底的一些血液而存在。 所以当心死了,身体就会变冷,这个心也停止了,因此死了,与大脑无关。 脑仅被视为骨髓。

    因此,脑死亡的人仍然可以活着并复活,一个著名的例子是中国凤凰卫视主持人在火车相撞后在英国被宣布脑死亡,但在中国通过中医完全复活。 其实脑死亡的人还能思考,但无法用肢体表达。 因此,当家人与他们交谈时,有时他们的眼泪会流下来。

    一个例子可以说明我们的思想实际上驻留在我们的心脏而不是大脑中:当您感到非常悲伤或高兴时,您是否在您的心脏区域或头部有这种感觉? 尝试。

    自古以来,人们就用“心”这个词来形容思想、情绪、心态、开心。 甜心、碎心、心碎等等从来不用动脑子。 所以不要在情书中画出大脑。

    即使是古老的中医,如果你读过,它也清楚地解释了我们的灵魂魂魄在我们的心脏里。 但最近的中国医生被西医腐蚀了,现在正在歪曲中医文本来表示其大脑的思考。

    如果您有兴趣了解更多我们的身心如何运作,请阅读佛教阿毗达摩。

    https://www.google.com.mm/url?q=http://www.buddhanet.net/pdf_file/abhidhamma.pdf&sa=U&ved=2ahUKEwij0PP9mufZAhVGj5QKHZpKDvIQFjAAegQICRAB&usg=AOvVaw0NMS-qtezWrxFT4mQ87Z0E

    • 回复: @PandaAtWar
    , @Talha
  282. Anonymous • 免责声明 说:
    @Joe Webb

    您确定您是有意还是无意地忘记删除手机默认删除的您的姓名吗? 你用匿名写了愚蠢的评论。

    任何平均智商低于90左右的种族,基本上都是一种暴政的“文化”。 给我看一个不是的。

    尼泊尔78,和平竞赛。
    不丹80岁,位居世界最幸福国家第一。
    当然你不会同意古巴,缅甸人民不是暴君,因为他们不听从骗子叔叔的路线。 来自马尔代夫、斯里兰卡、老挝、马绍尔群岛、秘鲁、毛里求斯、哥斯达黎加……一些非洲人的和平人民怎么样,贫穷并不等于暴君。 他们从来不会像西方那样侵略、殖民、奴役他人。

    美国的智商是98,英国的智商是100,但这两个国家是世界上最暴虐的国家,他们在他们的整个历史中不断制造战争,在他们的整个历史中到处犯下暴行,直到我们打字的这一刻。 欧洲的法国人、丹麦人、西班牙人、葡萄牙人……也是如此,有着悠久的暴政历史。 日本人极其残忍。 和智商有什么关系?

  283. bjondo 说:
    @PandaAtWar

    但他们可以建造一个。 爱因斯坦和萨姆·哈里斯则不然。 R.费曼也没有。 提出这些智商测试的“天才”们也做不到。

    • 回复: @bjondo
    , @PandaAtWar
  284. bjondo 说:
    @bjondo

    原住民不仅可以设计、制造回旋镖,他们还知道自己想用回旋镖做什么以及如何最好地使用。

    尿流的空气或动力学是什么? 打赌腹肌的尿流可以击中蜥蜴。 理查德·道金斯则不然。

  285. PandaAtWar 说:
    @bjondo

    是的,熊猫也能咀嚼竹子,创造性地用后牙把它们绑成可爱的捆,然后以贪婪的速度和活力吞下它们。 有哪个智商高的天才能做到这一点吗? 你想说什么?

    • 回复: @bjondo
  286. PandaAtWar 说:
    @TT

    心的大小也一样吗? 确实! 感谢您提供的链接,但在您完全被阿毗达摩形而上学迷惑之前,请用您的心思考一下最终的衡量标准是什么? 球的大小? 正如他们所说,无论脑死亡与否,没有胆量,没有荣耀,因此没有佛陀。

    • 回复: @TT
  287. bjondo 说:
    @PandaAtWar

    如果你愿意读书,而不是咀嚼竹子、喋喋不休,你就会明白我的观点。

    熊猫智商中等。

  288. Santoculto 说:
    @joe webb

    1900 年末,阿根廷几乎消灭了所有印第安人。这即使不是一件好事,也是一件好事。 必然

    错了,与其他“人”建立良好的、甚至非常积极/互惠互利的关系是绝对有可能的。

  289. Santoculto 说:
    @PandaAtWar

    [……或者不是这样……]

    毫无疑问,黑人的自尊心平均水平较高。

    顺便说一句,人类男子气概的真正含义远远超出了野生动物世界中所表现和定义的简单暴力程度。 例如,与重量级疯牛相比,一位 80 岁的软太极拳练习者可能会在更多方面(无论是广度还是深度)表现出男子气概。

    斯特劳曼,我从来没有说过男性气质仅基于攻击性水平,但这并不意味着男性气质与它没有实质性相关,而且确实如此。

    我在评论中展示了男性气质、认知和人口统计的另一个方面。

    此外,所谓“自尊”的心理概念和测试在某些方面存在严重缺陷。 学术界对它的定义如此迟钝,这开始惹恼了熊猫。 这个概念常常是在一种幻觉上进行检验的。 因此,在这种情况下,最好将其称为“错误的自尊”。

    好吧,定义它...

    自尊基本上或主要是“非依赖性自恋”,即人们无论思想或外表如何都爱自己,这是一些人以强烈的方式拥有的主要自爱。

    自尊通常与外向有关,而黑人是最外向的人群之一。

    这是更弱智的攻击性谈论自尊概念,如此简单的一个。

  290. Santoculto 说:
    @GammaRay

    首先,你需要将普通民众与精英以及特定的历史精英区分开来。
    例如,当法国殖民主义者在海地奴役非洲黑人时,大多数法国人自己也被当地精英奴役。

    大多数白人是

    驯化,我们在大多数“文明”的地方也可以看到,中国也是如此。

    他们太驯服了,无法面对历史精英的态度,而现在他们却因自己从未犯下的罪行而受到指责。 【只有当某些知识精英触发他们的敌意时】

    对于英格兰来说也是如此。 过去,大多数英国人都很穷,而最穷的人在工业革命期间受到了极大的剥削。

    事实上,当你说“白人”时,你不是在谈论普通白人,而是历史精英和入侵其他土地并制造所有这些废话的特定群体。

    为什么我要对那些绝对拒绝对其他人表示同情或理解的人表示同情或理解呢?

    因为很多人都陷入了绝望。 好吧,根据这个逻辑,你一定对犹太人也感到愤怒或漠不关心,不是吗?**

    更重要的是,因为他们与白人一起参与了其他国家的大多数“白人”犯罪。

    抽象的“白人”或“白人种族”是指责普通人的一大替罪羊

    历史精英[并非全部]

    托斯

    精英们本质上往往是古典主义者,这是他们的“种族主义”方式。

    • 回复: @Santoculto
    , @GammaRay
  291. GammaRay 说:
    @joe webb

    搞笑。 任何道德或伦理的伪装都会落空,WN 的真面目终于显露出来。 正如我所怀疑的那样(你相当代表了一般的 WN/另类右派); 白人民族主义从来都不是为了自我保护,而或多或少是对全球霸权的不加掩饰的攫取。 如果你们都坦率地表达重建全球白人至上的愿望,而不是试图隐藏在虚假的道德外表和“所有人民都有拥有自己家园的人权”的道德陈词滥调后面,我会对 WN 更加尊重。 快讯,你不是法官、陪审团和刽子手。 白人无权决定哪些文化可以生存,哪些文化必须灭亡。 白人目前处于(犹太人)剑的另一端,感觉不太好,不是吗? 那么,为什么你继续对那些你认为比你弱的人怀有恶意呢?

    而且,你似乎认为智商、战术、技术等都很重要。 你可以毫无疑问地为白人利用卓越的情报、技术、组织等手段对其他文化进行种族灭绝或破坏进行辩护; 话虽这么说,那么你为什么如此反对犹太人对白人的霸权呢? 根据你的达尔文主义世界观; 这确实关系到最强大或最聪明的人的生存。 话虽这么说,根据严格的 WN HBD 世界观,犹太人似乎拥有在现代世界取得成功所需的条件,并且毫无问题地超越外邦白人。 所以,用你自己的逻辑,现在的事态是有道理的,用你自己扭曲的道德标准。 犹太人比白人更聪明、更狡猾、更老练; 所以按照你的逻辑,他们的霸权是正当的。

  292. GammaRay 说:
    @joe webb

    “客观地说,我认为中国人会和平、平等地对待美洲人,而不是像欧洲人那样把他们视为征服者和被征服者”

    主观上你是个该死的傻瓜,而且很可能是个中国佬。 看看中国现在如何殴打少数民族或持不同政见者,然后就有了新的中国人终身独裁者。 说到生活,就买一个。

    你基本上很生气,因为我用可能的历史论证以及你无法反驳的现代例子来支持我的主张; 这就是为什么你必须诉诸软弱的“你现在”论点。 确实,这是你的错,如果你不想在这个话题上受到困扰,那么你一开始就不应该提出一些你并不真正了解的东西。

    你所说的任何关于中国的言论都不能反驳我所说的任何关于中国的言论。 西方人似乎天生就比较阿尔法、好斗、专横、外向。 这就是他们寻求征服整个世界的原因。 我很确定99%的WN都同意这个说法,我也在提出同样的论点。 同样,中国人似乎天生就比较贝塔、平和、顺从和内向,这就是为什么他们从来没有试图征服世界,而是试图以大致平等的方式与他人打交道,而不是试图公然统治他们。 你生气只是因为我一直在使用 WN HBD 逻辑,而在这种情况下,它恰好使白人对中国人产生负面影响。

  293. TT 说:
    @PandaAtWar

    我只想指出一个大多数人都不会意识到的医学证据,人类甚至不需要大脑的存在就可以正常生活。

    非物质性不能用物质性单位来衡量。 但既然你更喜欢用球来思考,考虑到你的智力高,球的尺寸必须相当大,所以浪费时间继续讨论是没有意义的。

    阿毗达摩起源于公元前2500多年前的印度佛教,其深刻性至今仍超越任何现存理论能够以最科学的方式完整解释名色的理论。

  294. GammaRay 说:
    @Santoculto

    我对你的论点有两个问题:

    1. 你试图指出,大多数白人由于自己的契约/奴隶身份而没有直接或间接地从历史上和现在的殖民主义中受益

    实际上,我们知道这基本上是不真实的,因为白人过着非常舒适的生活,并在他们的祖先从土著人民手中夺取的土地上创建了非常成功的国家; 英语圈充满活力和繁荣的事实就证明了这一点。 欧洲国家的精英无法凭一己之力征服和定居这些国家,他们还必须利用欧洲无产阶级的人力。 我认为,当谈到西方文明在全球的物化和持续传播时,你极大地低估了普通西方人在历史上和现代背景下的同谋,这是非常不诚实的。

    2.我的论点实际上并不是反对白人,我想我已经一遍又一遍地表达得很清楚了。 我的论点只是攻击 WN/另类右翼分子的虚伪,他们想声称白人是受害者,但同时又想庆祝白人对其他种族的毁灭。 如果你认为我在攻击白人(你显然是这样认为的)或者对白人怀有某种恶意,那么你就大错特错了。 当大多数白人都是好人,他们基本上只想过自己的生活并且不关心任何这些事情时,为什么我要对白人怀有一些怨恨呢? 我唯一反对的人是虚伪的 WN/另类右派

    为什么我要对那些绝对拒绝对其他人表示同情或理解的人表示同情或理解呢?

    在其最初的语境中,我明确指出为什么我应该对 WN/另类右翼人士表示同情或理解,而他们无法对其他人有任何同情或理解。 我并不是指整个白人,我认为这从我最初写的内容中已经非常清楚了。 你需要清楚地阅读和理解我所提出的论点,而不是急于写出反驳,因为反驳甚至可能没有解决正确的问题。

    话虽如此; 虽然我不反对白人,但同时我也不会说白人是现代背景下的受害者。 正如我之前所说,是西方国家通过殖民化开启了整个全球化进程,现在西方国家将不得不应对意想不到的后果。 我并不是在道德上反对白人,认为他们应该在文化和种族上解体;我只是在道德上反对白人。 相反,我提出一个实际的论点,即从因果关系来看,前几代白人发挥作用的力量现在正在卷土重来,并对当今几代白人产生负面影响。

    最重要的是; 所有这些基本上都是抽象的。 事实上,大多数白人要么支持多元文化主义和全球化,要么完全漠不关心。 事实上,当你将年轻一代与老一代进行比较时,这两派白人甚至更强大。 当你考虑到当今白人和非白人之间存在大量的跨种族关系和友谊时,这种情况变得更加复杂。 仅这些关系的存在就使得任何类型的“硬白”(玩弄“强硬右派”一词)政策的大规模实施几乎不可能实施。 广大白人根本不具备政治意志力。 这样说吧; 即使是数量惊人的另类右翼也无法控制他们对亚洲女性的迷恋,那么为什么他们期望其他被驱逐的白人能够拒绝种族混合或与其他种族的任何其他类型的良性互动呢? 因此实际上来说; 当大多数白人自己都不关心时,我为什么要费力倡导白人权利呢?

    • 回复: @Santoculto
    , @Santoculto
    , @AaronB
  295. Truth 说:
    @joe webb

    当冰雪在大约一千年后回归时,如果白人还在这里,我们将不得不向南迁移到非洲。 再见,非洲人。

    你们已经没有孩子了,所以我想你们最后的 47 个人可以不引人注目地偷偷溜过去。

    • 哈哈: Talha
  296. Santoculto 说:
    @GammaRay

    1. 你试图指出,大多数白人由于自己的契约/奴隶身份而没有直接或间接地从历史上和现在的殖民主义中受益

    对于欧洲历史的大部分来说,是的。 间接多于直接。

    而且总是同样的事情。 许多中国人[精英和中产阶级]也因剥削穷人而受益[不考虑被视为垃圾的非人类动物]。

    欧洲人和全球其他地区的区别在于,第一个原因是全球范围内所有道德上错误的事情造成的。

    实际上,我们知道这基本上是不真实的,因为白人过着非常舒适的生活,并在他们的祖先从土著人民手中夺取的土地上创建了非常成功的国家

    错误的。 即使没有从其他地方提取的自然资源,白人也能够在没有工业化/和人口爆炸的情况下创造一个美好的社会。

    再说一遍,大多数人都是被驯化的。 责备狗因“主人”的虐待狂而杀死狐狸仍然不是理想的正确推理。

    “原住民”在新世界中并不总是和谐的民族群体。 事实上,白人入侵者所做的事情总是发生在美洲。 是的。 这是不可原谅的。 你在这里言论的主要问题似乎是,你只是指责白人有肮脏的过去或忽视他们,即使这不是完全错误,也要给予特别考虑,因为“白人错误”已经全球化,现在包括中国在内的每个人,有机会效仿他们。

    你试图解决一个问题,却制造了其他问题,并导致了不公正。

    白人得到了好处,首先,是一个非常简单的叙述。 今天,就在最近,因为生活和开发新土地无疑是一项非常困难的工作,特别是对于普通白人来说,而且他们再次被剥削,过着悲惨的生活,特别是工人阶级。

    我认为,当谈到西方文明在全球的物化和持续传播时,你极大地低估了普通西方人在历史上和现代背景下的同谋,这是非常不诚实的。

    我想如果你对你的人民和政府也这样做的话。 我不是。 我试图以中立的方式分析这个复杂的东西,我赞成赔偿,批判性分析,不仅仅是白人文化和历史,而是所有人类文化和历史,但是,有些事实对我们的感受漠不关心。 建立价值观固然重要,但首先把握事实也很重要。

    海地这个问题超级严重的国家与过去的奴隶制没有什么关系,根本原因之一是因为海地人已经有足够的时间来重建自己的国家,不是吗?**

    这种种族和行为现实对于真正结束一连串愚蠢的人类冲突非常重要。

    好吧,我理解你关于 WN/另类右翼虚伪的观点,这种虚伪在大多数部落民族中都很流行,无论是关于国家、种族还是意识形态。 我也攻击了这种虚伪,实际上我对所有这些废话感到厌倦。

  297. Santoculto 说:
    @GammaRay

    话虽如此; 虽然我不反对白人,但同时我也不会说白人是现代背景下的受害者。 正如我之前所说,是西方国家通过殖民化开启了整个全球化进程,现在西方国家将不得不应对意想不到的后果。 我并不是在道德上反对白人,认为他们应该在文化和种族上解体;我只是在道德上反对白人。 相反,我提出一个实际的论点,即从因果关系来看,前几代白人发挥作用的力量现在正在卷土重来,并对当今几代白人产生负面影响。

    再说一遍,大多数人,特别是文明地方的人,都是被驯化的。 它们已被选中来被驯服。 当然有: 驯化程度; 驯化的类型。 大多数白人都是驯养的,智力不那么聪明。 是的,他们是现代环境的根本受害者,因为他们被指控犯有其大陆/欧洲之外的所有全球化西方错误,并被用作牺牲品[其所有文化都已被完全破坏,其原始土地,最重要的是,其物质安全正处于日益危险之中],就好像西班牙人民决定开发大西洋以外的新土地……这完全是一个精英计划,通常都是如此。 我们可以说,人们要么有罪,要么没有罪。 他们有罪,因为他们对自己的精英感到自满[就像狗对主人对狐狸的态度感到自满一样]。 但因为我们并不都是一样的,完全有能力达到理性,所以我们不能在指责精英之前责备人们,特别是当人们被驯化时。

    因此实际上来说; 当大多数白人自己都不关心时,我为什么要费力倡导白人权利呢?

    大多数另类右派似乎无法理解的要点之一。

    如果你想保护你的种族,就组织起来,但他们中的许多人相信,在不久的将来白人会觉醒并开始像他们一样思考,也许,也许不会……

    白人觉醒于许多谎言(与真相相结合)的运动已经发生,进展非常缓慢,但情况正变得如此危急……

    白人所发生的事情是不公正的,这与黑人在美洲(以及非洲)被奴役时所发生的事情是一样的。 我反对一切形式的不公正,这就是我的理由。

    你没有/没有回答关于犹太人的问题......

    • 回复: @GammaRay
    , @GammaRay
    , @GammaRay
  298. AaronB 说:
    @GammaRay

    感谢您的明智评论。 我并不同意你的所有观点,但我自己也提出过你的许多论点,但都无济于事。

    祝你好运,始终将 HBD 参数应用于 WN。 观看起来很有趣,但它可能不会给你带来任何帮助,因为 HBD 的作用只是一个神话,而不是一个合法的理论。

    同样,严慎是一个极端民族主义的亚洲人,他的帖子存在极大的缺陷和不一致,但其要点是有选择地筛选证据,以创造一个验证崛起大国的神话。 这是很自然的事,我们应该宽容。

    从这个意义上讲,我认为你关于中国人如何对待美洲原住民的想法有点太天真了。 是的,他们可能不会以这种方式屠杀和掠夺他们,但会有某种剥削。 肯定会更温和,但不是平等的关系。

    可悲的是,白人在西方确实受到攻击和妖魔化,即使这是他们释放力量甚至他们自己文化的结果。

    我认为一些白人为复兴旧文化做出最后的努力是正常的,但这些事情永远不会奏效。 不过,你也不能责怪他们。 对他们来说,指出白人文化的消极方面并没有帮助,因为白人文化创造了当前的条件。 他们已经被妖魔化了,他们也想要一个新的神话。 所有的权力争夺都需要自我验证的神话。

    我认为,与其试图复兴旧有的浮士德文化及其犯罪行为和不稳定的极端主义,不如让它自然消亡,让一些新的、或者可能是旧的和预先存在的东西——会更好。浮士德,出现。

  299. @AaronB

    同样,严慎是一个极端民族主义的亚洲人,他的帖子存在极大的缺陷和不一致,但其要点是有选择地筛选证据,以创造一个验证崛起大国的神话。 这是很自然的事,我们应该宽容。

    呃。 他的亚洲必胜主义有时会让我感到非常沮丧,因为我觉得它包含了各种半真半假的事实,以构建一个叙事——从这个意义上说,与硬核 WN 非常相似。 我还认为,这通常是由个人自尊心受伤的感觉驱动的,这可能会导致动机推理。

    我不太关心这一点,但我认为这可能会导致理解错误,因此与认识论的美德背道而驰:它不会导致更好地了解世界。 首先,我想更好地了解世界,并希望为人类知识贡献自己的一点力量。

    • 回复: @AaronB
  300. AnotherDad 说:
    @anony-mouse

    快讯:东亚人数学很好!

    以前从未听说过!

    Yan,白人和东亚人的不同认知特征是一个有趣的研究课题。 并且确实对亚洲和西方的未来轨迹产生重大影响。

    然而,你将这种偏差描述为富有洞察力,这很奇怪,尤其是作为 HBD 人忽视的事情。 当你注意到时,林恩也注意到了。 塞勒注意到了这一点。 它存在于所有 SAT 和 PISA 数据中。 我不知道有哪个 HBD 信徒认为白人和东亚人有相同的认知特征。 (正如我多次指出的那样,如果某个特征不是固定的,而是在种族群体内的个体之间有所不同,那么它在种族群体之间也一定有所不同。这只是数学。)我们大多数人根本不那么着迷主题如你所愿。 西方面临的这场危机正在揭穿“每个人都是一样的”和“多样性是我们的力量”的胡言乱语,这些胡言乱语让我们的家园充斥着外国人——尤其是智商低、文化上陌生、无法支持文明的外国人。 (我们仍在尝试了解 HBD 101。)

    但“亚洲人擅长数学”是如此明显,这不仅对 HBD 人来说是显而易见的,在情景喜剧比喻的层面上也是显而易见的。

    ~~~

    最后,您在这里获得了“基本”见解:

    我希望我已经让你相信,思考 HBD 的正确方法基本上是沿着数学/语言划分的方向,而不是沿着整体智商或 g 的方向,而不是那些概念没有它们的相关领域使用。

    完全错误。 从全球角度来看,一般情报完全主导 HBD 问题。 如果你有一个变量可以谈论种族和个人的表现……就是这样。

    即使在东亚的崛起中,东亚人的智商在数学上也只排在第三位。 你不仅错误地认为智商排名第一,你甚至还没有弄清楚亚洲人相对于白人(和其他人)来说最重要的是什么,帮助亚洲向前发展:

    #1——高智商。
    #2 — 东亚人的性格概况。 高度的责任心、合作性、服从性、吃苦耐劳的“磨床”性格。
    #3 — 数学方向。

    1+2实际上在帮助亚洲追赶上占据了主导地位。 即使亚洲人的数学/语言智商与白人相同,他们也能实现非常相似的飙升并迎头赶上。 这实际上与东亚人服从命令和努力工作的高能力所增强的智商有关。

    如果没有第三名,亚洲人就不会在高科技领域占据如此大的主导地位。 但东亚人认真勤奋的工作能力仍将使他们成为制造业强国——包括高科技。 (大多数东亚人并没有设计任何东西,他们只是把东西放在一起,但与西方相比,他们做得更好,缺陷更少,抱怨更少,成本更低,更不用说第三世界了。)

    亚洲人的数学/语言偏差真正脱颖而出的地方,与其说是他们已经实现的赶超,不如说是我们所看到的未来——首先是日本,然后是韩国,但最终我们会看到来自中国——他们实际上在中国处于领先地位并生产出更先进的产品。 (不仅是廉价生产的更好的产品,而且实际上是更先进的产品。)

    东亚的未来看起来相当光明。 (如果中国能够避免至上主义和对其他所有人发号施令的话)。 但事实是,即使如此,数学/语言的分歧也不是主要原因。 主要原因是西方正在通过多样性自杀。 如果东亚国家避免这样做……他们就会默认获胜。

  301. AaronB 说:
    @Daniel Chieh

    严申的动机在很大程度上肯定是受到了自尊心的伤害,我们可以从他对 unz、unz 之前的 Sailer 以及 Steve Hsu 网站的长期评论中看出这一点。 他实际上已经成熟了一点,这是有希望的。

    是的,它肯定会妨碍清晰的思维,但这种事情可能不会消失,人性就是如此。

    我们可以宽容地微笑,对许多人来说在心理上是必要的,容忍它,并做出平行的努力,真正试图理解一个更加复杂和迷人的世界,它不断变化和突然逆转。

    毕竟,我们任何人所说的任何话都不会对 WN 或任何其他类型的 N 产生任何影响,尽管展示它们的谬误仍然很有趣,并且它可以吸引人们对清晰思考更感兴趣。

    无论如何,对清晰视力感兴趣的人很少见,这需要一定程度的脱离生物生存的斗争,而我们谁都无法完全摆脱这一点。 生物的生存需要神话和叙述,而我们只能部分地摆脱它们。

    希望您对清晰的理解和知识感兴趣。 我绝对同意这一点。

    但太多清晰的理解可能与生物学利益不相容,也许我们甚至应该节制对知识的追求。

    虽然我打赌你会和我分道扬镳🙂

    • 回复: @Yan Shen
  302. Yan Shen 说:
    @AaronB

    严申的动机在很大程度上肯定是受到了自尊心的伤害,我们可以从他对 unz、unz 之前的 Sailer 以及 Steve Hsu 网站的长期评论中看出这一点。 他实际上已经成熟了一点,这是有希望的。

    根据评论者 AnotherDad 的说法,你呃还遗漏了约翰·德比郡的爱情生活或类似的事情让我受伤的部分。

    • 回复: @AaronB
  303. AaronB 说:
    @Yan Shen

    放松,Yan,你所说的一切都没有让我认为你的问题是浪漫的失败。

    不过,您可能会对约翰·德比郡产生痴迷。 我可以理解这一点。 他可能很自鸣得意。

    • 回复: @Daniel Chieh
  304. @AaronB

    不过,您可能会对约翰·德比郡产生痴迷。 我可以理解这一点。

    强大的数学智商甚至让男孩都对他感兴趣?

    • 哈哈: AaronB
  305. myself 说:
    @Talha

    科举制度就是近代之前中国人的智商测试。

    制度的概念可以追溯到统一的中华帝国之前,可能是战国时期之前。 战国对所有的战斗人员都有自己的版本,但科举制度在秦朝才正式化。 它一直使用到 1912 年左右。

    任何人都可以参加,甚至许多非汉族人也参加了考试。 没有收入或财产要求。 它测试了理论知识和实践知识。 自然哲学(后来被称为“科学”)、哲学、数学、天文学、军事实践、行政管理等。这些问题是政府能设计出的最难的问题。

    没有预留的通过位置,不尊重“遗产”(你的父亲是一位出色的官员,并不意味着如果你不合格就蹲下),没有恩惠。 任何被发现的作弊行为都会受到惩罚,从公开羞辱你的赞助商和家人,一直到处决。

    评分是“盲”进行的——评分者不知道考生的名字,只能对部分试卷进行评分,以防书法被识别。 每个考生都有多个评分员。 所以说还是比较公正的。

    如果你通过了,你就可以担任(非常)初级的正式职位,其中你必须证明你的技能和能力才能晋升到更高的职位。 虽然你是从最底层做起的,但现在你已经是一名政府官员了,按照你的能力,你会走上更大的道路。

    如果这不是古代版本的智商测试,我不知道什么才是。

    • 回复: @Ron Unz
    , @Talha
  306. Ron Unz 说:
    @myself

    科举制度就是近代之前中国人的智商测试。

    制度的概念可以追溯到统一的中华帝国之前,可能是战国时期之前。 战国对所有的战斗人员都有自己的版本,但科举制度在秦朝才正式化。 它一直使用到 1912 年左右。

    事实上,世界各地使用的笔试很可能最终都是源自中国模式:

    https://www.unz.com/runz/how-social-darwinism-made-modern-china-248/#p_3_4

    • 回复: @Truth
  307. Talha 说:
    @myself

    非常好的评论——谢谢您提供的信息。 我希望其他人也关注!

    和平:

  308. Truth 说:
    @Ron Unz

    NOPE。

    来自瓦坎达,小狗,查一下。

    • 哈哈: Yan Shen
  309. Talha 说:
    @AaronB

    是的 - GammaRay 已经很好地阐述了它 - 指出了不一致之处以及其他地方。

    非常好观察。

    和平:

  310. Yan Shen 说:
    @AaronB

    同样,严申是一个极端民族主义的亚洲人,他的帖子也有极大的缺陷

    当然,你已经重复了很多次了,你还没有指出我文章中的实际缺陷是什么。 另一方面,我的文章引用了近 50 条参考文献,并提出了基于真实数据的论点!

    现在,有些人认为我在描述社会差异如何归因于认知差异时有些过分了。 另一方面,东亚人之间存在数学/语言分裂肯定是心理测量学文献中相对完善的事实之一,但我认为这一事实没有得到充分重视,特别是在 HBD 的背景下。

    在我看来,这种经验现实对现实生活没有影响是相当令人震惊的。

    • 回复: @Yan Shen
  311. Yan Shen 说:
    @Yan Shen

    嗯没有机会完成我之前的评论的编辑。 我文章的主旨不是关于认知能力的种族差异,而是更多关于在分析尾部人才时,数学/语言分裂如何对整体智商或 g 等结构构成重大障碍。

    现在所有这些其他的东西都伤害了这个,伤害了那个,我认为弗洛伊德或其他人提出了心理投射的概念,但也许我会把它留给其他人自己决定......😉

    • 回复: @Santoculto
  312. AaronB 说:

    哦,孩子,我们让你生气了:)

    你的文章有很多错误,但最主要的是你无法区分解决问题的毅力(动机的作用)和智商得分的先天能力。 因此,任何试图将智商差异描述为与生俱来的,而不是历史上偶然的动机功能,无论它们是什么,无论多么可以理解,都只能是一个神话。

    目前亚洲的分裂似乎更有可能只是由于强烈关注在技术上追赶西方的结果。

    对于所有群体都是如此。 我不相信白人有像 WN 所说的特殊的“创造力基因”。 我认为白人只是暂时比其他任何人都更想要权力和技术,而犹太人则痴迷于财富和权力,正如我所知,他们是在他们中间长大的。

    这些都是并非永恒的历史时刻。 研究“文明的循环”。 每个崛起的文明都认为自己的时刻是永恒的。

    群体之间可能存在先天差异,但我们无法知道动机和能力的真正崩溃是什么。

    至于黑人和棕色人种,我们无法知道他们的低智商在多大程度上只是由于对技术和现代性等不感兴趣而造成的。

    我们现在所做的是,一个群体的懒惰名声与其智商密切相关。 这不是很奇怪吗?

    我是一个机会均等的人。

    不管怎样,这个论点很复杂,涉及很多领域,我不希望你在这里重复它。 有兴趣的话可以搜索我的账号。

    Ron Unz 在他的 IQ 文章中也对此做出了一些非常中肯的评论。

    我很高兴这将是亚洲世纪——我是亚洲传统文化的崇拜者,我对它如何与技术融合以及将出现的新文化形式着迷。

    所以,祝严神你世间一切顺利。

    很抱歉我不能说更多,但我无法与你在这个主题上的情感投入相匹配,我很高兴任何喜欢你的叙述的人相信它,如果它能让他们满意的话。

    我认为弗洛伊德或其他人提出了心理投射的概念

    你没有欺骗任何人🙂

    • 回复: @Talha
  313. Factorize 说:

    这个话题怎么看起来这么晦涩难懂呢? 就我个人而言,我想其他人并不完全熟悉东亚 M/V 倾斜。
    了解视觉空间上的倾斜程度也很有趣。

    人们怀疑,如果 V/M 的倾斜程度相同,那么亚洲大部分地区的经济轨迹将会大不相同。 考虑到这项技能的极端价值,我将非常有兴趣看到 M GWAS 的未来报告。

  314. Factorize 说:
    @PandaAtWar

    熊猫,这是一个令人惊讶的想法,它可能部分是由尼安德特人基因渗入造成的。
    当我们离开非洲时,我们立即遇到了尼安德特人,他们的基因与人类分离了至少 XNUMX 万年,并且可以转移帮助他们在欧洲生存的基因变异。 这是值得关注的科学。 他们能够给我们带来多少智商提升?

  315. Anon • 免责声明 说:
    @AaronB

    可悲的是,白人在西方确实受到攻击和妖魔化,即使这是他们释放力量甚至他们自己文化的结果。

    你如何妖魔化一个自从他们的文明开始掌权以来就在各地制造暴行的恶魔?

    • 回复: @Daniel Chieh
  316. Santoculto 说:
    @Yan Shen

    我想用两个假设来解释为什么尽管地球上有1,3亿中国人,但至少现在(在当代)他们中间还没有找到真正的天才,大人物而不是真正的天才[认知上]有天赋的人,他们改进了已经发明/现有的技术[如果现代中国艺术和哲学看起来一点也不伟大]。 “中国人”不再像他们的祖先那样具有创造力的历史原因是,随着驯化的增加,他们的人口发生了变化,基因库中的外来者被大量消除或减少。

    有天赋的人可分为三种基本类型:高成就者、聪明者和创意者。

    http://www.bertiekingore.com/high-gt-create.htm

    我的第一个假设是,尽管东亚大陆的平均智商较高,但他们却没有培养出相对较高比例的高度创造力的人,我的第一个假设是,他们的大多数“认知精英”都是由高成就者组成,而创造性思想家所占的比例却很小。

    我解释 blablabla 的组合假设是,较高的[认知]智力单独选择更好的能力来集中/关注现有的细节。 更高的认知智能基本上是强大的语义记忆和一般知识,至少在表面上是这样。 但创造力与大脑的“缺陷”有关:与记忆和过滤信息(即注意力能力)相关的大脑区域活动减少。

  317. @Anon

    非白人文化也有其暴行,将所有白人混为一谈是愚蠢的。 波兰人曾经有过奴隶吗?

    • 回复: @TT
  318. Talha 说:
    @AaronB

    我是亚洲传统文化的崇拜者,我对它如何与技术融合以及将出现的新文化形式着迷。

    我也是。 然而,让我难过的一件事是回顾中国主要城市中涌现的一些新建筑。 不幸的是,其中大部分内容都是与过去的决裂,并且充满了后现代的色彩。 有些很漂亮,但我遇到的大多数看起来与巴黎、纽约或迪拜建造的东西没有什么不同。

    真的很难过……

    但也许正如尼尔·波兹曼所写的那样; 超级技术与其说是拉拢,不如说是抛弃了其文化范式的其他竞争对手。

    在那些完全搭乘“技术垄断快车”的人的心中,正在出现的全球共识是关于什么为社会提供价值、社会的最终目的是什么,甚至这些价值所衍生的艺术印象,这是非常有说服力的。

    和平:

    • 回复: @AaronB
  319. Talha 说:
    @TT

    感谢这些链接。 这是一个很真实的现象。 我第一次在鲁珀特·谢尔德雷克(Rupert Sheldrake)的著作中看到关于人们看似正常生活但没有太多大脑物质的报道。 它确实彻底颠覆了唯物主义对心灵和意识的假设。

    和平:

    • 回复: @TT
  320. AaronB 说:
    @Talha

    你所说的肯定有很多道理,但我认为我们将开始看到的不是美丽,而是微妙不同的感性。 例如,日本有一些闪亮而现代的酒吧,在西方,这些酒吧正是我会避免的那种寒冷和疏远的地方,但不知何故,日本人已经把它们变成了这些温暖、亲密和舒适的地方。 这不是我的想象——我已经和日本人讨论过了。 它是技术与不同感受的结合。 霓虹灯在西方显得俗气和花哨,但在亚洲却显得温暖和凉爽。 不仅仅是日本——我在台湾、泰国和中国的一些地方也看到过这种情况。

    大型项目具有“威望”——它们具有不同的功能。 除此之外,我希望技术的“人性化”能够出现,与其说是美丽和宏伟,不如说是氛围。 看看亚洲正在建设的“森林城市”,虽然我对它们了解不够,无法做出判断。

    如果东亚能够使技术人性化并引入新的情感,我想我们都可以庆祝。 除此之外,他们将做出哪些发明、出于什么目的以及如何将其融入生活? 我想知道。 即使什么都没有,人性化的技术就足够了。

    • 回复: @Talha
  321. Talha 说:
    @AaronB

    如果东亚能够使技术人性化并引入新的情感,我想我们都可以庆祝。

    绝对同意那里。 也许我应该对此更加乐观。 毕竟,他们最近才开始利用超级技术的概念展开翅膀,因此他们复制一些已经飞行了一段时间的技术也就不足为奇了。

    也许一旦他们飞行了一段时间,他们就会想出自己独特的飞行模式*。

    和平:

    *注:刚刚谈论飞行让我想起了我是多么喜欢东亚文化制作那些精致而华丽的风筝的美丽方式。

  322. TT 说:
    @Talha

    您可以通过搜索找到更多案例。 谷歌似乎变得最糟糕,现在经常抛出不相关的垃圾。

    这位剑桥物理学家表示,韦斯特不想承认这种令人震惊的现象,因为这意味着西方的很多研究和理论都被揭穿了。

    人们仍然更喜欢讨论大脑大小和神经学方面来量化智商。 但医学证据表明,人类的正常生活并不需要大脑。

    而且在很多情况下,一些大脑尚未完全发育的小孩已经表现出了很高的能力或天赋。 例如。 数学家陶哲轩。 头看起来不大。 爱因斯坦的大脑比正常人要小。

    https://pumpkinperson.com/2015/05/28/einsteins-brain-size/amp/

    http://www-history.mcs.st-andrews.ac.uk/Biographies/Tao.html

    当特里 两岁 他的父母……看到他教五岁的孩子拼写和添加数字,……他回答说他一直在电视上看《芝麻街》。

    更多的情况是,有些人不用训练就能像孩子一样自然地弹钢琴,就像经验丰富的专家一样。 许多人记得他们的前世……

    https://m.theepochtimes.com/10-outstanding-cases-of-detailed-verified-past-life-memories_1778495.html

    古经记载了如何通过训练心日复一日、年复一年、生生世世地回想过去的生活。 就我个人而言,有相当多的朋友可以通过冥想练习来做到这一点,他们看到许多前世,直到世界循环毁灭和重建。 但不要要求我提供证据,只是我的分享,那些不信的人请忽略,不需要发表恶毒的评论。 谢谢。

    • 回复: @Talha
  323. Talha 说:
    @TT

    这位剑桥物理学家表示,韦斯特不想承认这种令人震惊的现象,因为这意味着西方的很多研究和理论都被揭穿了。

    正确,因为科学主义有它自己不能放弃的教条,因为这意味着科学家(我们后现代时代的神谕)无法解决所有问题。 震撼中的震撼!

    像我和你这样的人不介意一个包含神秘甚至一点——喘息——魔法的世界! 但某些人一想到有些事情超出了他们全面理解的能力,就会感到非常害怕……

    http://www.bbc.com/earth/story/20140916-mystery-fairy-circles-defy-explanation

    和平:

  324. TT 说:
    @Daniel Chieh

    对于驱逐国——尤其是捷克斯洛伐克和波兰——来说,对德语民众使用恐怖手段不仅是为了报复他们在战争中所受的伤害,而且也是引发跨境大规模踩踏事件并最终实现其政府目标的一种手段。战前建立种族同质的民族国家的野心。

    https://www.chronicle.com/article/The-European-Atrocity-You/132123

    国民党对中国人民和台湾原住民进行了大规模暴行(白色恐怖),但他们没有奴隶。 引用奴隶作为借口太愚蠢了。

    捷克有自己的故事要讲。 欧洲人在争夺主导地位时互相犯下暴行,美国和英国是幕后杰出的伪君子和主宰者。 一旦安顿下来,他们就向世界发起进攻,直到这一刻。

    美国北约仍在制造战争和破坏,你需要更好的理由来盲目捍卫白人。 他们通过沉默选举了他们的领导人,纵容和批准了他们的行动。 我并不是说所有白人都是坏人,这只是他们大多数人行为的概括。

    • 回复: @Daniel Chieh
  325. @TT

    我是国民党的直系后裔。 是的,确实存在腐败,但台湾的财富和繁荣很大程度上来自国民党的决定和必要行动。

    我不会盲目地为任何人辩护; 我只是认为将暴行归咎于一个种族是不准确的,因为从很多方面来看,错误在于人类而不是任何特定的人类群体。 当然,白人对现代世界的大部分负责——我认为这一点不可否认,在一段时间内,他们拥有类似于绝对权力的东西,但有些人滥用了它。 正如人们所说,绝对权力导致腐败; 我不确定另一个种族是否也会被绝对权力所腐蚀。

    是的,中国人的种族灭绝相对较少,我认为儒家文化是其中的重要组成部分。 但举例来说,郑成功也没有大规模处决大量投降的荷兰男子,并强迫妇女纳妾。 清满族对准噶尔人进行了种族灭绝。 蒙古人在征服世界的过程中屠杀了数百万人。

    谋杀和种族灭绝并不是“白人”独有的属性。

    • 同意: Talha
    • 回复: @TT
  326. TT 说:

    清满族对准噶尔人进行了种族灭绝。 蒙古人在征服世界的过程中屠杀了数百万人。

    是的,满洲人和蒙古人屠杀了汉人并入侵了其他人。

    那么,你们因为指出清朝马楚和蒙古的事迹而攻击和妖魔化中国人,这是一个“可悲”的案例吗?

    可悲的是,白人在西方确实受到攻击和妖魔化,即使这是他们释放力量甚至他们自己文化的结果。

    为什么一个人应该是“伤心”如果其他人只是指出大多数白人(美国和北约战争贩子)的错误,他们在过去的几个世纪里直到现在都给世界带来了苦难,没有喘息的机会,正如亚伦认为的那样,他们有着顽固不化的殖民文化? Yougov民意调查显示,超过40%的人仍然为他们的祖先残忍贪婪的殖民行为感到自豪。 如果有机会,他们会很高兴重温那段辉煌,不是吗? 去读一下梅首相在美国国会的演讲吧。

    它不应该因为直言不讳而被污蔑为对白人的集体攻击或妖魔化。 为什么我们要像美国一样,任何对犹太人或以色列的批评都是反犹太主义,或者因为罗恩是一个好人,在这里为我们提供了一个平台。

    白人*仍在中东、亚洲、东欧和非洲发动战争和颠覆,在文明的现代世界屠杀数百万人,却没有受到严厉谴责和惩罚。 正如战犯奥尔布赖特所说,一百万伊拉克人的生命值得以鲜血换取石油。

    像布什、奥巴马、布莱尔、萨科斯基、基辛格、克林顿这样的战犯……都逍遥法外,而国际刑事法院却对棕色人和黑人以及可怜的塞尔维亚人收费(哦,塞尔维亚人是“白人”,所以国际刑事法院到底是公平的吗?)。 而在美国北约的强烈威胁下,俄罗斯人也是“白人”,那么白人到底是受害者吗? 有些白人更白。

    其确实非常 悲伤 没有重要的白人愿意起来反抗所有这些战争罪行,推翻这些战争贩子。 但许多人为他们欢呼,为伊朗和朝鲜的另一场种族灭绝而死,甚至在乌兹别克斯坦也是如此。

    现在有人甚至将恶魔称为恶魔,这是一个“可悲”的案例,因为捷克人没有机会拥有奴隶。

    *美国、英国、北约、加拿大、澳大利亚(新西兰)、以色列的所有白人,加上瑞典人等斯堪的纳维亚人以及一些东欧人,压倒性地代表了整个白人种族。

  327. TT 说:
    @Daniel Chieh

    国民党的例子只是为了表明作为一个汉派,我准备指出它的黑历史。 这应该让那些因妖魔化中国人而受到指责的人感到悲伤吗? 国民党后裔也许是,但不要污名化。

    国民党正在对抗一种意识形态,决不放弃中国。 确实,它使台湾繁荣。 但美国/日本的傀儡绿党民进党正在通过分裂国家来摧毁台湾,以人民的危险来满足美国的愿望。 这句话固然让民进党的支持者感到难过,但却妖魔化了华人?

    1962年,台湾的人均国民生产总值(GNP)为170美元,该岛的经济水平介于扎伊尔和刚果之间。 到2008年,按购买力平价(PPP)调整后,台湾的人均国民生产总值已飙升至33,000美元(2008年估计),人类发展指数与其他发达国家相当。

    • 回复: @Daniel Chieh
  328. @TT

    没有什么能让绿党感到悲伤,他们推动不可能的分裂,同时又没有战斗的意愿。 这完全是令人费解的。 他们生活在自己的宇宙中,你允许的同性婚姻越多,你就会变得越强大?

    • 回复: @TT
    , @TT
  329. TT 说:
    @Daniel Chieh

    格林总理蔡永伟很可能是由美国控制的。 但选举她和绿党的大多数台湾人反映了他们的观点——分裂。 LGBT是美国游说者的一项政策,让每个人都陷入困境,孢子政府也在提倡LGBT,即使仍然有刑法/法律将鸡奸定为犯罪,多么讽刺。

    我认识的一位游历广泛的台湾退休大学教授(非政治专业)怀有这样的想法:中国领导人是无可救药的残酷共产主义者,因此分裂是毫无疑问的。 而日本就是这样仁慈的殖民主子,一定会以优势的军队保卫和打败中国。

    他坚称国民党作为叛徒摧毁了台湾经济,耗尽了储备并增加了债务,特别是在马英九领导下。 Green Chen SB & Cai YH接任后,台湾持续空前发展,许多好政策惠及民众。

    一个教授都有如此坚定的信念,何况是无知的普通人。

    顺便说一句,他的观点是否真实并反映了大多数人的观点? 这听起来好得让我难以置信。

    • 回复: @Daniel Chieh
  330. @TT

    我认识的一位游历广泛的台湾退休大学教授(非政治专业)怀有这样的想法:中国领导人是无可救药的残酷共产主义者,因此分裂是毫无疑问的。 而日本就是这样仁慈的殖民主子,一定会以优势的军队保卫和打败中国。

    啊哈哈哈。 是的,不幸的是,我在台湾认识一个有类似信仰的人,尽管他对仁慈的日本大师的成功更为悲观。 但他非常渴望成为一名日本殖民者。

    多数? 不,我不这么认为。 显然,总体而言,它的表现并不那么搞笑,蔡英文总统在台湾实际上不如习近平受欢迎(我认识的人只是指责台湾是共产党渗透者!)。

    我想说,整体情绪真的很绝望。 绝望导致人们产生各种怪异和疯狂的信仰,其中最有毒、最可悲和最搞笑的可能是乞求成为日本一部分的愿望。

    你知道,我隐约能理解,我相信你也看到了。 你去日本,一切都很美好,空气干净,水安全,女孩漂亮又有礼貌。 而在中国,雾霾、粗鲁和开水,护士对你大喊要快点,医院里一切都很公事公办。 然后你会想,好吧,他们是像我们一样的东亚人,我们有什么问题吗? 为什么我们不能都是日本人?

    对我来说,答案是日本就像一个活生生的博物馆,本质上都是假人。 仪式太多,规则太多,真正的希望却那么少; 如果你是一个年轻人,你基本上希望勾选所有正确的选项,进入正确的公司,并按照某种脚本生活和死亡。 我相信很多中国人会对此感到高兴,但也有很多人不会高兴——他们有更多的野心,更多的对冒险的热爱,以及更多的热爱,嗯,冲突和挣扎。

    你知道,在《三国演义》中,我并不像大多数人那样认同诸葛亮。 我一直认同周宇,因为如果你不能在生活中做你能做的每一件事,取得相当成功,然后某个拿着鹤羽扇的漂亮男孩就会在所有事情上打败你,那该死的。

    • 回复: @TT
  331. Factorize 说:

    res,你看到这个了吗? 哇!

    这是我见过的全球人工智能思维最好的 MRI。

    中国/东亚海岸的整个互联网网络在太阳之夜之前点亮。 晚上每个人都会关闭电脑吗? 感人的!

    我在印度和欧洲所看到的也给我留下了深刻的印象。 欧洲大部分地区的互联网流量达到最大; 当中国全面提高互联网普及率时,它将成为全球市场的领导者。

    另外,是的,墨西哥和南美洲海岸! 继续好好冲浪吧! 桑托库尔托干得好。

    我对美国的情况有些失望,尽管至少有相当多的人让网络火炬 24/7 持续燃烧。

    非洲是另一个似乎适用“可以改进”的地区。 判断非洲太阳能互联网白天或夜晚何时开始并不容易。 怎么网站流量这么少?

    您可以根据互联网流量(如下图所示)等统计数据构建“现实生活中的国民智商”分数,该分数可能与“国民智商”高度相关(特别是使用完整的智商 SNP 集)。

    • 回复: @res
    , @Santoculto
  332. TT 说:
    @Daniel Chieh

    你听说过没有大脑的人还能活着吗? 所以即使大脑也没关系。

    剑桥大学的一位物理学家曾经告诉我,在世界大战期间,有记录表明,当士兵的头皮被炸开时,有些人里面实际上没有大脑。

    时至今日,仍有一些人没有大脑而活着,他们生活正常,但到医院检查后才发现。

    您能就神经病学方面的问题给我您的意见吗? 谢谢。

    • 回复: @Daniel Chieh
  333. TT 说:
    @Daniel Chieh

    是的,中国人开玩笑说蔡英文和特朗普是共产党渗透者,是从天而降的礼物。

    你去日本,一切都很美好,空气干净,水安全,女孩漂亮又有礼貌。 而在中国,雾霾、粗鲁和开水,护士对你大喊要快点,医院里一切都很公事公办。

    当我在日本工作时,我可以看到阴暗面。 夜幕降临,许多无家可归者涌入火车站,睡在纸箱里过冬。 老人们在垃圾箱里翻找食物,或者因为羞于乞讨而经常被抓到商店里行窃。

    我很惊讶没有看到垃圾箱,街道却如此干净。 我的太平绅士同事笑了,不不,他们乱扔垃圾,我尝试扔垃圾,不知从哪里跳出来一个老清洁工把它扫了。 到处都是那么干净、新鲜、高效,但是人工冷。 但仍然令人印象深刻的好国家。

    女孩们大多都不漂亮,这是一个骗局,让我很难过,只有在 AV 中才能找到辣妹! 他们有萝卜腿……韩国也一样,我被骗了,它的整形手术和 Photoshop 奇迹。

    中国,哈哈哈,没有什么比广州的肮脏、香港的粗鲁、澳门的犯罪更胜一筹了。 中国医院还喊叫吗? 当我十年前访问时,这是相当高的标准。 但Dr的水平很烂,他们想通过注射草药提取物来用我作为实验小鼠,但无法回答我对其纯度的询问。

    对我来说,答案是日本就像一个活生生的博物馆,本质上都是假人。 仪式太多,规则太多,真正的希望却那么少; 如果你是一个年轻人,

    可悲的是,这也是我所看到的,只要给他们买足够的清酒,他们就会开始哭泣,讲述他们不为人知的故事……一个悲伤的社会,在一个难以忍受的高压锅里,无论年轻人还是老年人都死气沉沉、绝望。 清酒是他们的哀悼。

    毫无疑问,中国是现在最有趣的居住地。 我哥哥非常喜欢,整个新年都在探望他的工人远方的家,FSR给他留下了深刻的印象。 这是一个永远更新的国家。

    而那个拿着仙鹤羽扇的美少年哈哈,却因为逆天意付出了生命的代价。 曹操可以统一三个王国,免遭战争,没有 ZG,一个没有智慧的天才。

  334. @TT

    需要与证据联系起来,尽管我认为大脑的大部分用于技能发展,只有脑干是绝对需要的。

    • 回复: @TT
  335. TT 说:
    @Daniel Chieh

    这位剑桥物理学家表示,韦斯特不想承认这种令人震惊的现象,因为这意味着西方的很多研究和理论都被揭穿了。 许多未记录的案件。

    http://mysteriousuniverse.org/2017/07/miraculous-cases-of-people-who-lived-without-a-brain/

    http://www.dailymail.co.uk/femail/article-3293259/Baby-born-without-brain-amazes-doctors-celebrating-second-birthday-says-Mummy-time.html

    而且在很多情况下,一些大脑尚未完全发育的小孩已经表现出了很高的能力或天赋。 例如。 数学家陶哲轩。 头看起来不大。 爱因斯坦的大脑比正常人要小。

    https://pumpkinperson.com/2015/05/28/einsteins-brain-size/amp/

    http://www-history.mcs.st-andrews.ac.uk/Biographies/Tao.html

    当特里两岁时,他的父母……看到他教五岁的孩子拼写和添加数字,……他回答说他一直在电视上看《芝麻街》。

    更多的情况是,有些人不用训练就能像孩子一样自然地弹钢琴,就像经验丰富的专家一样。 许多人记得他们的前世……

    https://m.theepochtimes.com/10-outstanding-cases-of-detailed-verified-past-life-memories_1778495.html

    • 回复: @Daniel Chieh
  336. res 说:
    @Factorize

    有趣的。 谢谢。

    有关该图形的更多信息: https://web.archive.org/web/20140606062629/http://internetcensus2012.bitbucket.org/paper.html

    关于这样的图形有一件事。 您确实需要对人口密度进行标准化。 否则,这就是主要传达的内容。

    • 回复: @Factorize
  337. GammaRay 说:
    @Santoculto

    实际上,我们知道这基本上是不真实的,因为白人过着非常舒适的生活,并在他们的祖先从土著人民手中夺取的土地上创建了非常成功的国家

    错误的。 即使没有从其他地方提取的自然资源,白人也能够在没有工业化/和人口爆炸的情况下创造一个美好的社会。

    不,你错了,如果你看看人口蓬勃发展的全球英语圈,这一点是不言而喻的。 美洲原住民、原住民等曾经居住的地方,现在有数百万白人(同时这些地方的土著人民已成为极少数)。 数字不会说谎。 你试图(非常不诚实地)声称,从历史上看,白人无产阶级和其他人一样是受害者,而欧洲精英才是真正的罪魁祸首,但老实说,这种推理过于还原论和缺乏说服力。 曾经占领北美的土著人民(在白人手中)的处境比白人无产阶级从他们自己的精英那里得到的处境要糟糕得多。 令我困扰的是,你如此努力地试图将手头的真正问题混为一谈,而不是简单地直言不讳。 是的,许多白人被他们自己的精英利用和压迫; 但这并不能改变这样一个事实:从宏观角度来看,白人是全世界土著人民的全球压迫者。 白人被精英阶层驱使以某种方式行事这一事实与白人也从这种安排中获得物质利益这一事实并不相互排斥。 白人从这种安排中以多种方式获得物质利益,这一事实有力地反对了你的“白人无产阶级也是平等受害者”的立场。

    你基本上是一个白人民族主义者,试图煽动并提出(不令人信服的)道歉论点,试图通过表现得好像白人无产阶级与他们所征服的土著人民一样遭受平等的受害者,从而缓和西方殖民主义的残酷现实。流离失所。 那就是有钱啊!

    再说一遍,大多数人都是被驯化的。 责备狗因“主人”的虐待狂而杀死狐狸仍然不是理想的正确推理。

    使用这种站不住脚的推理基本上可以免除除精英之外的所有人的任何道德代理和责任。 这对你来说听起来明智吗?

    “原住民”在新世界中并不总是和谐的民族群体。 事实上,白人入侵者所做的事情总是发生在美洲。 是的。 这是不可原谅的。 你在这里言论的主要问题似乎是,你只是指责白人有肮脏的过去或忽视他们,即使这不是完全错误,也要给予特别考虑,因为“白人错误”已经全球化,现在包括中国在内的每个人,有机会效仿他们。

    我要揭穿 WN/另类右派总是喜欢用你们自己的逻辑来提出的这种推理。 我将其称为“两个错误即一个正确”谬论。 基本上,你试图证明欧洲人殖民北美并杀害当地人是有道理的,因为当地人已经互相残杀了。 好,可以:

    这也必然意味着西方的移民犯罪也是情有可原的,因为白人总是对其他白人做卑鄙的事情。 有道理不是吗?

    为什么要对黑人男性和白人女性之间的跨种族关系感到愤怒呢? 白人男性已经一直在操白人女性了,那么如果黑人男性也这样做的话,会有什么不同呢?

    我想这也意味着,如果未来欧洲列强之间发生一些大的争执(也许是欧洲内战、民族主义者与全球主义者/欧盟支持者?),那么中国入侵欧洲理论上是合理的。 我的意思是,欧洲人无论如何都在互相争斗; 如果中国加入战局并与他们全部作战又有什么关系呢?

    希望你能看到这个常见的 WN/另类右翼谬误中的漏洞,并理解为什么它听起来如此空洞。 如果你认为我“挑出”白人来殖民整个世界,那你也是妄想(典型的WN试图扮演受害者。“WN在打你时痛苦地哭泣”)。 我所做的唯一一件事就是(正确地)指出西方文化既有优点也有缺点,其中一个缺点是白人似乎有相当强烈的大规模侵略倾向和强烈的想要侵略的倾向。支配、征服和控制他人。 WN/另类右翼人士毫无问题(准确地)挑出其他种族的某些倾向和负面偏好,所以当同样的事情发生在你身上时,不要哭泣。

  338. GammaRay 说:
    @Santoculto

    是的,中国可能成为全球霸主。 但与历史上白人的情况完全不同。 中国人根本没有能力去积极地监管他人,也没有能力要求人们在文化和精神上完全服从他们想强加给他人的任何标准。 如果有一天中国人最终统治世界,那将是一种被动的方式(向任何人致敬?),而不是公开的。 如果你真的认为中国人会在他们的祖国对不同的种族进行屠杀和种族灭绝,并迫使其余的人开始说中文,那么你就是在妄想。 是否可以? 是的。 有可能吗? 不。就华人而言,历史上或文化上没有任何事情可以与白人所做的事情相媲美。 白人通常性格外向,因此他们将精力集中在外部,而中国人通常性格内向,因此他们倾向于将精力集中在内部。 如果有的话,中国人最大的受害者就是他们自己(内战、革命等)。

    只是为了让您和任何阅读的人清楚地了解。 我不是中国的辩护者,我发现中国文化中有很多愚蠢和愚蠢的事情; 话虽这么说,我在这里同样应用历史、文化和 HBD 推理,但在试图证明中国人将突然变得像白人一样,并试图在身体和文化上殖民每个人的奇怪论点时,我的表现有些不足。 。 抱歉,但这基本上是一种非常蹩脚的投影形式。 此外,如果你对中国文化或中国人有所了解,那么你就会知道这听起来有多么荒谬。 黑人会突然开始集体演奏莫扎特音乐和古典乐器吗? 你已经知道答案了,同样,这种情况发生的可能性与中国人实际上会不遗余力地以种族灭绝或强行改变文化的方式与人们搞砸的可能性是一样的。 这根本不是它们的接线方式。 如果中国人处于权力地位,他们就会要求尊重、顺从和贡品。 就是这样。 在这方面,他们非常直率、务实,没有任何多余的装饰。 白色的“力量”的功能就像动能,它猛烈地向外冲击并创造变化,中国的“力量”的功能就像重力,它轻轻地将你向内拉入它的轨道,差别很大。

    你试图解决一个问题,却制造了其他问题,并导致了不公正。

    这里唯一的不公平是标准的应用不均匀。 如果可以冷静地批评黑人、穆斯林、犹太人、亚洲人等,那么这意味着白人也是公平的游戏。 就我而言,这里没有人受到特殊待遇。 要么我们对每个人都政治不正确,要么我们对任何人都没有政治不正确。

    白人得到了好处,首先,是一个非常简单的叙述。 今天,就在最近,因为生活和开发新土地无疑是一项非常困难的工作,特别是对于普通白人来说,而且他们再次被剥削,过着悲惨的生活,特别是工人阶级。

    你认为白人工人阶级过得不好吗? 你认为美洲原住民的感受如何? 哎哟!

    我想如果你对你的人民和政府也这样做的话。 我不是。 我试图以中立的方式分析这个复杂的东西,我赞成赔偿,批判性分析,不仅仅是白人文化和历史,而是所有人类文化和历史,但是,有些事实对我们的感受漠不关心。 建立价值观固然重要,但首先把握事实也很重要。

    我觉得你并不中立; 这在你尝试构造的论证类型中非常明显。

    这种种族和行为现实对于真正结束一连串愚蠢的人类冲突非常重要。

    是的,我同意这一点。 让我们冷静而诚实地考虑不同种族的行为现实,而不是试图挑选出令人满意和不受欢迎的特征

    好吧,我理解你关于 WN/另类右翼虚伪的观点,这种虚伪在大多数部落民族中都很流行,无论是关于国家、种族还是意识形态。 我也攻击了这种虚伪,实际上我对所有这些废话感到厌倦。

    仅仅因为虚伪在部落民族中盛行,并不意味着 WN/另类右翼分子就可以摆脱困境,并且在他们这样做时不会让它变得“好”,这就是你似乎试图低调暗示的

  339. GammaRay 说:
    @Santoculto

    再说一遍,大多数人,特别是文明地方的人,都是被驯化的。 它们已被选中来被驯服。 当然有: 驯化程度; 驯化的类型。 大多数白人都是驯养的,智力不那么聪明。 是的,他们是现代环境的根本受害者,因为他们被指控犯有其大陆/欧洲之外的所有全球化西方错误,并被用作牺牲品[其所有文化都已被完全破坏,其原始土地,最重要的是,其物质安全正处于日益危险之中],就好像西班牙人民决定开发大西洋以外的新土地……这完全是一个精英计划,通常都是如此。 我们可以说,人们要么有罪,要么没有罪。 他们有罪,因为他们对自己的精英感到自满[就像狗对主人对狐狸的态度感到自满一样]。 但因为我们并不都是一样的,完全有能力达到理性,所以我们不能在指责精英之前责备人们,特别是当人们被驯化时。

    哦,当然,我怎么会犯这样的错误。 这些可怜的小西班牙征服者被精英阶层误导,他们前往新世界不仅仅是为了获得财富、当地妇女的后宫和自由土地。 你们错误地相信了错误的二分法,即必须责怪人民或精英; 事实上,有时人民和精英都应该受到指责。

    白人所发生的事情是不公正的,这与黑人在美洲(以及非洲)被奴役时所发生的事情是一样的。 我反对一切形式的不公正,这就是我的理由。

    不,更准确的说法是,发生在白人身上的事情是意外后果法则。 如果发生不公正现象; 那么我们应该责怪白人的祖先(欧洲殖民者以及所有直接和间接促成了欧洲对世界的殖民化这一多世纪现象的人)造成了这种情况。 前几代白人造成了这种混乱,而当代白人将不得不处理这个问题。

    其次,白人身上发生的事情真的被认为是不公正吗? 许多白人都接受多元文化主义和移民。 许多白人甚至与非白人结婚生子。 你认为这些人会认为目前的事态“不公正”吗? 许多其他白人完全漠不关心,也不关心。 您认为他们会认为目前的事态“不公正”吗? 在当前背景下,不公正是一个非常模糊的术语。 目前的事态只有另类右翼人士明确认为是不公正的。 但对其他人来说,现实情况要主观得多。

    你没有/没有回答关于犹太人的问题......

    代表我的疏忽,我的答案如下:

    因为很多人都陷入了绝望。 好吧,根据这个逻辑,你一定对犹太人也感到愤怒或漠不关心,不是吗?**
    更重要的是,因为他们与白人一起参与了其他国家的大多数“白人”犯罪。

    在我看来,你基本上是在试图将责任从白人身上转移到犹太人身上。 你本质上所做的事情被称为转移注意力。 这是我对此的看法。 从文化意义上来说,无论有或没有犹太人的参与,白人通常都是专横和好斗的。 一个完美的例子就是美国军队。 当然,最高层可能有犹太复国主义犹太人,他们为了以色列的利益而让美国卷入中东战争,但当谈到美国军队的普通士兵时,也有很多白人他们迫切地想要去杀戮并征服世界,他们是一毛钱一打。 事实上,有些犹太人可能为了自己的邪恶目的而操纵这些白人,但这并不能否认许多白人本身(没有犹太人的操纵)似乎天生就被侵略和征服所吸引。

    我认为 WN/另类右翼有时喜欢表现得好像白人不是真正的坏人,而只是犹太人让白人做坏事; 但从与 WN/另类右翼分子的互动以及他们的言行来看,很明显,无论有或没有犹太人,白人都可以自己做坏事(例如考虑一下乔·韦伯的扭曲言论) 。 正如我之前所说; 白人具有一定的活力和神气,有利于大型进取、雄心勃勃的企业; 犹太人似乎偶尔会有效地利用这种能量来谋取自己的利益,但这并不能神奇地免除白人自己的罪行。

    有趣的是,你试图痛斥我指责白人作为一个群体,但随后你又试图引诱我指责犹太人作为一个群体!

    • 回复: @Santoculto
    , @Santoculto
  340. GammaRay 说:
    @AaronB

    从这个意义上讲,我认为你关于中国人如何对待美洲原住民的想法有点太天真了。 是的,他们可能不会以这种方式屠杀和掠夺他们,但会有某种剥削。 肯定会更温和,但不是平等的关系。

    严肃的问题; 你真的了解中国历史,甚至了解中国人吗? 中国人历来对中国以外的事物不感兴趣。 因此,他们甚至不会关心中国以外的人来剥削他们。 事实上,如果西方没有“唤醒”他们,中国人很可能还在长眠之中。 否则你认为明朝为什么会做出自愿闭关自守的决定呢? 我并不是出于任何情感、个人原因来捍卫中国人;我只是为了保护中国人。 我只是(正确地)指出,由于历史、文化和HBD的原因,中国人一般不是那种公然剥削别人的人。 如果有的话,他们喜欢互相剥削。 从我所读到的关于美洲印第安人和中国文化之间某种前哥伦布时代接触的证据来看(诚然很少),这似乎是一种和平且互利的接触,围绕着商业和文化交易,而不是征服者/被征服者。动态的。 我们观点之间的差异在于,你相信(没有任何强有力的证据或论证来支持这一信念)中国人肯定会以某种方式剥削美洲印第安人; 虽然我认为这是一种可能性,但总的来说,鉴于其他相关信息否定了这种可能性,这种可能性不大。

    同样重要的是,你认为所有美洲印第安人的文化都是原始的、无组织的,并且完全无法抵制剥削它们的企图。 通过这样做,你试图消除美洲原住民的代理权,以支持你的主张,即中国人“肯定”会以某种方式剥削美洲印第安人。 然而我们知道,这显然是不真实的,因为不言而喻,有许多复杂且相对强大的美洲印第安文明能够以或多或少平等的方式与中国人打交道。 正如我上面所说,你可能故意忽略这个事实,以加强你自己的情况

    可悲的是,白人在西方确实受到攻击和妖魔化,即使这是他们释放力量甚至他们自己文化的结果。

    我认为一些白人为复兴旧文化做出最后的努力是正常的,但这些事情永远不会奏效。 不过,你也不能责怪他们。 对他们来说,指出白人文化的消极方面并没有帮助,因为白人文化创造了当前的条件。 他们已经被妖魔化了,他们也想要一个新的神话。 所有的权力争夺都需要自我验证的神话。

    我觉得一点也不悲伤。 这是一个自然循环、因果关系。 白人或多或少都是自己造成的,所以当他们最终播种他们的收获时,我为什么要悲伤呢? 当连环杀人犯和强奸犯入狱时,你会感到难过吗? 我知道我不知道,而且这种情况也没有太大不同。 不要把我的冷漠误认为是对白人的仇恨或不喜欢; 恰恰相反。 我只是说,我不会试图冒险帮助保护人们免受他们自己造成的后果。 更重要的是,正如我一直说的; 大多数白人要么对当前的事态感到满意,要么完全漠不关心。 他们真的不在乎,那我为什么要在乎呢? 如果连大多数白人自己都不为此感到悲伤,这真的很悲伤吗? 这不是代表我的小言辞,我提出这个问题的时候是非常严肃的。 另类右翼需要明白,他们并不代表大多数白人,而事实是,大多数白人并不真正关心这两种情况。

    指出白人文化造成当前状况的消极方面绝对有帮助。 有必要进行准确、理性的批评,以便人们正确调整世界观,并从过去的错误中吸取教训。 WN/另类右翼已经充满了妄想和他们告诉自己的半真半假的事实; 他们与现实的脱节对他们没有任何好处。 我不会只是为了让他们自我感觉更好而溺爱他们。 我只关心在这里进行真实、合乎逻辑的讨论; 我无意试图缓和问题以免伤害感情,但相反,我也无意试图无理地攻击白人或给予他们不应有的批评。 我认为从我之前的评论(你可以自由检查)中可以清楚地看出,我对白人的立场非常平衡和合理。 你似乎在暗示我只是非理性地试图堆积白人,这是不诚实的

    在白人的超意识中正在出现一些新的东西,但这是有机地发生的,实际上是不利条件的结果。 具有讽刺意味的是,通过我对白人文化做出公正而准确的批评,它正在帮助实现你自己渴望的这种趋势。 只有拒绝批评(从而鼓励妄想),我才会阻碍这一进步,并人为地保持过时和过时的世界观。 那么……你想要哪种方式? 已经决定了

    • 回复: @AaronB
    , @Factorize
  341. AaronB 说:
    @GammaRay

    嗯,你看起来比我想象的更有侵略性,是个激进的中国民族主义者,而且我不太有兴趣与你激烈争论。 我只想说,我认为您将中国与外国人的关系描述为非剥削性的,是基于对历史的高度选择性的解读和对事实的创造性解释。

    不过,我还是要指出几点。 中国从不关注技术,但现在却痴迷于技术。 对技术的痴迷是对传统的深刻决裂。 中国传统上培养了平衡和放松的人,但现在似乎培养了大量不稳定、具有高度攻击性的强迫症患者。 因此,中国显然因与西方的痛苦遭遇而发生了深刻的改变。

    目前尚不清楚这种情况有多严重。 我个人持谨慎乐观的态度,认为中国有足够多的优秀古老文化完好无损,中国世纪将为人类造福,但我并不完全确定。

    我的观点是,你关于中国传统文化是未来中国人行为的安全指南的论点(即使你弱化了传统文化)对我来说似乎没有那么有说服力——存在着根本的不连续性、断裂,而且还不清楚如何这很深。

    现代西方本身就是与其传统文化彻底断裂的产物——这一过程可能会在中国重演。

    至于对白人的同情,你会将多代白人视为一个单一的有机体,这是一个有点有趣但完全由情感驱动的抽象实体的具体化,这样你就可以发泄你的不满。 “白人”是一个抽象概念——那些对你们的文明造成如此伤害的特定白人已经不复存在。 你不能再伤害他们了。

    将巨大的历史进程个人化,并指责普通白人农民陷入了他们无法控制的进程中,这似乎是不成熟和不明智的,除非你主要对情感满足和培养狭隘的自私的世界观感兴趣。

    它很可能会回来困扰你。 目前,白人组织混乱、疲惫不堪——这是历史周期的自然转变。 如果你的傲慢太过了,你可能会发现自己面临着一个重新焕发活力的敌人,这对你来说太强大了。

    为什么不打破这个循环呢? 如今,欧洲人似乎比中国人好战得多。

    不管怎样,我想我弄错了你是谁,祝你好运,按照你的意愿去做。

    • 回复: @GammaRay
  342. Factorize 说:
    @res

    不过,你关于纠正人口密度的想法是正确的
    有时,查看鸟瞰图或在本例中为火星人的视角会很有帮助。

    然而,有时我担心,通过将一切正常化,现实会变得扭曲,
    只是以另一种方式。 未经校正的数据是现实生活的体验方式。

    我发现这张图的另一个有趣之处在于,您可以了解人类集体思维如何使用互联网。 在中国,直到互联网黄昏之前,整个海岸到大陆才会变成红色。 工人们从工厂回家,上网冲浪 unz.com.

    然而,在印度,你会注意到南方的大部分地区在当天早些时候变成了红色。 在这里,呼叫中心和其他信息技术中心实际上正在使用互联网作为其日常业务模式的一部分。 对我来说,这表明印度正在更好地将技术融入其知识经济。 令人惊讶的是,印度是唯一一个采用这种技术使用模式的主要互联网地区。

    在基于知识的技术经济中,互联网应该成为工作体验的核心特征。

  343. GammaRay 说:
    @AaronB

    如果您认为我对您的语气具有攻击性; 只是因为我饶有兴趣地回应了你那种居高临下、无所不知的语气。 冷静地提出论点并坚持事实,我们就能进行富有成效的对话。

    您错了。 我不是中国人,也不是中国民族主义者。 当你想间接攻击我的可信度,故意用人身攻击的方式称我为中国民族主义者,仅仅因为我使用了非常实际、合乎逻辑且没有任何意义的推理时,这充分说明了你自己的心态。 你说话就像一个WN/另类右翼人士,他对世界的所有看法和事实都是从互联网上的另类右翼回声室中获得的,结果只是重复他从其他人那里听到的内容,而没有任何实际的个人经验。他的意见基于。

    我再问你一次。 您广泛接触过真正的中国人吗? 除了另类右翼的陈词滥调之外,你对中国文化的了解是否还只是肤浅的了解? 我很抱歉打破了你的幻想,即一些神话般的“不稳定的、具有高度攻击性的强迫性怪人”中国超人将接管世界,但事实是,中国人作为一个整体是相当β和被动的。 唯一真正相信中国人有能力实施某种严厉的全球独裁统治的人是那些不知道中国人到底是什么样的人。 如果你熟悉中国人的真实面目,那么你就会意识到他们普遍缺乏社会地位和狡猾,无法胜任管理一个全球帝国。 简而言之,中国人不像西方人那样“有活力”,因此他们似乎缺乏积极“文明”他人的意志力和动力。 如果中国未来确实成为全球超级大国,与美国目前扮演的咄咄逼人的超级大国角色相比,它将是一个更加不干涉、更加客观的超级大国。

    人们总是想谈论中国威胁,但他们的看法总是基于对中国人和白人有多么不同的无知。 让我问你一些非常简单的问题:

    - 黑人执政的政府是什么样的? 上述政府的性质是否或多或少与您对现实生活中遇到的黑人的总体看法相关?

    -亚洲人民管理着什么样的政府? 上述政府的性质是否或多或少与您对现实生活中遇到的亚洲人的看法相关?

    -白人统治的政府是什么样的? 上述政府的性质是否或多或少与您对现实生活中遇到的白人的看法相关?

    WN/另类右派总是想煽动中国妖怪的幽灵,但似乎无法解释中国人会突然从哪里获得所有这些侵略性和活力。 如果他们在现实生活中遇到足够多的中国人,他们很快就会意识到与白人相比,中国人是多么低能和相对被动。 即使是相对好斗的中国人(我见过一些)也往往缺乏公平的智慧和社交狡猾,无法有效地利用他们的侵略性来获得社会统治地位。 话虽这么说,我无意在这里说中国人的坏话,我只是提及这些观察结果,因为它们与当前的主题无关。

    至于对白人的同情,你会将多代白人视为一个单一的有机体,这是一个有点有趣但完全由情感驱动的抽象实体的具体化,这样你就可以发泄你的不满。 “白人”是一个抽象概念——那些对你们的文明造成如此伤害的特定白人已经不复存在。 你不能再伤害他们了。

    将巨大的历史进程个人化,并指责普通白人农民陷入了他们无法控制的进程中,这似乎是不成熟和不明智的,除非你主要对情感满足和培养狭隘的自私的世界观感兴趣。

    哈哈,在这一点上你基本上是butthurt,在我的上一条评论中,我用一堆你无法反驳的论点来打击你,结果你跳过了它们,只是把所有这些东西写给了我。 仅仅因为您选择忽略我提出的观点并不意味着它们无效。 这只是意味着你无法反驳他们而已。

    再说一遍,我不得不重复自己多少次,这真是荒谬。 我对白人没有任何反对意见,一点也没有。 大多数白人通常都是好人,对他人没有恶意。 可笑的是,你试图将我之前的评论解释为对白人的某种仇恨和恶毒的谩骂。 任何阅读本文的人都可以自由阅读我的评论并得出自己的结论。 但我很确定他们会对我对白人缺乏真正的尖酸刻薄感到失望。 不管怎样,你基本上是想说我是反白人,只是因为我不愿意为西方国家辩护,反对他们自己造成的负面局面。 我保持中立还不够(这正是我的立场,冷静中立),为了不被你们认为是反白人,我必须积极发言来捍卫白人。 是啊,随便吧老兄。 你对我自己的心态和我对情况的看法的痴迷基本上只是你的一种防御机制,以帮助你自己对西方目前面临的惨淡现实感觉更好。 我可以肯定地告诉你; 无论你多么想(错误地)将某些心态投射到我身上,都不会改变当地的情况。 西方国家目前注定要失败,这是几个世纪以来的因果链终于开花结果的结果。 你或我对此的感受对结果几乎没有影响。

    在这个话题上,你不妨说,按照你的标准,大多数白人也是“反白人”。 我对白人命运的总体感受并不比一般白人对此事的感受差多少,这是我多次提到的事情,但出于某种原因你不想解决大象问题在房间里; 也就是说,大多数白人根本不在乎,或者他们实际上支持当前的趋势。 抱歉,你不是所有白人的代言人,大多数白人甚至不会接受你或互联网上其他随机的 WN/另类右翼人士作为他们的代言人。 我猜你正试图让我成为敌人,因为妖魔化我比妖魔化那些根本不同意你的想法的白人群众更容易。

    它很可能会回来困扰你。 目前,白人组织混乱、疲惫不堪——这是历史周期的自然转变。 如果你的傲慢太过了,你可能会发现自己面临着一个重新焕发活力的敌人,这对你来说太强大了。

    哈哈,谁在乎老兄。 我非常怀疑白人是否能够回归并以他们巅峰时期的方式统一,但即使他们做到了那又怎样呢? 你觉得我在乎吗? 如果它发生了,它就会发生。 我不会为此哭泣。 你一直想把我说成是一个怀着利斧的反白人,但你没有意识到我实际上对整个事情是相当公正的。 你只是生气了,因为我不会跳上白色的受害者列车,仅此而已

    • 回复: @AaronB
  344. AaronB 说:
    @GammaRay

    啊,是的,客观性——当然 🙂 我对此并不怀疑。 只是纯粹的中立。 也许是对真理的无私的学术兴趣?

    在我们戴的所有面具中,客观性一定是最平淡无奇的。 但是无所谓。

    是的,我与中国人打交道的经验很丰富,我见过很多比白人更有侵略性、权力意志更强的人。 目前,就是这样。 正如我的评论所示,我也不认为中国是某种妖怪。 细微差别,细微差别。 你对中国人的看法似乎与现实相去甚远,颇有些童话色彩。

    我并不是想“拯救”白人,只是从事我现在认为独特且令人不安的 SJW 式行为,并倡导一个和平与公平的世界。

    这是我的一种不良倾向,我不断地陷入这种倾向——我认为这是因为在美国度过了太多时间,并与自由派白人进行了社会融合。 这提醒我,我早就应该长期离开美国了。 我必须努力让这一切尽快实现。

    你的立场的主要弱点与困扰许多现代思维的弱点是一样的——你及时采取了行动,并使其变得普遍。 顺便说一句,这与本文作者所展示的弱点相同。

    我没有兴趣与你进行冗长的历史讨论——如果你对历史的阅读还没有让你相信,对于这个世界来说,没有什么比变化更基本的了,而且国家的特征正在经历巨大的变化,甚至完全逆转。短时间内,我的微弱努力很难削弱如此坚硬的精神铠甲。

    我同意白人国家正在收获他们在非个人的业力过程中所播下的种子——尽管与同情和内疚的问题无关。 我担心,如果中国不小心的话,可能会重蹈覆辙——我们是被敌人塑造的,长大后往往会变得和施虐者一样。

    至于大多数白人是否同意,我认为分歧比你说的要多得多——而且这个过程才刚刚开始。

    我心中的 SJW 感到遗憾的是,白人可能会遭受虐待,然后再次生气,这样的循环将会重复。

    无论如何,无论如何。 我不应该和你进行这种无用的讨论。

    这些天,我努力培养一种冷静的态度,接受这个世界的所有动荡的陌生和奇迹,而不是怀抱不可能实现的公平与和平的梦想,我现在意识到,这是西方进步错觉的残余痕迹。尽管我不断地反对它,但它还是以某种方式感染了我。

    我部分受到道教的启发——道教是我一直谈论的可怕的中国怪物。

    我会让你回到你高度客观、充满激情、完全中立的党派主张,这是由你完全不关心发生什么的态度决定的,导致你在这个主题上写下热情洋溢的长篇大论。

    我得去制定长假计划了。

    • 回复: @GammaRay
  345. GammaRay 说:

    是的,我与中国人打交道的经验很丰富,我见过很多比白人更有侵略性、权力意志更强的人。 目前,就是这样。 正如我的评论所示,我也不认为中国是某种妖怪。 细微差别,细微差别。 你对中国人的看法似乎与现实相去甚远,颇有些童话色彩。

    抱歉,你在 r/hapa 或 r/亚洲男子气概上遇到的中国涡轮自闭症匿名者不算数 😉

    中国人的权力意志强吗? 绝对地。 然而,这最终会导致他们对整个种族进行种族灭绝并迫使每个人学习普通话吗? 我非常怀疑这一点。 许多中国人希望中国成为超级大国; 但我怀疑许多中国人是否有兴趣种族灭绝整个种族并取代其余的人/奴役他们。 这确实是我们争论的症结所在。 你似乎坚持认为中国人可能会像历史上的白人一样贪婪、贪婪和残忍,我不同意这个观点。 如果说有什么不同的话,那就是绝大多数中国人宁愿让人们出于钦佩而仰望他们,而不是出于征服的立场来仰望他们。 拿枪逼着人家给你面子,这对“面子”不太好; 有点违背了目的。

    在我看来,你想推测中国可能成为一个残酷的、剥削性的超级大国的唯一原因很简单,你想为白人过去的不良行为开脱。 我觉得这对你来说确实或多或少是一种自我的问题。

    你的立场的主要弱点与困扰许多现代思维的弱点是一样的——你及时采取了行动,并使其变得普遍。 顺便说一句,这与本文作者所展示的弱点相同。

    错误的。 人和文化随着时间的推移而变化,这是事实。 然而,人和文化并没有完全改变,这是你的范式的问题。 你相信一切事物都是可变的,并且以循环的形式存在,一切事物都同样受到影响。 然而实际上,人、文化、遗传等只是部分可变的,而它们所面临的情况则填补了图片的其余部分。 所以你的基本范式只是半真半假

    我没有兴趣与你进行冗长的历史讨论——如果你对历史的阅读还没有让你相信,对于这个世界来说,没有什么比变化更基本的了,而且国家的特征正在经历巨大的变化,甚至完全逆转。短时间内,我的微弱努力很难削弱如此坚硬的精神铠甲。

    是的,改变是世界上最基本的事情。 西方现在正在发生变化,既然变化是不可避免的,你为什么还要抗议呢? 西方处于文明增长和发展的原始状态,持续了大约 500 年左右,并且很大程度上是建立在世界各地被征服的土著人民和文化的基础上的; 现在这个模式不再可持续,看起来整座大厦即将倒塌。 你试图暗示白人将集体团结起来扭转这一发展,但如果你真的相信变革的必然性和重要性,那么你为什么不接受当前的事态呢?

    如果西方的“上升趋势”持续了大约500年,那么谁知道下降趋势会持续多久。 尤其是在我们现在生活的全球化时代; 最终,不仅仅是白人,每个人都可能自愿传播自己的文化和遗传遗产; 我认为这迟早会发生。 你真的认为西方能够在种族奇点发生之前度过当前的衰退周期吗? 如果你问我,我认为这只是西方下行周期的开始,我们至少还有几个世纪的时间。 从现在起的短短几百年里,从种族意义上看世界会是什么样子,将会非常有趣。

    我同意白人国家正在收获他们在非个人的业力过程中所播下的种子——尽管与同情和内疚的问题无关。 我担心,如果中国不小心的话,可能会重蹈覆辙——我们是被敌人塑造的,长大后往往会变得和施虐者一样。

    西方不是中国,中国也不是西方。 它们是来自两个不同地方的两个独立实体。 是的,我们受到敌人的影响。 但我们并不完全由它们塑造。

  346. GammaRay 说:

    至于大多数白人是否同意,我认为分歧比你说的要多得多——而且这个过程才刚刚开始。

    我并没有说大多数白人对此表示同意。 我说过大多数白人要么对此表示同意,要么完全无动于衷。 很可能绝大多数白人完全漠不关心,除了 RAHOWA 1488 之外还有其他事情需要关注。

    这个过程才刚刚开始吗? 在我看来,白人中将继续存在低等级的“觉醒”,这种觉醒在未来将会放大,但它永远不会成为一种全面的白人权力/驱逐所有移民类型的情绪。 至少在美国; 我不能代表欧洲发言。 大多数白人永远不会签署全面的白人种族意识项目的主要原因可能是因为他们不想将自己与世界上的马修·海姆贝克斯、理查德·斯宾塞和安德鲁·安格林斯联系在一起。 正是由于极右极端分子完全是新纳粹分子/表现得像合法的仇恨团体,所以大多数白人对他们的信息感到厌烦(“黑人在基因上是劣等的!墨西哥人是败类!与犹太人在一起你就输了!!白人是最好的,我们殖民了整个世界,所以操你!!等等”)。 极右翼根本无法放弃他们自己强烈的种族沙文主义,降低他们的骄傲(实际上是狂妄自大),只是把事情缓和一些。 但值得庆幸的是,由于他们自己总是倾向于完全迟钝,这吓跑了本来会对他们的信息有所同情的白人。 我认为没有人真正愿意生活在白人民族主义者周围,甚至可能不是其他白人民族主义者,哈哈。

    我心中的 SJW 感到遗憾的是,白人可能会遭受虐待,然后再次生气,这样的循环将会重复。

    你说遭受虐待? 白人从殖民开始了这整件事,并且是最初的施虐者。 最初没有人虐待白人,然后让他们殖民并虐待整个世界,白人选择这样做是出于自己的意愿。 如果你真的这么想,你的世界观就严重扭曲了

    这些天,我努力培养一种冷静的态度,接受这个世界的所有动荡的陌生和奇迹,而不是怀抱不可能实现的公平与和平的梦想,我现在意识到,这是西方进步错觉的残余痕迹。尽管我不断地反对它,但它还是以某种方式感染了我。

    哈哈,我对你的自负感到非常惊讶。 你居然认为只有西方人才有能力编织公平与和平的梦想? 哈哈,好吧。 WN 种族沙文主义 FTW 再次出现

    我部分受到道教的启发——道教是我一直谈论的可怕的中国怪物。

    不正确。 你唯一的缺陷是你提出的论点并没有得到整体意义上的真正支持。 你假设中国人将成为全球种族灭绝者和文化破坏者(又名西方风格)的唯一理由是基于对阴阳概念的过于广泛的应用。 你认为因为西方过去太阳,现在中国将变得太阳,而西方则变得更阴。 虽然在一般意义上我相信这是真的,但我不同意中国将像西方在强国时期那样行事的观点。 仅仅因为动态随着时间的推移而变化并不意味着所涉及实体的基本内在性质完全改变; 相反,动态转变作用于先前存在的文化、遗传等基础并与其融合,从而最终导致新现实的诞生。

    如果中国人本来就是阳的,我只会担心中国变成一个种族灭绝的全球战争机器; 然而,鉴于中国人普遍比西方人更阴,这强烈表明,与遗产相比,即将到来的阳能量注入中国人的超意识将为整个世界带来更平衡、更良性和不那么极端的结果西方离开了,因为西方在开始其殖民灾难之前就已经开始更加阳刚了。

    我会让你回到你高度客观、充满激情、完全中立的党派主张,这是由你完全不关心发生什么的态度决定的,导致你在这个主题上写下热情洋溢的长篇大论。

    我对宏观层面的事件如何发展并不关心,因为我正确地认识到,也许我对它们无能为力。 但我确实喜欢在微观层面上与人进行口角,所以我才费力地写出“长篇大论”。

  347. GammaRay 说:
    @AaronB

    抱歉,我忘记在前两篇帖子中标记您,只是在这里标记您,以便您知道我回复了。

  348. @TT

    抱歉,最近很忙。 本周晚些时候将回复,但请注意,纪元是法轮功的废话。

  349. Santoculto 说:
    @Factorize

    人类是狂妄自大的野兽,正在摧毁这个美丽的星球……必须被人类消灭并从其基础上重塑……

  350. Santoculto 说:
    @GammaRay

    哦,当然,我怎么会犯这样的错误。 这些可怜的小西班牙征服者被精英阶层误导,他们前往新世界不仅仅是为了获得财富、当地妇女的后宫和自由土地。 你们错误地相信了错误的二分法,即必须责怪人民或精英; 事实上,有时人民和精英都应该受到指责。

    我不知道我在哪里说过这种二分法,两者都必须受到指责,但以不同的方式,因为精英是那些决定事情的人,人民,驯服的“小狗”[经常被压迫的小狗]也只是服从和遵守规则发生在中国人身上的事情……在所谓的“文化大革命”期间,中国发生了许多可怕的事情,没有考虑到“中国政府”对普通中国人和生活在这里/这个国家的非人类生物所做的许多可怕的事情。

    不,更准确的说法是,发生在白人身上的事情是意外后果法则。 如果发生不公正现象; 那么我们应该责怪白人的祖先(欧洲殖民者以及所有直接和间接促成了欧洲对世界的殖民化这一多世纪现象的人)造成了这种情况。 前几代白人造成了这种混乱,而当代白人将不得不处理这个问题。

    也是这样,但这并不能消除这样一个事实,即许多人正在为他们没有犯下的罪行付出代价,我不是一个错误的二分法……😉

    再说一次,你没有明白的基本点,它也解释了你的“自己的”人。 大多数人或普通人都是被驯服的[无论他们的智商如何,重要的是要注意]和/或无法改变他们的精英创造的世界,群众往往就像农场里分散的牛一样,他们没有组织,他们往往不太关心这一点,世界是由一小群组织良好、精力充沛、实际上很聪明、邪恶或无情的人塑造的。 是的,我并不是说我们需要停止责怪白人,而是停止只责怪他们,不仅因为根据你的道德标准,所有人在他们历史的某个时刻都倾向于有罪,而且还因为人类群体、个体往往既是无辜的,又是有罪的。

    其次,白人身上发生的事情真的被认为是不公正吗? 许多白人都接受多元文化主义和移民。 许多白人甚至与非白人结婚生子。 你认为这些人会认为目前的事态“不公正”吗? 许多其他白人完全漠不关心,也不关心。 您认为他们会认为目前的事态“不公正”吗? 在当前背景下,不公正是一个非常模糊的术语。 目前的事态只有另类右翼人士明确认为是不公正的。 但对其他人来说,现实情况要主观得多。

    这就像说

    那只蜜蜂没问题,因为黄蜂。 他们被欺骗了,完全被欺骗了,这是一种洗脑,但不是大多数人相信的那样。 人们相信今天有更明显的同性恋者,因为他们被洗脑了,不完全是这样,而是因为今天的文化对此不再那么愚蠢无知,但是,这并不能阻止人们从事许多知识分子的无知事物,甚至在他们的生活中将其内化和正常化,它现在正在西方发生。

    是的,由于欧洲人的一定程度的理性和可怕的全球历史,许多白人对自己的剥夺感到同意,许多人甚至称之为卡玛,但这种大规模的欺骗不是启蒙西方的,而是为了让他们做好准备(((有些-那些)))谁设计了这一切,似乎多元文化主义也被引入了远东,这是一种全球统治。

    他们之所以接受这一点,是因为许多人无法从多个角度思考,或者还不足以达到理性。 用于攻击欧洲白人抵抗运动的另外两个方面(((thoos)))是:

    本质主义和存在主义/接近个人主义的生活方式

    第一个被用来以超现实的实用方式展示谁在某种程度上种族特征是多余的。 智力、智慧[定性智力]还是什么,善良胜有蓝眼睛。 所以他们也创造了这种强制对话,许多人决定选择本质而不是外表。

    存在主义使人们以更加自助的方式过活在当下。 我们关注我们的权利并关注我们的愿望,这并不完全是坏事,但当我们完全超越共同或集体权利时,我们只是以非常自私的方式行事。

    物质主义是维持过度以个人为中心的生活方式的另一种方式,即使物质主义比本质主义更符合种族或先验的装饰/可识别特征。

  351. Santoculto 说:
    @GammaRay

    在我看来,你基本上是在试图将责任从白人身上转移到犹太人身上。 你本质上所做的事情被称为转移注意力。 这是我对此的看法。 从文化意义上来说,无论有或没有犹太人的参与,白人通常都是专横和好斗的。 一个完美的例子就是美国军队。 当然,最高层可能有犹太复国主义犹太人,他们为了以色列的利益而让美国卷入中东战争,但当谈到美国军队的普通士兵时,也有很多白人他们迫切地想要去杀戮并征服世界,他们是一毛钱一打。 事实上,有些犹太人可能为了自己的邪恶目的而操纵这些白人,但这并不能否认许多白人本身(没有犹太人的操纵)似乎天生就被侵略和征服所吸引。

    只是稻草人…

    我只是在这场辩论中使用“犹太人”这个词。

    平均而言,或者总体而言,我并不是想神圣化白人,即使因为我是这里不仅批评白人而且批评保守派的人之一。 是的,我不相信如果没有犹太瘟疫,白人也不会成为瘟疫,但即使在第二次世界大战之前,我们仍然不知道有关犹太人参与范围扩大的正确知识。 例如,广岛和长崎的核袭击……最大的区别在于,犹太人和与他们结盟的精英创造了一种方式来指责白人,作为一个民族和种族,并从自己的背后推卸自己的责任。 这就像一个非常聪明的高功能精神病患者,犯了罪,但却找到了一种非常聪明的方式来指责他们的“盟友”,甚至是最弱的一方,人民,而他们自己,以一种非常厚颜无耻的方式,表现为他们的救世主......自己的罪行。

    是的,你在这一部分的最后几句话表明,当你认为我主张避免对“白人”进行任何类型的指责时,你误解了我。

    我认为 WN/另类右翼有时喜欢表现得好像白人不是真正的坏人,而只是犹太人让白人做坏事; 但从与 WN/另类右翼分子的互动以及他们的言行来看,很明显,无论有或没有犹太人,白人都可以自己做坏事(例如考虑一下乔·韦伯的扭曲言论) 。 正如我之前所说; 白人具有一定的活力和神气,有利于大型进取、雄心勃勃的企业; 犹太人似乎偶尔会有效地利用这种能量来谋取自己的利益,但这并不能神奇地免除白人自己的罪行。

    好像白人不是真正的坏人……即使部落主义者在辩论有关“自己的”人民时具有典型的倾向性是正确的,你的话也可能表现出一种不诚实的观点,再次强调,白人才是唯一应该受到指责的人犹太人……好吧,犹太人只是被误解的天使。 在所有人群中都有非常坏的人、坏人、一般人(大多数人)、好人、非常好的人。 它基于您对真正道德问题的担忧程度。 我不认为美国白人平均比中国人差很多,即使是因为在不同的问题上中国人似乎很糟糕,例如,对待其他生物和自然的方式。 是的,你不需要记住我关于工业化的事情……作为可能的反驳。

    不幸的是,大多数漂亮善良的人都是准猎物,他们对自己太好了,在不创造他人的情况下无法帮助我们真正解决问题,而聪明的人,当善良与智慧结合在一起时,特别是在定性方面,往往会变得更聪明。相当罕见。

    犹太人似乎偶尔会有效地利用这种能量来谋取自己的利益,但这并不能神奇地免除白人自己的罪行。

    偶尔…

    你相信犹太人现在并没有主宰西方世界吗?如果不对西方已经发生和将要发生的所有令人毛骨悚然的事情承担全部责任,他们也不会参与其中** 告诉我..

    不幸的是,社会以非常严格的等级制度运作,所以如果你的人民想要统治另一个人,他们只需要关注最重要的地方,在那里做出真正的决定。 这是犹太人,特别是犹太知识分子黑手党自二十世纪初以来所做的事情。 白人作为一个民族不会因为强烈的自我憎恨而变得疯狂。 是的,我不同意你对所有西方悬而未决的看法,我希望你也不同意你的看法。

    另一件事是,高科技社会往往为普通人和易受影响的人创造安全观念。 人们不能决定国家的未来,只能决定他们自己的未来,而且是在非常狭隘的层面上。

    巴西是一个大多数人都不是欧洲白人的地方,他们被欺骗相信多元文化。 对于穆里卡的黑人、西班牙裔、东亚人和其他人来说也是如此,但仍然有不同的观点,其中大多数人而不是东亚人,经常以潜意识的方式利用所有这些后现代的发明和/或阴谋来比那些人占优势。建立了这一切帝国。 在穆里卡以及加拿大,东亚人的进步主义似乎更高,例如有很多异族通婚。

    • 回复: @GammaRay
    , @GammaRay
    , @GammaRay
  352. Factorize 说:
    @GammaRay

    我同意,中国人确实不适合“平等中的第一”的地缘政治结构,而这种结构在过去几千年中一直是西方文明的核心支柱。 这可能会成为 21 世纪(对某些人来说)更令人惊讶的启示之一。

    人们越来越期望中国将登上全球超级大国的应有地位。 然而,尽管从统计数据来看这是现实的,但中国实际的长期文化行为与这样的评估并不相符。 Veni、Vidi、Verti 更能描述中国人过去(也可能是未来)的行为。

    考虑到中国处于智商金字塔的顶端,中国与世界其他国家的互动究竟能获得什么好处? 无论谁赢得了全球超级大国的称号,都将承担起努力帮助地球上一些最不发达和功能最失调的社会的负担。

    对于本论坛上的任何人来说,21 世纪将是帮助非洲的世纪都不足为奇。 然而,即使是最强大的全球超级大国也似乎不太可能应对这一挑战。 那么,对于任何人来说,在某个时候中国将放弃全球霸主的崇高荣誉就不足为奇了。

    未来更有可能的道路是中国继续专注于成为一个经济超级大国:寻找让我们所有人的生活变得更美好的产品和技术,通常是在西方没有竞争力的行业。 过去认为聪明人让我们的生活变得更糟的怨恨已不再成立。 在这个新技术经济中,伟大的想法创造巨大的财富,并且认知财富是共享的。 中国成为国际社会不可或缺的一员,为世界带来了巨大优势。 如果他们能够创造出一种新的技术形式,使技术变得更加人性化,我们都会因此变得更加富有。

    • 回复: @Yan Shen
    , @GammaRay
  353. Yan Shen 说:
    @Factorize

    对于本论坛上的任何人来说,21 世纪将是帮助非洲的世纪都不足为奇。 然而,即使是最强大的全球超级大国也似乎不太可能应对这一挑战。 那么,对于任何人来说,在某个时候中国将放弃全球霸主的崇高荣誉就不足为奇了。

    中国这些年不也是通过大规模的基础设施建设来换取与非洲的资源贸易吗? 西方列强首先残暴地对待非洲国家,掠夺他们的资源,然后转向另一端,简单地向非洲国家倾销财政援助,同时不断在人权和民主问题上对他们进行威吓,而中国与非洲大陆的互动似乎相当务实,而且也很务实。相比之下,它富有成效,没有意识形态的姿态。 正如“一带一路”倡议所表明的那样,中国似乎满足于以身作则、以身作则,而不是通过不断的空谈和入侵/邀请世界。

    • 回复: @Factorize
    , @Truth
  354. Factorize 说:
    @Yan Shen

    我正在更具体地考虑一种更强有力的参与形式。 这是西方几千年来的模式。 也许中国可以寻求一种新的方式来解决这些问题,但据猜测,与非洲发展互动的主要商业模式可能还不够。

    这正是我的观点:成为世界超级大国在很大程度上意味着要承担其他国家问题的重担。 过去几十年来,美国面临的压力很大程度上与试图帮助日益功能失调的社会有关。 几乎不可能想象第二次世界大战(对美国来说)主要是为了解决现在高度发达的欧洲国家的内部社会经济问题。

    现在,经过几十年的发展,整个世界(撒哈拉以南非洲除外)已经确保了人口和地缘政治稳定的未来。 剩下的一个大问题是如何在非洲复制这一未来。 那里尚未发生人口转变,因此这将是一场长期的斗争(大约一个世纪或更长时间)。 我非常不清楚谁会愿意承担这个任务。 一段时间以来,非洲局势持续恶化。 世界其他地区加起来完全有可能无法应对非洲存在的严重功能障碍。 然而,如果未来需要人类的集体资源,而不是像中国这样的超级大国,这可能是我们拥有的最好的机会。

    • 回复: @Yan Shen
  355. Yan Shen 说:
    @Factorize

    我正在更具体地考虑一种更强有力的参与形式。 这是西方几千年来的模式。 也许中国可以寻求一种新的方式来解决这些问题,但据猜测,与非洲发展互动的主要商业模式可能还不够。

    呃,如果你没有注意到的话,近几十年来,美国与第二和第三世界大部分国家的交往模式绝对是一场灾难。 我们试图将我们自己的价值观强加给那些在文化和制度上不适合采用这种价值观的国家,但我们却惨遭失败。 至少可以说,我们入侵世界和国家建设的记录相当糟糕。

    事实上,美国对非洲等地区的态度也好不到哪儿去。 我们往往在监督最少的情况下向这些国家倾销援助,助长了腐败,同时天真地要求这些国家的政府采用美国的民主规范,就好像赤贫国家没有真正的中产阶级,除了威权主义之外没有任何历史一样。规则可以以某种方式在一夜之间神奇地体现盎格鲁文化圈的价值观!

    从某种意义上说,美国政客就像21世纪过去几个世纪的欧洲传教士一样,他们沉迷于提供赦免和拯救的理想主义观念,往往造成的伤害远大于任何真正的好处。 事实上,现代中国人冷静的实用主义几乎肯定会带来更多、更持久的成果,并应该为世界各地的其他人树立榜样,让他们了解如何以成熟的视角看待此类全球事务。

  356. GammaRay 说:
    @Santoculto

    我不知道我在哪里说过这种二分法,两者都必须受到指责,但以不同的方式,因为精英是那些决定事情的人,人民,驯服的“小狗”[经常被压迫的小狗]也只是服从和遵守规则发生在中国人身上的事情……在所谓的“文化大革命”期间,中国发生了许多可怕的事情,没有考虑到“中国政府”对普通中国人和生活在这里/这个国家的非人类生物所做的许多可怕的事情。

    为什么要提中国人? 没有人否认他们的衣柜里也有骷髅。 你所做的只是逻辑谬误“tu quoque”的一种弱形式。 更搞笑的是,你发布该视频显然是为了引发某种强烈的情绪反应,以缓冲你的(微弱的)论点。

    我很抱歉,但你关于如何只应该指责精英(而不是执行他们命令的人)的论点并没有那么有说服力或令人信服。 我将使用您相同的逻辑来进行类比,以显示您的论点的本质弱点:

    假设有一个巴基斯坦儿童美容团伙在英国活动; 该儿童诱骗团伙的成员直接对英国未成年女性进行性虐待,而该团伙的主谋甚至从未碰过该团伙所诱骗的任何女性; 然而,他是从最高层资助和组织整个事情的。 依你的意见; 到底谁在道德上更有罪责呢? 是帮派头目还是他的追随者?

    从这个类比中可以看出,实际上; 正确地归咎于你并不像你的行为那样容易。 你继续试图推动这样的论点:其他人实际上都可以被剥夺代理权,所有的责任都可以归咎于精英,但很明显,在现实生活中,这些情况比你试图采取的行动要混乱得多就像他们一样。

    不,更准确的说法是,发生在白人身上的事情是意外后果法则。 如果发生不公正现象; 那么我们应该责怪白人的祖先(欧洲殖民者以及所有直接和间接促成了欧洲对世界的殖民化这一多世纪现象的人)造成了这种情况。 前几代白人造成了这种混乱,而当代白人将不得不处理这个问题。

    也是这样,但这并不能消除这样一个事实,即许多人正在为他们没有犯下的罪行付出代价,我不是一个错误的二分法……😉

    再一次,你的理解是有缺陷的,这里不存在错误的二分法。 如今,几代白人正在为他们没有犯下的罪行付出代价(我在任何地方都没有否认这一点,尽管我确实部分不同意这种推理); 但这是我实际写的:

    前几代白人造成了这种混乱,而当代白人将不得不处理这个问题。

    我在这里写的内容没有任何情感修辞; 我所写的只是一个简单、客观、真实、符合事实的陈述。 我并不是在表达任何“我认为应该是真实的”的个人偏见,而只是对事件因果关系进行事实观察。 现代几代白人必须面对他们祖先的错误,这一事实是真实的,独立于任何表明现代几代白人应该受到这种惩罚的道德/哲学论点。 你误解了所提出的论点,这一事实取决于你,而不是我。

    再说一次,你没有明白的基本点,它也解释了你的“自己的”人。 大多数人或普通人都是被驯服的[无论他们的智商如何,重要的是要注意]和/或无法改变他们的精英创造的世界,群众往往就像农场里分散的牛一样,他们没有组织,他们往往不太关心这一点,世界是由一小群组织良好、精力充沛、实际上很聪明、邪恶或无情的人塑造的。 是的,我并不是说我们需要停止责怪白人,而是停止只责怪他们,不仅因为根据你的道德标准,所有人在他们历史的某个时刻都倾向于有罪,而且还因为人类群体、个体往往既是无辜的,又是有罪的。

    你犯了WN/另类右翼的常见错误,即认为任何批评西方殖民主义的人都专门针对白人; 然而事实并非如此。 如果这次谈话的主题是关于任何其他犯有种族灭绝/暴行的种族/群体,那么我客观地分析他们的罪责和潜在的罪责是没有问题的。 话虽这么说,评论部分这部分的辩论主题完全与欧洲殖民主义有关,这就是为什么我们正在讨论西方/白人的潜在罪责。 如果您认为我不以平等标准对待所有其他人/种族,那您就错了。

    我知道你基本上想暗示什么,我将继续指出你的论点。 你基本上试图提出这样的论点:白人并不是唯一对欧洲殖民主义负有责任的人,因为其他每个种族都做过类似的事情,等等。话虽这么说,这是一个欺骗性和不诚实的论点。 我将用一个简单的类比来说明为什么会这样:

    黑人杀人,但其他种族的人也杀人; 那么我们为什么要关注黑人犯罪呢? (很明显,黑人谋杀率远高于其他种族,这就是为什么我们关注黑人谋杀率而不是白人或亚洲人谋杀率。)

    同样,白人殖民人民,但其他种族的人也殖民人民; 那么我们为什么要担心白人殖民人民呢? 显然,我们应该担心白人殖民他人(并且对种族灭绝方面也应特别彻底),因为不言而喻,白人更熟练/最倾向于殖民和种族灭绝其他人(白人殖民/试图殖民整个世界并且种族灭绝了无数的土著文化)。 其他人也犯有殖民和种族灭绝罪,但白人做得最好,而且许多人(实际上是大多数 WN/另类右翼分子)都愚蠢到公开承认这一点并为此感到自豪,尽管这实际上是在起诉他们道德/伦理水平。

  357. GammaRay 说:
    @Santoculto

    是的,由于欧洲人的一定程度的理性和可怕的全球历史,许多白人对自己的剥夺感到同意,许多人甚至称之为卡玛,但这种大规模的欺骗不是启蒙西方的,而是为了让他们做好准备(((有些-那些)))谁设计了这一切,似乎多元文化主义也被引入了远东,这是一种全球统治。

    这是一个主观的论点。 你说它是洗脑,因为你个人不喜欢它,但仅仅因为你不喜欢某件事并不意味着它是洗脑

    是否有某个建筑师试图将多元文化主义推向全世界? 可能。 这是多元文化主义增长和传播的唯一解释吗? 没有。 对多元文化主义的接受也可能是由于有机的、自然发生的因素。 面对现实吧; 多元文化主义和对多元文化主义的开放是不可避免的,即使没有假设性的尝试对每个人进行洗脑以使其追随它; 无论如何,它是不可避免的,原因在于两项技术创新:

    1.飞机
    2. 互联网

    仅这两件事就使世界比几百年前小得多。 以这个速率; 阻止多元文化政策的有机实施无异于将手指放在水坝上。

    话虽如此; 关于对多元文化主义传播的阴谋导向的解释,假设这是真的,我真的不认为它会在亚洲获得关注,我也不认为这是“建筑师”的意图混合剩下的世界。 相反,我认为他们唯一的目标是西方; 我认为他们这样做的原因是因为他们坚信业力的概念。 很好地尝试通过提出多元文化主义可能威胁远东这一事实来让我同意你的观点。 另外,我觉得很有趣的是,你试图指责“建筑师”想要追求全球统治,同时你又试图淡化西方(及其大量人口)实际上参与了统治整个世界的事实。世界。 唉,真是缺乏自我意识啊

    存在主义使人们以更加自助的方式过活在当下。 我们关注我们的权利并关注我们的愿望,这并不完全是坏事,但当我们完全超越共同或集体权利时,我们只是以非常自私的方式行事。

    欧洲殖民主义难道不是一种至高无上的自私行为吗? 这是一种至高无上的自私行为,而那些欧洲殖民者的后裔仍然直接从中受益? 哈哈,好的

    平均而言,或者总体而言,我并不是想神圣化白人,即使因为我是这里不仅批评白人而且批评保守派的人之一。 是的,我不相信如果没有犹太瘟疫,白人也不会成为瘟疫,但即使在第二次世界大战之前,我们仍然不知道有关犹太人参与范围扩大的正确知识。 例如,广岛和长崎的核袭击……最大的区别在于,犹太人和与他们结盟的精英创造了一种方式来指责白人,作为一个民族和种族,并从自己的背后推卸自己的责任。 这就像一个非常聪明的高功能精神病患者,犯了罪,但却找到了一种非常聪明的方式来指责他们的“盟友”,甚至是最弱的一方,人民,而他们自己,以一种非常厚颜无耻的方式,表现为他们的救世主......自己的罪行。

    这是荒谬的。 在第二次世界大战发生之前的几个世纪里,白人一直试图对抗和开放日本。 第二次世界大战只是西方对东亚长期对抗过程的顶峰。 你再次试图用广岛/长崎的例子来推卸白人的责任(这与犹太人有什么关系?),然后忽视白人试图开放和剥削的数百年历史,这是不诚实的不只是日本,而是整个东亚。 这就是为什么你不能被认真对待,你不能提出严肃的论点。

    • 回复: @Santoculto
  358. GammaRay 说:
    @Santoculto

    我认为 WN/另类右翼有时喜欢表现得好像白人不是真正的坏人,而只是犹太人让白人做坏事; 但从与 WN/另类右翼分子的互动以及他们的言行来看,很明显,无论有或没有犹太人,白人都可以自己做坏事(例如考虑一下乔·韦伯的扭曲言论) 。 正如我之前所说; 白人具有一定的活力和神气,有利于大型进取、雄心勃勃的企业; 犹太人似乎偶尔会有效地利用这种能量来谋取自己的利益,但这并不能神奇地免除白人自己的罪行。

    好像白人不是真正的坏人……即使部落主义者在辩论有关“自己的”人民时具有典型的倾向性是正确的,你的话也可能表现出一种不诚实的观点,再次强调,白人才是唯一应该受到指责的人犹太人……好吧,犹太人只是被误解的天使。 在所有人群中都有非常坏的人、坏人、一般人(大多数人)、好人、非常好的人。 它基于您对真正道德问题的担忧程度。 我不认为美国白人平均比中国人差很多,即使是因为在不同的问题上中国人似乎很糟糕,例如,对待其他生物和自然的方式。 是的,你不需要记住我关于工业化的事情……作为可能的反驳。

    你显然误解了我写的语气。

    我认为 WN/另类右翼有时喜欢表现得好像白人不是真正的坏人, 相反,这只是犹太人让白人做坏事; 但从与 WN/另类右翼分子的互动以及他们的言行来看,很明显,无论有或没有犹太人,白人都可以自己做坏事(例如考虑一下乔·韦伯的扭曲言论) 。

    很明显,我是在元时尚中这么说的,并不打算直接或字面地理解这一点。 再一次,由于你的阅读理解能力错误,你错误地认为我在说一些我实际上并没有说的话。 因为你的段落的其余部分是基于你对我所说内容的误解; 剩下的就不用我说了

    你相信犹太人现在并没有主宰西方世界吗?如果不对西方已经发生和将要发生的所有令人毛骨悚然的事情承担全部责任,他们也不会参与其中** 告诉我..

    我相信,如果犹太人现在主宰西方世界,那只是因为他们作为业力的力量,通过西方所崇拜的事物(玛门和战争)来瓦解西方。 借用成吉思汗的一句话来更好地解释我对此事的感受:

    我是神的惩罚……如果你没有犯下大罪,神不会对你施以我这样的惩罚。

    如果西方没有犯下大罪,那么它就不会像犹太人那样受到惩罚。

    再次回到原点。 我不同意你的观点,即人们不负责,只有精英才负责。 这是一个愚蠢的论点,这基本上就是你试图提出的论点,尽管你(不令人信服的)试图提出相反的主张。 我认为人民和精英都有责任,两者可以以不同的方式负责,有时人民和精英的意图非常吻合,在这种情况下我会平等地认为人民和精英都负责负责任的。 它真的非常简单,并不像你故意让它看起来那样做作和复杂。

    最后,我觉得很搞笑,你完全跳过了我给你的另外两条回复; 顺便说一句,几乎已经解决了您在最新回复中提出的所有要点。 我浪费了时间再次把所有这些写给你,而实际上我应该让你阅读并重新阅读我最初写的内容。

    • 回复: @Santoculto
  359. GammaRay 说:
    @Factorize

    感谢您的周到答复。

    国际海事组织认为,非洲人口统计危机最终会自行解决。 我认为,作为全球超级大国的中国可能不会直接参与帮助非洲国家解决自己的问题,但中国会顺便通过其不干涉政策以及商业/基础设施项目来帮助非洲。

    我认为全球超级大国必须帮助功能失调/弱小的国家的观念更多的是西方的想法。 据我所知,没有任何真正的力量可以迫使一个全球超级大国这样做。 完全有可能成为全球超级大国,而不必为帮助弱国承担负担。 事实上,我怀疑这正是许多第三世界国家所欢迎的,也就是说,他们希望获得有效的政治主权和任何霸道的全球超级大国实体的不干涉。 如果真是这样(我强烈怀疑是这样),那么中国可以轻松地扮演一个相对有益/中立的全球超级大国的角色,同时拒绝(直接)帮助解决第三世界国家事务的负担。

  360. Santoculto 说:
    @GammaRay

    首先,

    冷静点,我现在不想在这场辩论中与你“战斗”。

    其次,

    不,无论我在这里做什么,我从来没有意图操纵别人,我不需要它,因为我实际上所做的是相反的,我总是试图消除我认为像你们许多人一样不诚实或迟钝的评论。

    只是说我的论点很弱,不要让它变弱,让你的论点变弱。

    回答这个问题:所有中国人或中国人民/华人种族都必须为在你们国家和国外犯下的暴行负责**

  361. Santoculto 说:

    抱歉,你关于如何只应该指责精英的争论 (而不是执行命令的人)并没有那么有说服力或令人信服。 我将使用您相同的逻辑来进行类比,以显示您的论点的本质弱点:

    有一个 假想 在英国活动的巴基斯坦儿童美容团伙; 该儿童诱骗团伙的成员直接对英国未成年女性进行性虐待,而该团伙的主谋甚至从未碰过该团伙所诱骗的任何女性; 然而,他是从最高层资助和组织整个事情的。 依你的意见; 到底谁在道德上更有罪责呢? 是帮派头目还是他的追随者?

    我没有时间也没有耐心与像你这样的人辩论……从你误解我的评论那一刻开始[我从来没有说过只有精英才应该受到指责……或者你错误地阅读了我写的内容,或者你故意使用这个,好像我不会记得我写过什么]当你使用这样令人震惊和具有启发性的词“假设”来描述确实发生的事情时。

    所以我会留在这里

    以可以理解的方式重新解释我认为我写的内容

    当你把“假设”这个词放在实际上可怕的事实中时,你会反对你猖獗的不诚实和邪恶……

  362. Santoculto 说:
    @GammaRay

    你在这里评论的整个第一部分都是基于误解……时期。

    你把我的评论的一部分分开,我所说的“只是责怪精英……”并开始反驳[贬低我的论点]。

    “为什么要提中国人? ”

    为什么不*

    另一个宝贵的建议,减少你的傲慢……看来你被触发了……

  363. MEH 0910 说:

    严深,你是不是一个在写字的时候不停地插入“嗯”字,暗自向母校致敬的女钢琴神童? U大学 H奥斯顿?

    http://www.uh.edu/kgmca//music/tmf/about-the-festival/orchestra-fellows/2017/profiles/yan-shen.php

  364. Santoculto 说:

    因为你说

    这种假设性的集体修饰……你将自己置于有问题的境地,也将你自称的诚实置于有问题的境地。

    讨论结束

    白人可以被集体指责,但这并不意味着他们应该被消灭……你能理解或接受吗*

  365. Santoculto 说:

    你犯了WN/另类右翼的常见错误,即认为任何批评西方殖民主义的人都专门针对白人; 然而事实并非如此。

    不,我从来没有说过任何批评西方殖民主义的人都专门针对白人,我在哪里写了这篇文章** 给我看看。 即使因为我是批评西方文明的人之一,并且经常将大部分罪责归咎于欧洲人,就这样。

    如果这次谈话的主题是关于任何其他犯有种族灭绝/暴行的种族/群体,那么我客观地分析他们的罪责和潜在的罪责是没有问题的。

    因为他们现在正在遭受种族灭绝。

    话虽这么说,评论部分这部分的辩论主题完全与欧洲殖民主义有关,这就是为什么我们正在讨论西方/白人的潜在罪责。 如果您认为我不以平等标准对待所有其他人/种族,那您就错了。

    我主要是白人/欧洲人和西方人,因为我出生在西半球,并且已经融入/接受了其文化的一部分,但我对过去所有犯罪的内疚感为零,因为我出生在 80 年代末,而且我的潜力是的,为这些罪行辩护的愧疚感已经减少了,因为我过去已经试图将其中的许多罪行“合理化”。

    两种叙事方式

    第一个是抽象的神话

    人们这样做了,

    马人发明了现代性
    blablablabla

    在超现实中,我是一个没有发明任何东西,也没有奴役人民的个体。

    我们遇到麻烦是因为我们的方法似乎不同。 你的方法似乎更加二元化,而我的方法倾向于通过“多重”视角。 我将我对这个问题的看法分成许多部分或部分,而不是创建一个单一的叙述。

    我知道你基本上想暗示什么,我将继续指出你的论点。 你基本上试图提出这样的论点:白人并不是唯一对欧洲殖民主义负有责任的人,因为其他每个种族都做过类似的事情,等等。话虽这么说,这是一个欺骗性和不诚实的论点。 我将用一个简单的类比来说明为什么会这样:

    黑人杀人,但其他种族的人也杀人; 那么我们为什么要关注黑人犯罪呢? (很明显,黑人谋杀率远高于其他种族,这就是为什么我们关注黑人谋杀率而不是白人或亚洲人谋杀率。)

    即使说我不只是在争论这一点也不是完全错误的,即使我已经说过,西方巨大的道德历史问题之一是它已经或已经完全全球化。 巨大或有问题。

    我并不是说,当你试图稻草人化时,西方人在任何情况下都不能受到西方殖民/剥削/任何邪恶和愚蠢的指责。

    你的简单逻辑看起来很简单。

  366. Santoculto 说:
    @GammaRay

    黑人杀人,但其他种族的人也杀人; 那么我们为什么要关注黑人犯罪呢? 同样,白人殖民人民,但其他种族的人也殖民人民; 那么我们为什么要担心白人殖民人民呢?

    完全扭曲了我的论点,我不知道为什么

    显然,我们应该担心白人殖民他人(并且对种族灭绝方面也应特别彻底),因为不言而喻,白人更熟练/最倾向于殖民和种族灭绝其他人(白人殖民/试图殖民整个世界并且种族灭绝了无数的土著文化)。 其他人也犯有殖民和种族灭绝罪,但白人做得最好,而且许多人(实际上是大多数 WN/另类右翼分子)都愚蠢到公开承认这一点并为此感到自豪,尽管这实际上是在起诉他们道德/伦理水平。

    是的,如果我不同意你的观点,请告诉我。

    不过,如果殖民者足够明智的话,我并不认为殖民化绝对是最糟糕的事情; 如果其他人可以成为你的盟友或从根本上成为你的良性合作者,你就不需要消灭或征服他们,这看起来很理想主义和天真,但我认为事实并非如此。

    其他人也犯有殖民和种族灭绝罪,但白人做得最好,而且许多人(实际上是大多数 WN/另类右翼分子)都愚蠢到公开承认这一点并为此感到自豪,尽管这实际上是在起诉他们道德/伦理水平。

    就像大多数保守派一样,无论其种族如何,请记住,可以通过征服、替代或任何其他纯粹邪恶的策略在自己的领土上“殖民”,例如在西藏发生的事情。

  367. Santoculto 说:
    @GammaRay

    这是一个主观的论点。

    当有人使用这个原始论点时,我总是相信他/她试图冒充知识分子。

    你说它是洗脑,因为你个人不喜欢它,但仅仅因为你不喜欢某件事并不意味着它是洗脑

    啊啊啊啊,不。

    许多人还喜欢说今天的同性恋是洗脑的产物,嗯,我是同性恋,我并不是特别被洗脑,因为在我的第一个童年时期,我更关心恐龙而不是宏观地缘政治中发生的事情。

    洗脑和欺骗是非常相似的事情,即使不是基本相同。 [洗脑是欺骗的隐喻]。 因为人们更容易成为小狗,所以他们被训练[和自我训练]相信:

    犹太人是受迫害的天使。

    黑人也是如此,他们绝对是受害者,他们的大部分智力和行为“缺陷”都是直接由于西方“殖民化”或占领或系统性种族主义造成的。

    人类并不存在,而“实际上”却存在,而且这一点是显而易见的。

    这是经典条件反射的典型案例,因为大多数人都有二元思维,所以他们无法克服这种捏造的困境。 X 或 Y*

    基本上你已经应该知道的,正如你自己已经在这里所说的:你认识的大多数白人并不关心“白人种族”,为什么*尝试找出原因*

    是否有某个建筑师试图将多元文化主义推向全世界? 可能。

    如果你还处于这个水平……当韩国所有的教科书都在“教导”多元文化有多么伟大时……

    这是多元文化主义增长和传播的唯一解释吗? 没有。

    这是基本解释或该解释的一部分,其他解释充其量是次要的或反应性的或该第一意图的产物。

    对多元文化主义的接受也可能是由于有机的、自然发生的因素。 面对现实吧; 多元文化主义和对多元文化主义的开放是不可避免的,即使没有假设性的尝试对每个人进行洗脑以使其追随它; 无论如何,它是不可避免的,原因在于两项技术创新:

    1.飞机
    2. 互联网

    不存在这样的事情“不可避免”,除非有(((someONES)))与针对全球主义的反作用作斗争。

    • 回复: @Santoculto
  368. Santoculto 说:

    仅这两件事就使世界比几百年前小得多。 以这个速率; 阻止多元文化政策的有机实施无异于将手指放在水坝上。

    你的头脑简单[我正在使用你自己的傲慢毒药,抱歉]无法达到多重视角。 完全有可能生活在没有混合种族的多元文化社会中,但给定假设国家的政府有必要使用不同或新的策略来管理这种情况。

    话虽如此; 关于对多元文化主义传播的阴谋导向的解释,假设这是真的,我真的不认为它会在亚洲获得关注,我也不认为这是“建筑师”的意图混合剩下的世界。 相反,我认为他们唯一的目标是西方; 我认为他们这样做的原因是因为他们坚信业力的概念。

    我不知道如何开始。 首先,他们的行为方式非常微妙、隐蔽且相对懒惰。

    日本已有约 2 万移民。 韩国是远东地区文化最多元化的国家。 看来你已经迈出了第一步。 意大利也有一些移民,今天正变得越来越全球化。

    是的,如何解释爱尔兰**

    爱尔兰在其整个历史上一直是英国的种植园。 因果报应究竟在哪里** 请解释一下…

    还有瑞典**

    愤怒地参与西方殖民主义。

    还有波兰** 还有爱尔兰版..

    很好地尝试通过提出多元文化主义可能威胁远东这一事实来让我同意你的观点。 另外,我觉得很有趣的是,你试图指责“建筑师”想要追求全球统治,同时你又试图淡化西方(及其大量人口)实际上参与了统治整个世界的事实。世界。 唉,真是缺乏自我意识啊

    你正在自毁你的议论基础,也许是你的内心,太情绪化了,你在这里暴露了你的肮脏游戏……

    你让自己变得复杂起来,让我失去了今天的早晨……干得好。

    似乎是一种隐含的分裂叙述。 你西方,我东方。 有趣的是,如果自从全球主义第一次事件以来,我就不是以一种极其明智的方式来反对全球化的话。 不幸的是,通常情况下,一个无脑但精明的名人/寄生虫和政客之间没有什么区别。

    回到真实的具体世界。 我是西方人,所以***

    我是其中的一部分**

    我统治了整个世界,哈哈哈哈

  369. Santoculto 说:
    @GammaRay

    我相信,如果犹太人现在主宰西方世界,那只是因为他们作为业力的力量,通过西方所崇拜的事物(玛门和战争)来瓦解西方。 借用成吉思汗的一句话来更好地解释我对此事的感受:

    我是神的惩罚……如果你没有犯下大罪,神不会对你施以我这样的惩罚。

    吉伊埃苏斯·克莱斯特斯

    我不读这个……

    这里的不真实程度是如此之大……

    你还引用了成吉思汗的话,他是有史以来最凶残的混蛋之一……

    我无法反驳你的愚蠢

    你赢了!!!

    伽玛射线的奇幻世界……

    所以,是的,对你们来说,犹太人是天使的惩罚者……

    如果西方没有犯下大罪,那么它就不会像犹太人那样受到惩罚。

    呼吁神奇的思维,太棒了!

    下一步你会说白种人的星座没有好消息。

    犹太人正在统治西方,从最初的创造者欧洲白人手中夺取它,就像寄生黄蜂寄生并杀死蜘蛛以夺取她的网一样。

    这与实践正义无关,但用它来对付欧洲白人,这不是

    犹太人正在摧毁西方世界或从白人手中夺走它,
    西方世界是一个如此糟糕的地方,它统治、“殖民”和奴役全世界人民,
    白人“一直”沾沾自喜,

    犹太人是善良的=白人是邪恶的……

    有可能在没有糟糕的策略和不公正的情况下改革一个[大部分]糟糕的文明*我认为是的,这是可能的……这不是犹太人正在做的事情。

    最好的方法是

    诚实并真诚

    的确

    每个人都开始辩论过去的罪行,不仅针对他人,而且针对自己,

    非常清楚、客观、不捏造谎言,比如“没有人类,混血绝对比其他种族好”……

    • 回复: @Santoculto
  370. Santoculto 说:

    您仍然不明白的另一个大问题是

    所有社会,特别是大社会,规则和命令都是极其等级化的。 再说一次,我并不是说人们永远不会有罪恶感,而是精英是那些真正指挥社会的人,所以如果发生不好的事情,那些作为这个社会所有者的人必须首先受到指责,看起来很明显会被否认。

  371. Santoculto 说:
    @Santoculto

    因此,根据你的主观逻辑“由于中国人也有历史罪行和当前罪行,犹太人的下一步就是移民到中国并开始欺骗[更多]中国人”……

    • 回复: @GammaRay
    , @GammaRay
    , @GammaRay
  372. Factorize 说:

    这是一个很重要的观点,因为美国在过去几十年里一直试图与之互动的许多发展中国家缺乏发达的机构。 我怀疑事情正在向前发展,而许多人没有意识到帮助者和被帮助者之间已经出现了一种新的、更危险的关系。

    尽管如此,美国将球推进到这个领域对于人类来说是一个巨大的进步。 几个世纪以来,阿富汗一直抵制现代化力量。 看来他们还要继续抵抗未来,走向遥远的未来。 还没有; 现在,他们有可能走上与人类共同未来的道路。 整个世界正在走向一个共同的现实……除了撒哈拉以南非洲地区。

    亚洲的巨大成功分散了世界对非洲正在发生的事情的注意力。 联合国现在的目标是到 2030 年消除贫困。我会投票支持消除贫困,尽管投票不会改变当地的状况。

    我们如何应对1亿贫困人口面临的巨大发展挑战? 尽管非洲有些国家的艾滋病毒感染率高达 25%,但令人难以置信。 过去几十年来需要帮助的 SSA 以外的国家通常有 20 万左右的人口(并且正在朝着人口稳定的方向发展)。 它们给美国的资源造成了巨大的负担。然而,SSA的近1亿人口(人口稳定遥遥无期)很可能完全超出全世界集体努力的支持能力。

    GammaRay,是的,人口状况将会得到解决,尽管可能不是在本世纪。 人口统计是势头的最终体现。 由于涉及到如此多的惯性,因此不需要很高的洞察力就能预测未来几十年。 很明显,国防部还需要一段时间才能与我们合作而不是反对我们。

    我不确定让事情在没有干预的情况下发展是否完全现实。 几十年来,SSA 的局势一直在恶化。 非洲越来越无法养活自己。 到了某个时候,我们将会意识到,与其说21世纪属于亚洲,不如说它属于非洲。

    • 回复: @Santoculto
    , @GammaRay
  373. Truth 说:
    @Yan Shen

    美国对非洲的援助又是一个骗局。 这不是“援助”,而是“贷款”,各国必须偿还资金,并且负债累累,直到时间结束,就像我们在这里一样。 以色列,以及可以说是拥有许多美国军事基地的国家,是唯一获得美国“援助”的公司。

  374. GammaRay 说:
    @Santoculto

    冷静点,我现在不想在这场辩论中与你“战斗”。

    你分明是想和我战斗啊。 老实说,在你给我的大部分回复中,你都显得精神错乱、情绪不稳定。

    不,无论我在这里做什么,我从来没有意图操纵别人,我不需要它,因为我实际上所做的是相反的,我总是试图消除我认为像你们许多人一样不诚实或迟钝的评论。

    我真的不相信这一点。 事实是,你常常试图以极其客观的借口恶意地争论。 其次,你的英语不太好。 我犹豫是否要提出这个问题,因为我不想给人留下混蛋的印象,但既然你想推动这个问题; 事实是,您似乎难以阅读理解和理解我在回复您时所做的细微差别。 由于你无法正确理解我实际上提出的论点,你最终会误解一切,然后试图根据我实际上并没有提出的论点来攻击我。 不仅如此,我也经常无法弄清楚你想说什么。 总的来说,很难与你进行任何建设性的对话; 这并不是因为我们有很多分歧,而是因为你们的理解能力和沟通能力不是很好。

    回答这个问题:所有中国人或中国人民/华人种族都必须为在你们国家和国外犯下的暴行负责**

    如果中国人已经殖民了整个世界,并且中国民族/人民普遍从这种安排中受益,那么我认为将某种普遍的责任归咎于中国是适当的。 你似乎陷入了这样的想法,即我不愿意将我的标准均匀地应用于其他种族; 但事实并非如此。 我将全球殖民和种族灭绝归咎于西方/白人,因为这是事实,而不是因为我对白人作为白人有一些非理性的怨恨。

    我很抱歉,但你关于如何只应该指责精英(而不是执行他们命令的人)的论点并没有那么有说服力或令人信服。 我将使用您相同的逻辑来进行类比,以显示您的论点的本质弱点:

    假设有一个巴基斯坦儿童美容团伙在英国活动; 该儿童诱骗团伙的成员直接对英国未成年女性进行性虐待,而该团伙的主谋甚至从未碰过该团伙所诱骗的任何女性; 然而,他是从最高层资助和组织整个事情的。 依你的意见; 到底谁在道德上更有罪责呢? 是帮派头目还是他的追随者?

    我没有时间也没有耐心与像你这样的人辩论……从你误解我的评论那一刻开始[我从来没有说过只有精英才应该受到指责……或者你错误地阅读了我写的内容,或者你故意使用这个,好像我不会记得我写过什么]当你使用这样令人震惊和具有启发性的词“假设”来描述确实发生的事情时。

    如果你没有时间或耐心与我这样的人辩论; 那你为什么还要跟我争论呢? 行动胜于雄辩,不是吗?

    此外,您显然误解了“假设”一词在其使用上下文中的使用。 是的,我们已经知道实际上有一个巴基斯坦儿童美容团伙在英国运作; 没有人对此提出异议。 不过,这个事件是否真实发生,与我使用“假设”一词无关。 我在这种情况下使用“假设”一词只是为了强调我所概述的情况的修辞性质,仅此而已。 事实上,你误解了我写的内容,认为我试图“淡化”巴基斯坦儿童诱骗案确实存在的事实,这更多地与你自己缺乏理解有关,而不是与我试图淡化巴基斯坦儿童诱骗事件的暴行有关。确实发生了。

    你从未说过“只有精英才应该受到指责”,但这显然是你整个论点的主旨。 任何阅读我们交流的人都可以返回您之前的评论并验证这一点。 当你试图回避你确实试图提出“只有精英应该受到指责”这一客观事实时,你的争论是恶意的,而且是极其不诚实的。 如果这不是要点,那么我还应该如何解释你试图提出的论点呢? 我多次说过,精英和无产阶级都同样有可能受到指责(我认为这个立场是合理的),但不知为何你还想继续争论这个问题,因为你发现我的方式有问题。思维; 然后你又继续声称普通民众就像牛一样,没有代理能力,完全被精英所操纵。 你的话,不是我的话:

    再说一次,你没有明白的基本点,它也解释了你的“自己的”人。 大多数人或普通人都被驯服了[无论他们的智商如何,重要的是要注意]和/或无法改变他们的精英创造的世界,群众往往就像农场里分散的牛一样他们没有组织,而且往往也不太关心这个问题,世界是由一小群组织良好、精力充沛、实际上很聪明、邪恶或无情的人塑造的。

    你非常明显地试图提出“只有精英才应该受到指责”的论点,所以我不知道你为什么继续否认这一点。 如果你想否认这个指控,那么……你为什么不先停止这样的争论呢?

    我也在等待您对假想的巴基斯坦儿童美容团伙的类比的回应; 不要因为“假设”的场景而恼怒,直接回答问题即可。

    另一个宝贵的建议,减少你的傲慢……看来你被触发了……

    并不真地。 任何读过这篇文章的人都清楚,我写给你的内容显然激起了你的情绪,并让你感到不安。

    白人可以被集体指责,但这并不意味着他们应该被消灭……你能理解或接受吗*

    嗯,这就是我一直在说的。 你缺乏阅读理解能力来理解这一点并不是我的错。 白人必须应对几个世纪殖民统治带来的意想不到的后果,这并不是一个有争议的说法。 我也没有说他们应该被消灭。 我在之前给你的回复中已经向你解释过这一点,你不明白它不是我的错:

    前几代白人造成了这种混乱,而当代白人将不得不处理这个问题。

    我在这里写的内容没有任何情感修辞; 我所写的只是一个简单、客观、真实、符合事实的陈述。 我并不是在表达任何“我认为应该是真实的”的个人偏见,而只是对事件因果关系进行事实观察。 现代几代白人必须面对他们祖先的错误,这一事实是真实的,独立于任何表明现代几代白人应该受到这种惩罚的道德/哲学论点。 你误解了所提出的论点,这一事实取决于你,而不是我。

    如果这次谈话的主题是关于任何其他犯有种族灭绝/暴行的种族/群体,那么我客观地分析他们的罪责和潜在的罪责是没有问题的。

    因为他们现在正在遭受种族灭绝。

    这就是我所说的关于你无法理解我所提出的论点的一个很好的例子。 我明确指的是正在犯下种族灭绝/暴行的种族/群体,但你显然误解了我所说的,因为我在谈论人们被种族灭绝。 我认为考虑到你的理解水平较低; 也许不值得继续与您进行对话。 这并不是因为我们彼此不同意,而是因为几乎不可能与您进行令人信服的对话,因为您的英语水平不够高,无法实际讨论手头的问题,而不是您不断地误解我的事情我正在写。

    我们遇到麻烦是因为我们的方法似乎不同。 你的方法似乎更加二元化,而我的方法倾向于通过“多重”视角。 我将我对这个问题的看法分成许多部分或部分,而不是创建一个单一的叙述。

    不完全是,我们唯一的麻烦是因为你的英语不够好,无法参加这样复杂的讨论。 话虽如此,我的做法实际上是相当合理和全面的。 我对这种情况的解读是,你更喜欢恶意争论,并有一个隐藏的议程(很可能是反犹太主义的),但试图伪装成理性和富有同情心,而实际上你想做的只是指出无论任何其他类型的论点(不指责犹太人),你对犹太人的指责都可以具有同等甚至更多的解释力。

    我并不是说,当你试图稻草人化时,西方人在任何情况下都不能受到西方殖民/剥削/任何邪恶和愚蠢的指责。

    那我们还争论什么呢? 我的立场很简单。 我认为,当谈到殖民化的指责时,西方负有最大的责任,更重要的是,西方目前正在通过意想不到的事件转变而收获其为殖民化播下的种子,这些事件目前正在对西方产生负面影响(多元文化主义) 、全球化和大规模移民)。 我在这里的基本论点非常简单,其真理性是不言而喻的。 我不明白我的基本论点怎么会被误解为不公平或不合逻辑。

  375. GammaRay 说:
    @Santoculto

    显然,我们应该担心白人殖民他人(并且对种族灭绝方面也应特别彻底),因为不言而喻,白人更熟练/最倾向于殖民和种族灭绝其他人(白人殖民/试图殖民整个世界并且种族灭绝了无数的土著文化)。 其他人也犯有殖民和种族灭绝罪,但白人做得最好,而且许多人(实际上是大多数 WN/另类右翼分子)都愚蠢到公开承认这一点并为此感到自豪,尽管这实际上是在起诉他们道德/伦理水平。

    是的,如果我不同意你的观点,请告诉我。

    好的,我同意。 这是你最初写的:

    是的,我并不是说我们需要停止责怪白人,而是停止只责怪他们,不仅因为根据你的道德标准,所有人在他们历史的某个时刻都倾向于有罪,而且还因为人类群体、个体往往既是无辜的,又是有罪的。

    我写给你的内容是直接回应,解释了为什么白人比其他人承担更高的殖民主义/种族灭绝罪行负担,这仅仅是因为他们犯下的种族灭绝/殖民主义的规模和程度比其他人大得多从历史上讲。 我对你的回复确实没有什么争议; 所以我不太确定为什么你觉得有必要解决这个问题。 你想声称每个种族都有可能做坏事,所以我们不应该只责怪白人;而应该责怪白人。 然后我继续清楚地解释了为什么我在殖民主义的具体背景下不同意这一观点,以及为什么我们应该让白人/西方对殖民主义负有特别责任,因为这种罪行的规模和程度巨大(这确实是一个世界范围的罪行)持续了几个世纪的犯罪活动)。 如果您对我回复您某件事有疑问,那为什么还要费力写信给我呢?

    不过,如果殖民者足够明智的话,我并不认为殖民化绝对是最糟糕的事情; 如果其他人可以成为你的盟友或从根本上成为你的良性合作者,你就不需要消灭或征服他们,这看起来很理想主义和天真,但我认为事实并非如此。

    错错错。 你对此完全错误。 殖民化是一种犯罪,无论是谁干的,都是犯罪行为。 白色、黑色或黄色。 良性合作与“殖民”有很大不同。 事实上,你试图以某种方式为殖民化辩护,这很大程度上说明了你自己的心态。

    就像大多数保守派一样,无论其种族如何,请记住,可以通过征服、替代或任何其他纯粹邪恶的策略在自己的领土上“殖民”,例如在西藏发生的事情。

    首先我不是中国人

    其次,西藏发生的事情是错误的(假设我们听到的关于西藏发生的事情一开始就是事实)

    第三,虽然对西藏的殖民化是错误的,但它比欧洲殖民者几个世纪以来对整个世界所做的事情要严重得多。 当你深知西藏的逐步殖民化与欧洲殖民者对全世界进行的暴力种族灭绝和殖民化难以相提并论时,不要不诚实地试图对西藏进行反驳。

    第四,中国殖民西藏的主要原因是为了消除西藏被中央情报局操纵并用作反中国政府集结地的可能性。 在西藏问题上,美国实质上是在动中国的手。 我觉得,如果不是美国不断试图破坏中国政府(以及世界各地无数其他主权政府)的稳定,那么中国政府对西藏将会采取更加自由的政策。 当然,假设所有关于中国政府对待西藏的负面指控实际上都是真实的。

    洗脑和欺骗是非常相似的事情,即使不是基本相同。 [洗脑是欺骗的隐喻]。 因为人们更容易成为小狗,所以他们被训练[和自我训练]相信:
    犹太人是受迫害的天使。
    黑人也是如此,他们绝对是受害者,他们的大部分智力和行为“缺陷”都是直接由于西方“殖民化”或占领或系统性种族主义造成的。

    你知道 WN/另类右翼也有自己的洗脑形式吗? 除了他们试图将其他人描绘成非人类,然后试图表现得好像白人是这个星球上唯一有权生存的人一样。 更滑稽的是常见的西方国家/另类右翼错觉,认为西方在历史上没有做错任何事,而他们现在正受到不公平的受害者哈哈。 更可悲的是,许多WN/另类右翼人士不加批判地接受了这一点,实际上开始相信自己的妄想和洗脑。

    是否有某个建筑师试图将多元文化主义推向全世界? 可能。

    如果你还处于这个水平……当韩国所有的教科书都在“教导”多元文化有多么伟大时……

    是的,如果这是真的,我不会感到惊讶。 虽然是因为某种阴谋,还是因为经济因素甚至文化影响(即美国)? 另外,你去过韩国吗? 你真的和那么多韩国本土人谈论过这个问题的严重性吗? 或者这只是你在一些不起眼的另类右翼论坛上读到的东西? 我注意到,许多 WN/另类右翼人士想要对那些他们只是道听途说的事情感到兴奋,而这些事情发生在外国,甚至与他们无关。 这有点可笑不是吗?

    关于韩国​​的多元文化主义,我的印象是,这只是一个问题,因为城市化的韩国女性不想嫁给农村农民,因此农民从越南和菲律宾进口妻子。 就是这样。 这与其说是有什么巨大的阴谋,不如说是一些相当平凡的因素。 顺便说一句,只是给你一个提示,极端的反犹太主义会腐蚀批判性思维能力😉

    不存在这样的事情“不可避免”,除非有(((someONES)))与针对全球主义的反作用作斗争。

    是的,有这样的事情是不可避免的。 飞机和互联网的存在使得多元文化不可避免; 然而,多元文化主义的确切程度仍有争议。 话虽这么说,不幸的是,对于 WN/另类右翼极端民族主义者来说,如今不可能完全将自己与世界其他地方隔离开来。 即使像朝鲜这样的极权国家也逐渐屈服于全球影响(特别是来自其南部邻国),是什么让您认为 WN/另类右翼分子会过得更好?

    仅这两件事就使世界比几百年前小得多。 以这个速率; 阻止多元文化政策的有机实施无异于将手指放在水坝上。

    你的头脑简单[我正在使用你自己的傲慢毒药,抱歉]无法达到多重视角。 完全有可能生活在没有混合种族的多元文化社会中,但给定假设国家的政府有必要使用不同或新的策略来管理这种情况。

    嗯……我猜? 您似乎相信,为了妥善管理一个多元文化社会,政府需要阻止人们种族混合。 哇,这听起来真的很吸引人。 这听起来几乎像是某种“种姓制度”。 不过,祝社会上的其他人都同意这一点吧; 我认为大多数人不会善待互联网上的另类右翼陌生人试图立法是否允许他们与自己的身体混合比赛。 你还指责我傲慢,哈哈。 事实上,你认为人们会同意政府立法,这一事实表明你的思维过程与其他人有多么脱节。 你在/pol/上花费了太多时间,你的自闭症正在向朋友显现

    话虽如此; 关于对多元文化主义传播的阴谋导向的解释,假设这是真的,我真的不认为它会在亚洲获得关注,我也不认为这是“建筑师”的意图混合剩下的世界。 相反,我认为他们唯一的目标是西方; 我认为他们这样做的原因是因为他们坚信业力的概念。

    日本已有约 2 万移民。 韩国是远东地区文化最多元化的国家。 看来你已经迈出了第一步。 意大利也有一些移民,今天正变得越来越全球化。

    是的,这些国家有多元文化; 但这是否是因为什么阴谋? 或者是由于世俗因素(例如我解释的有关韩国的因素)? 此外,这些趋势会减少、停滞还是增加? 这一点目前还不清楚,因此目前认为多元文化主义将像西方那样在东亚真正获得牵引力并起飞的假设是不正确的。

  376. GammaRay 说:
    @Santoculto

    是的,如何解释爱尔兰**

    爱尔兰在其整个历史上一直是英国的种植园。 因果报应究竟在哪里** 请解释一下…

    我说过“他们”似乎坚信业力/回报/复仇等概念。我没有直接指出西方发生的事情是业力的结果。 阅读理解我的朋友...

    话虽这么说,实际上,在美国建国期间,有很大一部分爱尔兰人移民到美国(以及其他殖民地); 我毫不怀疑,这些爱尔兰移民中的许多人可能在帮助直接将美洲原住民从他们祖先的土地上赶走以及帮助欧洲人重新居住在这些土地上发挥了作用。 正如爱尔兰人经常利用大英帝国提供的机会选择成为世界各地的移民一样,爱尔兰现在也可以接受移民。

    我的建议是,你应该让爱尔兰人在这个问题上为自己说话,而不是自愿代表他们说话。 当另类右派试图代表他们知之甚少、从未涉足过的国家发言时,他们看起来真的很愚蠢,而他们对这些国家唯一了解的就是极右极端主义互联网出版物上的危言耸听的新闻文章。

    还有瑞典**

    瑞典实际上拥有殖民地,并有限地参与了奴隶贸易。 瑞典积极参与了 17 世纪和 18 世纪欧洲争夺美洲新殖民地所造成的混乱; 因此,说瑞典与殖民化没有任何关系实际上是不正确的。 如果你费心去做自己的研究,而不只是鹦鹉学舌地重复你在各种另类右翼回声室中听到的事情,你就会知道这一点。 我对西方在殖民主义方面的罪责的印象是,许多欧洲国家以某种或大或小的方式、直接或间接地参与了欧洲全球殖民化计划。 殖民主义显然是一个泛欧洲(也许主要是西欧?)的事业。

    还有波兰** 还有爱尔兰版..

    波兰几乎没有多元文化。 我找不到他们采取大规模移民政策的任何证据。 很困惑你为什么提起它们。 毫无疑问,你可能从一些另类右翼的回声室中获得了关于波兰的错误信息,并且自动相信现在波兰正在被移民占领,即使你从未踏足过这个国家,也不了解那里的实际情况。

    我相信,如果犹太人现在主宰西方世界,那只是因为他们作为业力的力量,通过西方所崇拜的事物(玛门和战争)来瓦解西方。 借用成吉思汗的一句话来更好地解释我对此事的感受:
    我是神的惩罚……如果你没有犯下大罪,神不会对你施以我这样的惩罚。

    吉伊埃苏斯·克莱斯特斯
    我不读这个……
    这里的不真实程度是如此之大……

    再次表明您的(英语)阅读理解水平较低。 很明显,我在这里做了一个诗意的、比喻性的陈述,并不打算完全按照字面意义来理解。 当然,这完全超出了你的想象。

    只是重申我原来的观点; 从我迄今为止所看到的一切来看,如果犹太人真的有瓦解西方的阴谋,那么犹太人似乎主要是通过推行利用西方的颓废、傲慢、崇拜玛门和好战欲望的政策来实现这一目标的。 因此,犹太人似乎是在利用西方文化本身的缺陷来象征性地追究西方的业力。

    你还引用了成吉思汗的话,他是有史以来最凶残的混蛋之一……

    是的,这是真的,我不否认成吉思汗可能不是一个很好的人。 话虽如此,我发现他的这句话离题且高度相关,所以我使用了它。 你对这个问题看得太多了。

    犹太人正在统治西方,从最初的创造者欧洲白人手中夺取它,就像寄生黄蜂寄生并杀死蜘蛛以夺取她的网一样。

    和? 在我看来,犹太人主宰西方似乎是为了与世界其他地区分享,而不是仅仅利用它来为自己谋取利益。 在我看来,这是崇高的,甚至是合理的,因为西方认为从其他人手中夺走土著土地是合适的。 此外,为什么你继续使用犹太人是寄生黄蜂的类比,而同样的类比可以很容易地用来指代欧洲人和他们篡夺和寄生世界各地土著人民的漫长而血腥的历史? 你选择使用这个类比的事实表明你的自我意识是多么的少。

    话虽如此; 无论犹太人是否参与其中,这都不会改变西方仍然把目前的事态归咎于自己的事实。 如果不是西方首先实现了全球化,犹太人不可能实现这一切。 就像我之前说过的,不能把所有坏事都归咎于犹太人,相反,犹太人似乎擅长利用白人创办的企业,然后将其用于自己的目的; 无论这些目的是什么。 在这种情况下,犹太人似乎正在做崇高的事情,他们正在利用全球化,在世界上所有种族之间重新分配权力、财富和土地,而不是允许所有这些权力、财富和土地集中于一个种族(他们一开始就以不公正的方式获得了所有这些)

    这与实践正义无关,但用它来对付欧洲白人,这不是
    犹太人正在摧毁西方世界或从白人手中夺走它,
    西方世界是一个如此糟糕的地方,它统治、“殖民”和奴役全世界人民,
    犹太人是善良的=白人是邪恶的……

    实际上; 如果你真的想去那里,那么我们就去。 就在 20 世纪初,所有观察家都认为西方将最终接管整个世界,而且没有一个非白人国家能够严重威胁这一安排(确实如此)日本正在迅速现代化,但日本本身无法扰乱整个全球西方世界秩序)。 当时,西方殖民地遍布全球,大英帝国本身尤其强大,正处于鼎盛时期。 如果不是第一次世界大战和第二次世界大战的灾难(根据阴谋论者的说法,这是由犹太人造成的),那么白人很可能会继续不受阻碍地努力逐步净化和取代世界上的其他一切种族,以使世界成为只有白人生存的地方。

    更重要的是,当时白人大约占世界人口的1/3,所以如果白人愿意的话,他们很容易对世界上另外2/3的人口进行种族灭绝,从种种迹象来看,似乎是这样的。这或多或少是他们随着时间的推移打算做的事情(如果他们对待美洲原住民的方式预示着即将发生的事情)。 因此,实际上确实显得犹太人是善良的,白人是邪恶的。 话虽这么说,我不同意这样极其广泛的概括。 我遇到了太多善良的白人(以及来自各个种族的好人),无法将整个种族视为邪恶。 但是,如果我们真的想客观地看待这种情况,并且如果你坚持使用黑/白启发法,那么是的,白人似乎对地球上所​​有其他种族来说通常都是一种邪恶的存在

    有可能在没有糟糕的策略和不公正的情况下改革一个[大部分]糟糕的文明*我认为是的,这是可能的……这不是犹太人正在做的事情。
    最好的方法是
    诚实并真诚

    如果诚实和真诚是最好的方式,那么你应该以身作则。

    所有社会,特别是大社会,规则和命令都是极其等级化的。 再说一次,我并不是说人们永远不会有罪恶感,而是精英是那些真正指挥社会的人,所以如果发生不好的事情,那些作为这个社会所有者的人必须首先受到指责,看起来很明显会被否认。

    错误的。 我责怪两者。 你只是试图提出主要指责精英的论点,因为你暗地里想将犹太人等同于精英。 真的就是这么简单; 你显然想把矛头指向假想的犹太精英,而不是简单地承认有时大众可以拥有代理权,并且无​​论精英是否参与,他们的行为都可能在道德上是错误的。

    因此,根据你的主观逻辑“由于中国人也有历史罪行和当前罪行,犹太人的下一步就是移民到中国并开始欺骗[更多]中国人”……

    不,不是真的。 中国人民从未在全球范围内对人民进行种族灭绝和殖民统治; 因此,假设犹太人是地球上所有被压迫种族的捍卫者(无论如何这是我对他们的理解),那么为什么犹太人会试图渗透并摧毁中国呢?

  377. GammaRay 说:
    @Factorize

    我真的不明白非洲的问题是什么。 如果非洲人口过剩并处于崩溃的边缘,那么为什么这会成为世界其他地区的问题呢? 实际上,我不明白这里的问题是什么。 您介意概述一些场景,以便我更好地了解您在说什么吗?

    老实说,我认为在后西方世界中,非洲的问题仍将是非洲的问题。 如果非洲崩溃了,而没有人真正能够帮助它(或不愿意),那么最糟糕的情况会是什么? 只有当人们想让非洲成为问题时,非洲才会成为问题。 我知道另类右翼有很多末日色情片,说非洲将变得人口过剩,整个世界都会充满非洲人; 但从现实意义上来说,我并没有真正看到这种情况发生。 在当今,将大量非洲难民/移民等从非洲转移到其他国家将是一场政治和后勤噩梦,但在后西方世界尤其如此。

  378. Factorize 说:

    GammaRay,我认为想象一个在某种程度上不共享的人类未来是不完全现实的。 很难忽视的是,一百年来非洲出现的最改变世界的事情就是艾滋病。

    许多人没有意识到,非洲正在走向更加严重的发展危机。 他们如何能够可持续地养活自己已经不再明显。

    新兴的智商增强技术对发达国家来说可能是一件幸事。 通过基因工程使人拥有几百点的智商肯定会改变我们的生活。 大规模 GWAS 即将发布,其中描述了 3000 个教育程度 SNP。 增强人类的时代正在迅速临近。

    然而,人们完全有理由担心如果在不太理想的情况下发生这种增强可能会发生什么。 例如,考虑一下在撒哈拉以南非洲国家出生的具有基因增强智商的儿童,该国家的冲突持续不断,艾滋病患病率高达 25%。

    • 回复: @GammaRay
  379. GammaRay 说:
    @Factorize

    抱歉,但我真的觉得这种推理没有说服力。 是的,世界确实正在变得越来越小。 但我并不认为非洲自身的本地困境会(肯定)影响世界其他地区。 在后西方世界,如果非洲人无法养活自己,那又怎样? 我认为,从更广泛的角度来看,不会发生太多事情。

    话虽这么说,我可以看到这可能会如何导致欧洲的难民危机; 但如果这就是我们(真正)担心的问题,那么我们就直接解决这个问题,而不是将情况推断为全球灾难。

    恕我直言,我仍然没有将非洲人口/饥饿问题与全球灾难联系起来。 我真的不认为除了在另类右翼中如此流行的黑人世界的戏剧性末日色情片之外,对此还有什么强有力的论据。

  380. Factorize 说:

    Gamma,谢谢你的反驳,因为它给了我一个机会来确认我想要传达的内容。

    确实,世界在很大程度上忽视了非洲的困境。 在非洲以外,各国的内政也受到了实质性的干预。 俗话说,我们现在已经排干了沼泽。 现在剩下的似乎完全无法管理。

    我认为我关于一个智商提高和发展挑战巨大的世界的建议需要重申。 约翰·冯·诺依曼是 20 世纪最聪明的人之一,他帮助开发了计算机和原子武器,并意识到随着计算机的出现,奇点将会出现。 他还高度赞成对苏联进行无端的首次核打击

    智商的基因增强将改变人类,但也会有风险。 我们需要从现在开始为即将到来的超人类创造条件,让他们不会有破坏我们世界的动机。 科技甚至赋予普通公民巨大的力量。 我想,如果我下定决心,我可以创造出一个基因驱动器,让世界充满 50 英尺的蚂蚁。 我们需要确保我们现在创造的世界充满爱和同情,而不是仇恨。 如果我们不这样做,就很难想象我们有未来。

    从这个角度看,非洲就让人感到警惕。 在非洲一些国家,25%的成年人感染了艾滋病毒! 其中许多国家的发展水平很低。 将天才级智商引入这样的环境可能会带来严重的全球安全风险。 冯·诺依曼的智商接近 200,据我所知,最新的 EA GWAS 已经揭示了 400 的智商。 我不想看到一个智商400、充满仇恨的孩子会发明什么武器系统。

  381. Factorize 说:

    我对中国最近对美国关税的反应感到有点惊讶。
    这并不令人震惊,也不令人吃惊,甚至也不太令人惊讶……有人有同义词库吗?

    请评论!

  382. Factorize 说:

    是否有针对中国或其他东亚样本的大规模 IQ GWAS 研究发表?

    该博客引用了美国 40 年前的心理测量研究。 使用最近发现的 IQ SNP 是否可以清楚地证明国家因智商而存在差异的说法?

  383. @Yan Shen

    严,

    写得很好的文章,有详细的,我应该说或多或少准确的观点,一次又一次与数据兼容。 我也在我的博客上写过类似的内容。

    具有讽刺意味的是,虽然现在东亚人被认为偏向于 STEM,但东亚文明在理论科学的发展方面却相当欠缺,尽管这并不完全是由于不平衡的认知状况。 虽然总体而言,我发现 STEM 领域的东亚人普遍更具体、更计算、更空间,但也出现了相当多的人被认为是更深入、更理论化的思想家,V 非常高。对此,我有些惊讶地了解到,例如,东亚人(主要是日本人,但也有杨、李)实际上对 20 世纪后期的理论物理学做出了多少贡献。 据我所知,中国老一代的顶尖数学家和理论物理学家都非常精通文化和语言。

    顺便说一句,我在许的博客上看到了你关于歧视亚裔美国人的评论,我肯定能认同一些。 尽管他们可能很愿意,但他们并没有真正处于经济或政治地位来反对它,尽管这种情况正在改变。 我确实相信,如果亚裔美国人能够少关心别人的想法,更多地按照自己的自然倾向发展,他们会做得更好,尤其是在高层。 然而,这与现实相去甚远,尤其是考虑到亚洲配额和某些当权者的偏见。

    您写了有关中国 STEM 的文章。 在过去的几十年里,它似乎急剧上升。 在工程和技术应用方面,它已经达到或接近世界一流水平。 虽然在基础科学研究方面,与日本不同,这似乎还很遥远,但这又只是时间问题。 现代科学或多或少是从希腊人开始的西方传统,虽然中国人确实有才华,但发展正确的传统和文化需要几代人的时间。 最终,华裔应该在中国进行最好的科学研究,而不是在美国的机构中,后者只能容纳这么多人。 我真诚地相信科学思想流派的更多多样性和独立性将有利于科学和整个世界。

    同时,感谢Ron Unz为Yan提供了这样一个场所来宣扬他的深思熟虑的想法。

    • 回复: @Yan Shen
  384. Yan Shen 说:
    @gmachine1729

    感谢您的评论!

    在应用方面,在工程和技术方面,它已经达到或接近世界一流水平。

    目前它在工程和技术方面是最强的,但在物理科学方面它距离世界一流也并不遥远,与物理相比,化学可能更是如此。

    我认为我在这篇文章中想争论的最重要的事情是,我认为东亚不会在基于数学/语言分裂的软件或生命科学方面与英语圈竞争。

    归根结底,华人应该在中国做他们最好的科学,而不是在只能容纳这么多人的美国机构。

    http://www.ecns.cn/2018/04-08/298317.shtml

    但根据教育部最新公布的数据,540,000年中国赴海外留学人数约2016万人,回国约430,000万人。 与2011年相比,2016年中国留学人数增长37.61%,海归人数增长56.95%。

    “在我看来,中国在新技术方面的机会比美国多,”在“千人计划”下在斯坦福获得学士和博士学位的博士后学者雷说。

    “在过去的几年里,我一直致力于一个基于具有超高表面积的新型石墨碳材料的研究项目,”研究有机电子器件制造的雷说。

    “在电子和技术供应链中,有一个非常大且经常不断增长的份额位于中国。

    “因此,中国是让我的项目投入生产的最佳地点,政府对创新和研究的大力支持以及更多的投资机会。”

    回国人数似乎正在增加。 我认为随着中国的进一步崛起,来美国留学的人数也将不可避免地减少。 因此,这是一个将在未来几十年内自行解决的“问题”。

    • 回复: @gmachine1729
  385. @Yan Shen

    严,

    我在我的以下博客文章中对您的文章发表了更多评论,您可能会觉得这很有趣: https://gmachine1729.com/2018/04/23/back-to-blogging/. 此外,非常欢迎您(这意味着请这样做)与 Ron Unz 本人分享。 我当然不介意认识他。 🙂我非常尊重他的智力诚实。

  386. Edward 说:

    严,

    非常感谢你写这篇文章。 我或多或少同意你所说的。 虽然不应低估智力 (g) 的一般因素的重要性,但也必须考虑特定的认知能力。 我认为它如下:高 g 因子是整体生活成功的最佳预测指标,但特定的认知能力可以预测成功的类型。

    正如数学早熟青年研究 (SMPY) 所记录的,虽然所有参与者都在人口的前 1% 中,但相对于空间能力而言,语言能力较高的人往往会进入法律和商业等领域,而那些相对于语言能力具有更高空间能力的人倾向于进入数学、物理和工程专业。 全能型人才倾向于进入医学等领域。 事实上,我对 SMPY 的解读是,语言-空间分割可能比语言-数学分割更重要。

    说到陶特伦的具体点,他似乎还是有很多语言能力的。 我们可以仅从他的博客中得出这一点。 然而,有更好的证据支持我的断言:SMPY 的推动者 Julian Stanley 在 2006 年一篇关于 Terence Tao 和 Leonhard Ng 的评论文章中写道,Tao 的“SAT 语言成绩比 [8 岁时] 低得多” ] 但在接下来的几年里迅速攀升至 700 年代。”

    换句话说,在语言能力方面,陶也比同龄人领先了好几年。

    Sumber: http://journals.sagepub.com/doi/abs/10.1177/001698620605000404

  387. Anonymous [又名“杰森1423”] 说:

    嗯,如果测试的高数学能力是在数学和物理科学方面表现出色(在所有方面不仅仅是在测试中)并且东亚人在测试中具有更高的数学能力,那么这个论点并没有多大意义,那么您会期望看到东亚人完全主导了数学、科学和工程的所有领域,因为他们的文化繁荣至少比大多数欧洲文化早 1000 年。 例如,欧洲在数学、物理、化学、工程学方面取得的进步极少,在医学和文学方面取得了重大成就。

    这不是我们所看到的。 欧洲人(尤其是欧洲犹太人)在数学、物理、化学和工程学的最高成就水平上占主导地位,尽管人口少得多,但数百年来一直如此。 只有在向亚洲大规模转移知识和技术之后,东亚人才开始崛起。

    几乎所有在数学和物理科学领域最重大的进步都是由欧洲人实现的。

    至于东亚人在西方取得的成就,很大程度上是移民到西方的那种移民因素,即更聪明的东亚人。 如果您的人口是您要迁移到的东道国的 4-6 倍,并且更聪明的人倾向于迁移,那么他们平均比当地人做得更好也就不足为奇了,印度人也是如此。 留在原籍国的人与移徙者是不同的群体。

    智商测试是好的和有用的,但不是优秀和完美的。 真正的证据是你的文化所取得的造福人类的成就,而不是你创造的专利数量(例如,有许多无用的专利)。 改进计算机的某些元素是一回事,发明计算(例如)完全是另一回事。 此外,那些在测试中取得好成绩的人通常不会将这种成就转化为现实世界的成就,反之亦然。 智能定义太差了,应该是我们的首要任务,什么是智能? 它与智商的关系有多密切。

    我认为为什么某些群体在某些领域做得更好的真正答案将比数学/语言分裂复杂得多,听起来太简洁了,不适合现实世界,理论也有太多例外保持水。

  388. @Joe Hide

    任何写作的人

    请继续为 we 语言认知先进…

    是迟钝而不是“先进”。

  389. @Daniel Chieh

    是否需要不是问题。 人工智能以光速进行通信。 人类以音速前进。

  390. anonymous[135]• 免责声明 说:
    @anony-mouse

    哈!,作者的数学/语言分裂扭曲了语言。

  391. Anonymous[402]• 免责声明 说:

    到目前为止,“语言智商”根本不是智商测量(或“r”),因为它在很大程度上取决于词汇量。 如果可能的话,智商测量应该将所有其他非原始智商拐杖降到零,但这里我们有一个学习词汇量的污点结果,坦率地说,这是淫秽的。

    因此,我们得到一个书虫或 SJW 沙鼠的表现优于数学/工程/物理/IT 天才,纯粹是因为知道“mansplaining”是什么意思。

    LOL

  392. Blankaerd 说:

    有趣的文章,如果有点长。 本来可以浓缩更多,同时仍然得到相同的观点。 尽管如此,我还是发现了一些问题,其中一些问题在本质上更为普遍,因此不应过于个人化

    1. 我从来没有真正理解为什么亚洲人被分成子类别而白人不是。 东亚人虽然数量多得多,但基因库要小得多。 白人是一个更广泛的群体,当你像你和其他人对亚洲人所做的那样将他们分开时,你会看到智商的分布更广泛,其中(西北欧血统的人)最高,其次是那些南欧血统,其次是东欧血统,这里有一些异常值,那里有异常。 如果不划分亚洲人,白人几乎在所有方面都会胜过亚洲人。
    2. 东亚钟形曲线更高,但更窄,这意味着频谱两端的异常值更少。 长期以来,白人一直遵循“天才策略”,即他们倾向于进行更多的远亲繁殖,从而产生更多的天才。 您在文章中提到了 OLED 之父,但与弹道导弹或蒸汽机的发明相比,OLED 似乎是一个相对较小的发明。 亚洲人似乎改进了现有的技术,但似乎并没有自己发明很多新东西。
    3. 自工业革命以来,西方的智力一直在下降。 那件事标志着达尔文式的情报选择的结束。 福利国家允许突变存活并显着提高愚蠢人的繁殖成功率。 此外,聪明的人更有可能从避孕中受益(更高的智力与更高的冲动控制相关,这意味着更少的不想要的婴儿)在女权主义和大规模移民的时代,我们可以预期智力以更快的速度下降。 过去白人的智商要高得多,如果我们可以测试一个 110 年前来自大英帝国的普通白人,我们可能会发现他的平均智商在 120-XNUMX 之间。 这种下降肯定也会影响中国、韩国和日本。 也就是说,除非他们以某种形式实施优生学计划,我认为这是西方绝对需要的,以防止未来的情报危机。
    4. 成为天才可能与发育迟缓有关,例如由于出生过早。 艾萨克牛顿是这方面最显着的例子。 爱德华·达顿(Edward Dutton)提出的一种理论认为,由于出生得太早,牛顿大脑的某些部分比其他部分发育得更快,导致他在某一方面是绝对的天才,而在其他方面则是绝对的迟钝者。 这可能是天才中的一个趋势:他们往往患有自闭症,他们往往很少或没有生育成功,他们往往在某些领域表现出色,而在其他方面却很差。 即使在正常情况下,空间能力智商高于平均水平,也可能意味着您的语言能力智商也高于平均水平。 但在孩提时代遭受发育迟缓后,它可能成为规则的例外。

  393. 对智商存在误解。 这是我们发明的一个数字。

    “智力”只是思考的能力。 没有特别的方向。 我们尝试做广泛的事情来定义这种获取数字的能力。 想什么不是问题。 有些人比其他人更能思考任何或许多事情。 然后,我们尝试对它与其他 100 个人的比较进行评分。

    忘记刻板印象的数字,能够比其他人思考“更好”的事情的人通常会更聪明,当然在给定的环境中也是如此。 统计数据表明,一般来说,这个人对任何事情都会比其他人“思考”得更好。 因为它不是关于主题,而是关于思考的能力。

    Gr

当前评论者
说:

发表评论-对超过两周的文章发表评论,将在质量和语气上进行更严格的判断


 记得 我的信息为什么?
 电子邮件回复我的评论
$
提交的评论已被许可给 Unz评论 并可以由后者自行决定在其他地方重新发布
在翻译模式下禁用评论
通过RSS订阅此评论主题 通过 RSS 订阅所有严慎评论